eureka 2003

186
CONTEÚDO XIV OLIMPÍADA BRASILEIRA DE MATEMÁTICA 2 Problemas e Soluções da Primeira Fase XIV OLIMPÍADA BRASILEIRA DE MATEMÁTICA 14 Problemas e Soluções da Segunda Fase XIV OLIMPÍADA BRASILEIRA DE MATEMÁTICA 25 Problemas e Soluções da Terceira Fase XIV OLIMPÍADA BRASILEIRA DE MATEMÁTICA 44 Problemas e Soluções da Primeira Fase - Nível Universitário XIV OLIMPÍADA BRASILEIRA DE MATEMÁTICA 49 Problemas e Soluções da Segunda Fase - Nível Universitário XIV OLIMPÍADA BRASILEIRA DE MATEMÁTICA 58 Premiados AGENDA OLÍMPICA 62

Upload: demi-de

Post on 22-Mar-2016

320 views

Category:

Documents


25 download

DESCRIPTION

XIV OLIMPÍADA BRASILEIRA DE MATEMÁTICA 14 Problemas e Soluções da Segunda Fase XIV OLIMPÍADA BRASILEIRA DE MATEMÁTICA 2 Problemas e Soluções da Primeira Fase XIV OLIMPÍADA BRASILEIRA DE MATEMÁTICA 25 Problemas e Soluções da Terceira Fase CONTEÚDO XIV OLIMPÍADA BRASILEIRA DE MATEMÁTICA 49 Problemas e Soluções da Segunda Fase - Nível Universitário XIV OLIMPÍADA BRASILEIRA DE MATEMÁTICA 44 Problemas e Soluções da Primeira Fase - Nível Universitário

TRANSCRIPT

Page 1: Eureka 2003

CONTEÚDO

XIV OLIMPÍADA BRASILEIRA DE MATEMÁTICA 2Problemas e Soluções da Primeira Fase

XIV OLIMPÍADA BRASILEIRA DE MATEMÁTICA 14Problemas e Soluções da Segunda Fase

XIV OLIMPÍADA BRASILEIRA DE MATEMÁTICA 25Problemas e Soluções da Terceira Fase

XIV OLIMPÍADA BRASILEIRA DE MATEMÁTICA 44Problemas e Soluções da Primeira Fase - Nível Universitário

XIV OLIMPÍADA BRASILEIRA DE MATEMÁTICA 49Problemas e Soluções da Segunda Fase - Nível Universitário

XIV OLIMPÍADA BRASILEIRA DE MATEMÁTICA 58Premiados

AGENDA OLÍMPICA 62

Page 2: Eureka 2003

Sociedade Brasileira de Matemática

EUREKA! N°16, 2003

2

XXIV OLIMPÍADA BRASILEIRA DE MATEMÁTICAProblemas e Soluções da Primeira Fase

PROBLEMAS – NÍVEL 1

1. A razão 28

84

)4(

)2( é igual a:

A) 4

1 B) 2

1 C) 1 D) 2 E) 8

2. Num armazém foram empilhadas embalagens cúbicas conforme mostra a figuraa seguir. Se cada caixa pesa 25 kg, quanto pesa toda a pilha?

A) 300 kg B) 325 kg C) 350 kg D) 375 kg E) 400 kg

3. Na balança a seguir temos pesadas bolas de chumbo, todas iguais, e levessaquinhos de plástico, todos com a mesma quantidade de bolinhas, iguais às queestão fora dos mesmos. Quantas bolinhas há em cada saquinho?

aa

a

a

a

a

a

A) 1 B) 2 C) 3 D) 5 E) 6

4. Escreva os números inteiros de 1 a 9 nos nove quadradinhos, de forma que assomas dos quatro números em cada uma das pás da “hélice” sejam iguais e demaior valor possível. Esse valor é:

A) 23 B) 22 C) 21 D) 20 E) 19

Page 3: Eureka 2003

Sociedade Brasileira de Matemática

EUREKA! N°16, 2003

3

5. Qual é a quantidade total de letras de todas as respostas incorretas desta questão?A) Quarenta e oito. B) Quarenta e nove. C) Cinqüenta.D) Cinqüenta e um. E) Cinqüenta e quatro.

6. Toda a produção mensal de latas de refrigerante de uma certa fábrica foi vendidaa três lojas. Para a loja A, foi vendida metade da produção; para a loja B, foramvendidos

5

2 da produção e para a loja C, foram vendidas 2500 unidades. Qual

foi a produção mensal dessa fábrica?A) 4166 latas B) 10000 latas C) 20000 latas D) 25000 latasE) 30000 latas

7. Um quadrado de área 1 foi dividido em 4 retângulos congruentes, conformeindicado no desenho à esquerda. Em seguida, os quatro retângulos foramreagrupados de maneira a formar um quadrado, com um buraco quadrado nocentro, conforme indica o desenho à direita.

A área do buraco é igual a:

A) 21 B)

16

9 C) 25

16 D) 43 E) 1

8. A linha poligonal AB é desenhada mantendo-se sempre o mesmo padrãomostrado na figura. Seu comprimento total é igual a:

1 2 3 30 31

1

2

A B4 5 6 7 8 9

A) 31 B) 88 C) 90 D) 97 E) 105

9. A diferença entre os quadrados de dois números inteiros positivos consecutivos ésempre:A) um número primo.B) um múltiplo de 3.C) igual à soma desses números.D) um número par.E) um quadrado perfeito.

Page 4: Eureka 2003

Sociedade Brasileira de Matemática

EUREKA! N°16, 2003

4

10. Marcelo leva exatamente 20 minutos para ir de sua casa até a escola. Uma certavez, durante o caminho, percebeu que esquecera em casa a revista Eureka! que iamostrar para a classe; ele sabia que se continuasse a andar, chegaria à escola 8minutos antes do sinal, mas se voltasse para pegar a revista, no mesmo passo,chegaria atrasado 10 minutos. Que fração do caminho já tinha percorrido nesteponto?A)

52 B)

20

9 C) 21 D)

3

2 E) 10

9

11. O gráfico abaixo mostra o faturamento mensal das empresas A e B no segundosemestre de 2001.

jul

a go

set

out

nov

d ez

milh

ões

de re

ais

1 00

1 20

1 40

1 60

1 80

2 00

Com base nesse gráfico, podemos afirmar que:A) houve um mês em que o faturamento da empresa A foi o dobro do

faturamento da empresa B.B) no mês de julho, a diferença de faturamentos foi maior que nos demais

meses.C) a empresa B foi a que sofreu a maior queda de faturamento entre dois meses

consecutivos.D) no semestre, o faturamento total de A foi maior que o de B.E) a diferença entre os faturamentos totais do semestre excedeu os 20 milhões

de reais.

12. Patrícia mora em São Paulo e quer visitar o Rio de Janeiro num feriadoprolongado. A viagem de ida e volta, de ônibus, custa 80 reais, mas Patrícia estáquerendo ir com seu carro, que faz, em média, 12 quilômetros com um litro degasolina. O litro da gasolina custa, em média, R$1,60 e Patrícia calcula que teráde rodar cerca de 900 quilômetros com seu carro e pagar 48 reais de pedágio. Elairá de carro e para reduzir suas despesas, chama duas amigas, que irão repartircom ela todos os gastos. Dessa forma, não levando em conta o desgaste do carroe outras despesas inesperadas, Patrícia irá:A) economizar R$20,00.

B) gastar apenas R$2,00 a mais.

Page 5: Eureka 2003

Sociedade Brasileira de Matemática

EUREKA! N°16, 2003

5

C) economizar R$24,00. D) gastar o mesmo que se fosse de ônibus. E) gastar R$14,00 a mais.

13. Uma escola vai organizar um passeio ao zoológico. Há duas opções detransporte. A primeira opção é alugar "vans": cada van pode levar até 6 criançase seu aluguel custa R$60,00. A segunda opção é contratar uma empresa parafazer o serviço: a empresa usa ônibus com capacidade para 48 crianças e cobraR$237,00, mais R$120,00 por ônibus utilizado. A escola deve preferir a empresade ônibus se forem ao passeio pelo menos N crianças. O valor de N é:A) 28 B) 31 C) 32 D) 33 E) 36

14. O produto de um milhão de números naturais, não necessariamente distintos, éigual a um milhão. Qual é o maior valor possível para a soma desses números?

A) 1 000 000 B) 1 250 002 C) 1 501 999 D) 1 999 999E) 13 999 432

15. Se você tiver uma mesa de bilhar retangular cuja razão entre a largura e ocomprimento seja 5/7 e bater em uma bola que está em um canto, de modo queela saia na direção da bissetriz do ângulo desse canto, quantas vezes ela baterános lados antes de bater em um dos cantos?A) 10 vezes B) 12 vezes C) 13 vezes D) 14 vezes E) 15 vezes

16. Na malha quadriculada a seguir, todas as circunferências têm centro em M.Então pode-se concluir que a área preta é:

M

A) dois quintos da área do círculo maior.B) três sétimos da área do círculo maior.C) metade da área do círculo maior.D) quatro sétimos da área do círculo maior.E) três quintos da área do círculo maior.

Page 6: Eureka 2003

Sociedade Brasileira de Matemática

EUREKA! N°16, 2003

6

17. As figuras a seguir são construídas com palitos pretos e brancos. Para construiras figuras, os palitos pretos foram colocados apenas nas bordas e os brancosapenas no interior. A figura de número n corresponde a um retângulo 3 por n.Continuando esse procedimento, quantos palitos brancos teremos na figura2002?

1 2 3

A) 2001 B) 4004 C) 12006 D) 10007 E) 10010

18. Um produtor de leite engarrafa diariamente toda a produção de leite de suafazenda. Depois de tirado, o leite segue para um tanque de forma cilíndrica eentão é engarrafado, conforme vemos na figura a seguir. Na tabela vemos aquantidade de garrafas que foram enchidas e o nível do leite dentro do tanque.Depois de quantas garrafas serem enchidas o tanque ficará vazio?

Quantidade de garrafasenchidas 0 200 400 600

Nível do tanque (cm) 210 170 130 90

A) 1000 B) 1050 C) 1100 D) 1150 E) 1200

19. Escrevendo todos os números inteiros de 100 a 999, quantas vezes escrevemos oalgarismo 5?

A) 250 B) 270 C) 271 D) 280 E) 292

20. Uma usina comprou 2000 litros de leite puro e então retirou certo volume Vdesse leite para produção de iogurte e substituiu esse volume por água. Emseguida, retirou novamente o mesmo volume V da mistura e novamentesubstituiu por água. Na mistura final existem 1125 litros de leite. O volume V é:A) 500 litros B) 600 litros C) 700 litros D) 800 litros E) 900 litros

Page 7: Eureka 2003

Sociedade Brasileira de Matemática

EUREKA! N°16, 2003

7

PROBLEMAS – NÍVEL 2

1. Um comerciante comprou dois carros por um total de R$ 27.000,00. Vendeu oprimeiro com lucro de 10% e o segundo com prejuízo de 5%. No total ganhouR$ 750,00. Os preços de compra foram, respectivamente,A) R$ 10.000,00 e R$ 17.000,00B) R$ 13.000,00 e R$ 14.000,00C) R$ 14.000,00 e R$ 13.000,00D) R$ 15.000,00 e R$ 12.000,00E) R$ 18.000,00 e R$ 9.000,00

2. Veja o problema No. 15 do Nível 1.

3. Dizer que uma tela de televisão tem 20 polegadas significa que a diagonal da telamede 20 polegadas. Quantas telas de televisão de 20 polegadas cabem numa de60 polegadas?A) 9 B) 10 C) 18 D) 20 E) 30

4. Veja o problema No. 20 do Nível 1.

5. Dois irmãos, Pedro e João, decidiram brincar de pega-pega. Como Pedro é maisvelho, enquanto João dá 6 passos, Pedro dá apenas 5. No entanto, 2 passos dePedro equivalem à distância que João percorre com 3 passos. Para começar abrincadeira, João dá 60 passos antes de Pedro começar a persegui-lo. Depois dequantos passos Pedro alcança João?

A) 90 passos B) 120 passos C) 150 passos D) 180 passosE) 200 passos

6. Veja o problema No. 9 do Nível 1.7. Veja o problema No. 10 do Nível 1.8. Veja o problema No. 4 do Nível 1.9. Veja o problema No. 12 do Nível 1.

10. Traçando segmentos, podemos dividir um quadrado em dois quadradinhoscongruentes, quatro trapézios congruentes e dois triângulos congruentes,conforme indica o desenho abaixo, à esquerda. Eliminando algumas dessaspartes, podemos montar o octógono representado à direita. Que fração da área doquadrado foi eliminada?

Page 8: Eureka 2003

Sociedade Brasileira de Matemática

EUREKA! N°16, 2003

8

A) 9

1 B) 9

2 C) 4

1 D) 3

1 E) 8

3

11. Veja o problema No. 11 do Nível 1.12. Veja o problema No. 14 do Nível 1.

13. O lava-rápido "Lave Bem" faz uma promoção:Lavagem simples R$5,00

Lavagem completa R$7,00

No dia da promoção, o faturamento do lava-rápido foi de R$176,00. Nesse dia,qual o menor número possível de clientes que foram atendidos?A) 23 B) 24 C) 26 D) 28 E) 30

14. Veja o problema No. 7 do Nível 1.

15. Quantos números inteiros positivos menores que 900 são múltiplos de 7 eterminam em 7?A) 10 B) 11 C) 12 D) 13 E) 14

16. Dado um triângulo ABC onde °=80A e °= 40C , a medida do ângulo agudo

formado pelas bissetrizes dos ângulos A e B é:A) 40° B) 60° C) 70o D) 80° E) 110o

17. Na malha quadrada abaixo, há 6 quadrados de lado 30 cm. A área do triânguloABC é:

A B

C

A) 150 cm2 B) 100 cm2 C) 75 cm2 D) 50 cm2 E) 25 cm2

18. Veja o problema No. 8 do Nível 1.19. Veja o problema No. 19 do Nível 1.

Page 9: Eureka 2003

Sociedade Brasileira de Matemática

EUREKA! N°16, 2003

9

20. Se xy = 2 e x2 + y2 = 5, então 22

2

2

2

++x

y

y

x vale:

A) 2

5 B) 4

25 C) 4

5 D) 2

1 E) 1

21. Veja o problema No. 13 do Nível 1.

22. Durante sua viagem ao país das Maravilhas a altura de Alice sofreu quatromudanças sucessivas da seguinte forma: primeiro ela tomou um gole de umlíquido que estava numa garrafa em cujo rótulo se lia: "beba-me e fique 25%mais alta". A seguir, comeu um pedaço de uma torta onde estava escrito: "prove-me e fique 10% mais baixa"; logo após tomou um gole do líquido de outragarrafa cujo rótulo estampava a mensagem: "beba-me e fique 10% mais alta".Finalmente, comeu um pedaço de outra torta na qual estava escrito:"prove-me efique 20% mais baixa". Após a viagem de Alice, podemos afirmar que ela:A) ficou 1% mais baixaB) ficou 1% mais altaC) ficou 5% mais baixaD) ficou 5% mais altaE) ficou 10% mais alta

23. Vamos provar que 4 é maior que 4.Sejam a e b dois números tais que a > 4 e a = b.

1) Vamos subtrair 4 dos dois termos desta equação:a = b

a – 4 = b – 42) Colocamos –1 em evidência no segundo membro da equação:

a – 4 = –1 (– b + 4)a – 4 = –1 (4 – b)

3) Elevamos ambos os termos da equação ao quadrado:22 )]4(1[)4( ba −⋅−=−222 )4()1()4( ba −−=−

22 )4(1)4( ba −⋅=− 22 )4()4( ba −=−4) Extraímos a raiz quadrada dos dois membros da equação:

22 )4()4( ba −=−a – 4 = 4 – b

5) Como a = b, substituímos b por aa – 4 = 4 – a

Page 10: Eureka 2003

Sociedade Brasileira de Matemática

EUREKA! N°16, 2003

10

6) Resolvemos a equação:a – 4 = 4 – a

2a = 8a = 4

Como escolhemos a tal que a > 4, chegamos à inacreditável conclusão de que 4 > 4. Onde está o erro no argumento acima? A) Na passagem 2. B) Na passagem 3. C) Na passagem 4.

D) Na passagem 5. E) Na passagem 6.

24. Veja o problema No. 5 do Nível 1.

25. O resto da divisão por 9 de 222221111111111− é:A) 0 B) 1 C) 3 D) 6 E) 8

PROBLEMAS – NÍVEL 3

1. Veja o problema No. 11 do Nível 1.

2. Se q

p é a fração irredutível equivalente a

...444,2

...888,6 o valor de p + q é igual a:

A) 38 B) 39 C) 40 D) 41 E) 42

3. Veja o problema No. 1 do Nível 2.

4. A seguir vemos quatro vasos, os quais Angela vai encher com água, numatorneira cuja vazão é constante.

1 2 3 4

Os gráficos A e B a seguir representam o nível da água (eixo vertical), em doisdos vasos, de acordo com o tempo (eixo horizontal).

A B

Qual dos vasos corresponde ao gráfico A e qual ao gráfico B, respectivamente?A) 3 e 4 B) 2 e 4 C) 1 e 3 D) 2 e 3 E) 1 e 4

Page 11: Eureka 2003

Sociedade Brasileira de Matemática

EUREKA! N°16, 2003

11

5. Veja o problema No. 13 do Nível 1.6. Veja o problema No. 22 do Nível 2.7. Veja o problema No. 10 do Nível 1.8. Veja o problema No. 8 do Nível 1.9. Veja o problema No. 10 do Nível 2.10. Veja o problema No. 20 do Nível 2.

11. A média aritmética das idades de um grupo de médicos e advogados é 40 anos.A média aritmética das idades dos médicos é 35 anos e a dos advogados é 50anos. Pode-se, então, afirmar que:A) O número de advogados é o dobro do número de médicos no grupo.B) O número de médicos é o dobro do número de advogados no grupo.C) Há um médico a mais no grupo.D) Há um advogado a mais no grupo.E) Existem as mesmas quantidades de médicos e advogados no grupo.

12. Os valores de x, y e z que satisfazem às equações 51 =+y

x , 11 =+z

y e

21 =+x

z são tais que zyx 23 ++ é igual a:

A) 5 B) 6 C) 7 D) 8 E) 9

13. Veja o problema No. 23 do Nível 2.14. Veja o problema No. 5 do Nível 1.

15. Sejam x, y, z números inteiros tais que x + y + z = 0. Sobre 333 zyx ++ sãofeitas as seguintes afirmativas:i) É necessariamente múltiplo de 2.ii) É necessariamente múltiplo de 3.iii) É necessariamente múltiplo de 5.Podemos afirmar que:

A) somente i) é correta.B) somente ii) é correta.C) somente i) e ii) são corretas.D) somente i) e iii) são corretas.E) i), ii) e iii) são corretas.

Page 12: Eureka 2003

Sociedade Brasileira de Matemática

EUREKA! N°16, 2003

12

16. Seja f uma função real de variável real que satisfaz a condição:

xx

fxf 32002

2)( =

+

para x > 0. O valor de f(2) é igual a:A) 1000 B) 2000 C) 3000 D) 4000E) 6000

17. Veja o problema No. 25 do Nível 2.

18. Na circunferência abaixo, temos que: AB = 4, BC = 2, AC é diâmetro e os

ângulos DBA ˆ e DBC ˆ são iguais. Qual é o valor de BD? A

B

D

C

A) 132 + B) 5

9C) 23 D) 52 + E) 4

19. Seja α a maior raiz de x2 + x – 1 = 0. O valor de α5 – 5α é :A) – 1 B) – 2 C) – 3 D) 1 E) 2

20. Qual é o dígito das unidades de 7...7777 , onde aparecem 2002 setes?

A) 7 B) 9 C) 3 D) 1 E) 5.

21. Em um trapézio ABCD de área 1, a base BC mede a metade da base AD. Seja Ko ponto médio da diagonal AC. A reta DK corta o lado AB no ponto L. A área doquadrilátero BCKL é igual a:

A) 4

3 B)

3

2C)

3

1D)

9

2E)

9

1

22. N = ❑ 539984❑ é um número inteiro positivo com oito algarismos, sendo o

primeiro e o último desconhecidos. Sabendo que N é um múltiplo de 198,encontre o algarismo das unidades de N / 198.A) 5 B) 6 C) 7 D) 8 E) 9

Page 13: Eureka 2003

Sociedade Brasileira de Matemática

EUREKA! N°16, 2003

13

23. No triminó marciano, as peças têm 3 números cada (diferente do dominó daterra, onde cada peça tem apenas 2 números). Os números no triminó marcianotambém variam de 0 a 6, e para cada escolha de 3 números (não necessariamentedistintos) existe uma e somente uma peça que contém esses 3 números. Qual é asoma dos números de todas as peças do triminó marciano?A) 756 B) 1512 C) 84 D) 315 E) 900

24. No triângulo ABC, o ângulo A mede 60° e o ângulo B mede 50°. Sejam M oponto médio do lado AB e P o ponto sobre o lado BC tal que AC + CP = BP.Qual a medida do ângulo MPC?A) 120° B) 125° C) 130° D) 135° E) 145°

25. Duas pessoas vão disputar uma partida de par ou ímpar. Elas não gostam dozero e, assim, cada uma coloca 1, 2, 3, 4 ou 5 dedos com igual probabilidade.A probabilidade de que a pessoa que escolheu par ganhe é:A) 1/2 B) 2/5 C) 3/5 D) 12/25 E) 13/25

GABARITONÍVEL 1 (5a. e 6a. Séries)

1) C 6) D 11) D 16) C2) C 7) B 12) C 17) D3) B 8) D 13) B 18) B4) B 9) C 14) D 19) D5) D 10) B 15) A 20) A

NÍVEL 2 (7a. e 8a. Séries)1) C 6) C 11) D 16) C 21) B2) A 7) B 12) D 17) C 22) A3) A 8) B 13) C 18) D 23) C4) A 9) C 14) B 19) D 24) D5) E 10) B 15) D 20) B 25) D

NÍVEL 3 (Ensino Médio)1) D 6) A 11) B 16) B 21) D2) E 7) B 12) B 17) D 22) C3) C 8) D 13) C 18) C 23) A4) C 9) B 14) D 19) C 24) E5) B 10) B 15) C 20) C 25) E

Page 14: Eureka 2003

Sociedade Brasileira de Matemática

EUREKA! N°16, 2003

14

XXIV OLIMPÍADA BRASILEIRA DE MATEMÁTICAProblemas e Soluções da Segunda Fase

PROBLEMAS – NÍVEL 1

PROBLEMA 1O ano 2002 é palíndromo, ou seja, continua o mesmo se lido da direita para aesquerda.

a) Depois de 2002, quais serão os próximos quatro anos palíndromos?b) O último ano palíndromo, 1991, era ímpar. Quando será o próximo ano

palíndromo ímpar?

PROBLEMA 2Um fazendeiro resolveu repartir sua fazendapara seus cinco filhos. O desenho ao lado (forade escala) representa a fazenda e as partes dosherdeiros, que são da forma triangular, de

modo que4

BCBD = , ,

3

ACAE =

2

DCDF = e

EG = GC. O filho mais novo recebeu o terrenorepresentado pelo triângulo escuro, de 40alqueires. Quantos alqueires tinha apropriedade original?

A

BD

E

G

FC

PROBLEMA 3Dado um número, pode-se escrever o seu dobro ou suprimir o seu algarismo dasunidades. Apresente uma seqüência que começa com 2002 e termina com 13,usando somente essas duas operações.

PROBLEMA 4Três amigas foram para uma festa com vestidos azul,preto e branco, respectivamente. Seus pares de sapatoapresentavam essas mesmas três cores, mas somente Anausava vestido e sapatos de mesma cor. Nem o vestidonem os sapatos de Júlia eram brancos. Marisa usavasapatos azuis. Descreva a cor do vestido de cada uma dasmoças.

Page 15: Eureka 2003

Sociedade Brasileira de Matemática

EUREKA! N°16, 2003

15

PROBLEMA 5No jogo pega-varetas, as varetas verdes valem 5 pontos cada uma, as azuis valem 10pontos, as amarelas valem 15, as vermelhas, 20 e a preta, 50. Existem 5 varetasverdes, 5 azuis, 10 amarelas, 10 vermelhas e 1 preta. Carlinhos conseguiu fazer 40pontos numa jogada. Levando em conta apenas a quantidade de varetas e suas cores,de quantas maneiras diferentes ele poderia ter conseguido essa pontuação, supondoque em cada caso fosse possível pegar as varetas necessárias?

PROBLEMA 6Nas casas de um tabuleiro 8 × 8 foram escritos números inteiros positivos de formaque a diferença entre números escritos em casas vizinhas (quadrados com um ladocomum) é 1. Sabe-se que numa das casas está escrito 17 e, em outra, está escrito 3.Desenhe um tabuleiro 8 × 8, preencha-o segundo essas regras e calcule a soma dosnúmeros escritos nas duas diagonais do tabuleiro.

PROBLEMAS – NÍVEL 2

PROBLEMA 1Geraldinho e Magrão saíram de suas casas nomesmo instante com a intenção de um visitar ooutro, caminhando pelo mesmo percurso.Geraldinho ia pensando num problema deolimpíada e Magrão ia refletindo sobrequestões filosóficas e nem perceberam quandose cruzaram. Dez minutos depois, Geraldinhochegava à casa de Magrão e meia hora maistarde, Magrão chegava à casa de Geraldinho.Quanto tempo cada um deles andou?

Observação: Cada um deles anda comvelocidade constante.

PROBLEMA 2

����

������

�������

�����

Um grande painel na forma de um quarto de círculo foicomposto com 4 cores, conforme indicado na figura aolado, onde o segmento divide o setor em duas partesiguais e o arco interno é uma semicircunferência. Qual éa cor que cobre a maior área?

Page 16: Eureka 2003

Sociedade Brasileira de Matemática

EUREKA! N°16, 2003

16

PROBLEMA 3Nas casas de um tabuleiro 8 × 8 foram escritos números inteiros positivos de formaque a diferença entre números escritos em casas vizinhas (quadrados com um ladocomum) é 1. Sabe-se que numa das casas está escrito 17 e, em outra, está escrito 3.Calcule a soma dos números escritos nas duas diagonais do tabuleiro.

PROBLEMA 4

B C

A D

O professor Pardal está estudando o comportamentofamiliar de uma espécie de pássaro. Os pontos A, B, C eD da figura ao lado, representam a disposição de quatroninhos desses pássaros. O professor construiu um postode observação equidistante dos quatro ninhos.Todos os ninhos e o posto de observação estão em ummesmo nível de altura a partir do solo, a distância de B

a D é de 16 metros e °= 45ˆDAB . Determine a distânciaque o posto guarda de cada ninho.

PROBLEMA 5O primeiro número de uma seqüência é 7. O próximo é obtido da seguinte maneira:Calculamos o quadrado do número anterior 72 = 49 e a seguir efetuamos a soma deseus algarismos e adicionamos 1, isto é, o segundo número é 4 + 9 + 1 = 14.Repetimos este processo, obtendo 142 = 196 e o terceiro número da seqüência é 1 + 9+ 6 + 1 = 17 e assim sucessivamente. Qual o 2002o elemento desta seqüência?

PROBLEMA 6O ano 2002 é palíndromo, ou seja, continua o mesmo se lido da direita para aesquerda.a) Depois de 2002, quais serão os próximos quatro anos palíndromos?b) O último ano palíndromo, 1991, era ímpar. Quando será o próximo ano

palíndromo ímpar?c) O último ano palíndromo primo aconteceu há mais de 1000 anos, em 929.

Determine qual será o próximo ano palíndromo primo.

PROBLEMAS – NÍVEL 3

PROBLEMA 1Veja o problema No. 5 do Nível 2.

PROBLEMA 2Para quais inteiros positivos n existe um polígono não regular de n lados, inscrito emuma circunferência, e com todos os ângulos internos de mesma medida?

Page 17: Eureka 2003

Sociedade Brasileira de Matemática

EUREKA! N°16, 2003

17

PROBLEMA 3Determine o maior natural k para o qual existe um inteiro n tal que 3k dividen3 – 3n2 + 22.

PROBLEMA 4Quantos dados devem ser lançados ao mesmo tempo para maximizar a probabilidadede se obter exatamente um 2?

PROBLEMA 5Em um quadrilátero convexo ABCD, os lados opostos AD e BC são congruentes e ospontos médios das diagonais AC e BD são distintos.Prove que a reta determinada pelos pontos médios das diagonais forma ângulosiguais com AD e BC.

PROBLEMA 6Colocamos vários palitos sobre uma mesa de modo a formar um retângulo m × n,como mostra a figura.Devemos pintar cada palito de azul, vermelho ou preto de modo que cada um dosquadradinhos da figura seja delimitado por exatamente dois palitos de uma cor e doisde outra cor. De quantas formas podemos realizar esta pintura?

m

n

. . .

. . .

. . .

� � � �

Page 18: Eureka 2003

Sociedade Brasileira de Matemática

EUREKA! N°16, 2003

18

SOLUÇÕES – NÍVEL 1

SOLUÇÃO DO PROBLEMA 1a) Os palíndromos entre 2000 e 3000 são da forma 2aa2, onde a é um algarismo.Logo os próximos quatro serão 2112, 2222, 2332 e 2442.

b) Como o primeiro algarismo é igual ao último, um palíndromo ímpar maior que2002 deve começar e terminar por um número ímpar maior ou igual a 3. Logo opróximo será 3003.

SOLUÇÃO DO PROBLEMA 2Seja S a área do triângulo ABC.

Se ,4

BCBD = então .

4)(

SABD =

Se ,3

ACAE = então .

434

3

34

3

)()(

SSS

SADC

AED ==−

==

Se ,2

DCDF = então .

42

44

2

)()(

S

SSS

DECDEF =

+−

==

Se EG = EC, então .82

4

3

2

)()(

SS

SEFC

GFC =

==

Como (GFC) = 40 temos 320408

=⇔= SS alqueires.

SOLUÇÃO DO PROBLEMA 3Uma possível solução é:2002, 200, 20, 2, 4, 8, 16, 32, 64, 128, 256, 512, 51, 102, 204, 408, 816, 1632, 163,326, 652, 1304, 130, 13.

SOLUÇÃO DO PROBLEMA 4Como os sapatos de Marisa eram azuis, e nem o vestido nem os sapatos de Júliaeram brancos, conclui-se que os sapatos de Júlia eram pretos e portanto os sapatos deAna eram brancos.O vestido de Ana era branco, pois era a única que usava vestido e sapatos da mesmacor; conseqüentemente, o vestido de Júlia era azul e o de Marisa era preto.

SOLUÇÃO DO PROBLEMA 5A soma dos pontos é 40. Segundo as regras do jogo, as possibilidades são:

Page 19: Eureka 2003

Sociedade Brasileira de Matemática

EUREKA! N°16, 2003

19

20

20

15 – 5

1010

5 – 5

5 – 5 – 5 – 5

20 + 20

20 + 15 + 5

20 + 10 + 10

20 + 10 + 5 + 5

20 + 5 + 5 + 5 + 5

(1)

(2)

(3)

(4)

(5)

15

155 – 5

10 10

5

5 – 5 – 5 – 5 – 5

15 + 15 + 1015 + 15 + 5 + 5

15 + 10 + 10 + 5

15 + 10 + 5 + 5 + 5

15 + 5 + 5 + 5 + 5 + 5

(6)(7)

(8)

(10)

10

5

5 – 5 (9)

105 – 5 – 5 – 5

10

5 – 5

10 + 10 + 5 + 5 + 5 + 5

10 + 10 + 10 + 10

(12)(13)

(11)

10

1010 + 10 + 10 + 5 + 5

– 5 – 5 – 5 – 5 – 5 – 5 não dá, pois há apenas 5 varetas verdes.

A resposta é portanto: de 13 maneiras diferentes.

SOLUÇÃO DO PROBLEMA 6Como a diferença entre o 17 e o 3 é 14, esses números devem estar em posiçõesafastadas de 14 casas, contadas na horizontal ou vertical.Portanto 17 e 3 devem ocupar as extremidades de uma das diagonais do tabuleiro.A partir disso, o preenchimento das diagonais é feito de maneira única. E umamaneira de se preencher o tabuleiro é a seguinte:

17 16 15 14 13 12 11 1016 15 14 13 12 11 10 915 14 13 12 11 10 9 814 13 12 11 10 9 8 713 12 11 10 9 8 7 612 11 10 9 8 7 6 511 10 9 8 7 6 5 410 9 8 7 6 5 4 3

a soma dos números escritos nas diagonais é: 8 × 10 + (3 + 5 +...+ 17) = 160.

Page 20: Eureka 2003

Sociedade Brasileira de Matemática

EUREKA! N°16, 2003

20

SOLUÇÕES – NÍVEL 2

SOLUÇÃO DO PROBLEMA 1Seja t > 0 o tempo, em minutos, decorrido desde a saída de Geraldinho e Magrão atéo instante do encontro.Sejam g e m as distâncias entre o ponto de encontro e as casas de Geraldinho eMagrão, respectivamente. Como Geraldino percorre a distância g em t minutos e a

distância m em 10 minutos, temos .10

t

m

g=

Analogamente, .40

tm

g= Logo 20400

40

102 =⇔=⇔= tt

t

t (pois t > 0). Logo

Geraldinho andou 10 + 20 = 30 minutos e Magrão andou 40 + 20 = 60 minutos.

SOLUÇÃO DO PROBLEMA 2

w

x y

z

Sejam x, y, z e w as áreas das regiões branca, amarela,azul e verde, respectivamente.

Seja R o raio do semicírculo. Temos 2

2Ryx

π=+

e 2

)2(8

1 22 R

Rwxzyππ ==+=+

Assim, x + y = y + z = x + w, logo x = z e y = w.Como se x é a área de um segmento circular de ângulo

90° e raio R, 222

4

2

24R

RRx

−=−= ππ e

.4

2 2Ry

+= π Assim x = z < y = w.

SOLUÇÃO DO PROBLEMA 3Como a diferença entre o 17 e o 3 é 14, esses números devem estar em posiçõesafastadas de 14 casas, contadas na horizontal ou vertical.Portanto 17 e 3 devem ocupar as extremidades de uma das diagonais do tabuleiro.A partir disso, o preenchimento das diagonais é feito de maneira única. E umamaneira de se preencher o tabuleiro é a seguinte:

17 16 15 14 13 12 11 1016 15 14 13 12 11 10 915 14 13 12 11 10 9 814 13 12 11 10 9 8 713 12 11 10 9 8 7 612 11 10 9 8 7 6 511 10 9 8 7 6 5 410 9 8 7 6 5 4 3

Page 21: Eureka 2003

Sociedade Brasileira de Matemática

EUREKA! N°16, 2003

21

a soma dos números escritos nas diagonais é: 8 × 10 + (3 + 5 +...+ 17) = 160.

SOLUÇÃO DO PROBLEMA 4Observe que o posto do observador coincide com o centro do círculo circunscrito ao

quadrilátero ABCD. Como 16=BD , sendo O o centro do círculo circunscrito, temosˆ ˆ2 90BOD BAD= ⋅ = ° e rODBO == , donde ,16 222 rr += pelo teorema de

Pitágoras, e logo .28128 ==r Assim, a distância do posto (que deve ficar em O)

aos ninhos será de 28 metros.

SOLUÇÃO DO PROBLEMA 5Os primeiros números da seqüência são (7, 14, 17, 20, 5, 8, 11, 5...) donde vemosque, exceto pelos 4 primeiros termos, a seqüência é periódica com período 3. Como2002 deixa resto 1 quando dividido por 3, o número procurado coincide com aqueleque ocupa o 7o. lugar na seqüência, a saber, 11.

Observação:Para qualquer termo inicial, a seqüência construída de acordo com método descritono enunciado do problema será eventualmente periódica, (isto é teremos an + k = ak

para todo k ≥ m, para certos valores positivos de m e n).

SOLUÇÃO DO PROBLEMA 6a) Os palíndromos entre 2000 e 3000 são da forma 2aa2, onde a é um algarismo.Logo os próximos quatro serão 2112, 2222, 2332 e 2442.

b) Como o primeiro algarismo é igual ao último, um palíndromo ímpar maior que2002 deve começar e terminar por um número ímpar maior ou igual a 3. Logo opróximo será 3003.

c) Um palíndromo de quatro algarismos é da forma abba = a + 10b + 100b + 1000a= 1001a + 110b, que é múltiplo de 11, já que 110 e 1001 são múltiplos de 11. Logoo próximo ano palíndromo primo tem no mínimo 5 algarismos.Os menores palíndromos de 5 algarismos são 10001, que é múltiplo de 73 e 10101,que é múltiplo de 3. O próximo é 10201 = 1012, divisível por 101. O seguinte,

10301, é primo, pois não é divisível por qualquer primo menor que .10210301 <

Page 22: Eureka 2003

Sociedade Brasileira de Matemática

EUREKA! N°16, 2003

22

SOLUÇÕES – NÍVEL 3

SOLUÇÃO DO PROBLEMA 1Veja a solução do problema No. 5 do Nível 2.

SOLUÇÃO DO PROBLEMA 2Seja C a circunferência de centro O circunscrita ao polígono A1A2...An. Os triângulos

AiAi + 1 O (com An + 1 = A1) são isósceles. Seja .ˆ1+= iii AAOα

Então(1) .... 1433221 αααααααα +==+=+=+ n

Portanto.

=======

2

642

531

...

...,

αααααααα

n

O

αn

αn

α1

α1 α2

α2

α3

α3

A2

A1 A3

Se n for ímpar, então ,...21 nααα === logo todos os ângulos 1ˆ

+ii AOA serão iguaise o polígono será regular.Para n par, não é necessário que todos os ângulos sejam iguais.

Escolhendo x ≠ y de modo que x + y = ângulo interno = n

n )2(180 − e fazendo

131 ... −==== nx ααα ,

ny ααα ==== ...42 , obtemos um polígono inscritível não regular com todos osângulos de mesma medida.

Portanto, para n par ≥ 4, existe um polígono de n lados satisfazendo as condições doproblema.

SOLUÇÃO DO PROBLEMA 3

Se n = 3r, então 22)3(3)3(223 2323 +⋅−=+− rrnn é a soma de um múltiplo de 3com 22, logo não é múltiplo de 3.Se n = 3r + 1, então

Page 23: Eureka 2003

Sociedade Brasileira de Matemática

EUREKA! N°16, 2003

23

−⋅−+⋅+⋅+=++−+=+− 2232323 )3(31)3(3)3(3)3(22)13(3)13(223 rrrrrrnn

20)3(3)3(223)3(23 3 +⋅−=+−⋅⋅ rrr , que também não é múltiplo de 3.

Finalmente, se n = 3r – 1, então =+−−−=+− 22)13(3)13(223 2323 rrnn

,1839)3(6)3(223)3(23)3(31)3(3)3(3)3( 23223 +⋅+⋅−=+−⋅⋅+⋅−−⋅+⋅−= rrrrrrrrque é a soma de um múltiplo de 27 com 18, e portanto é múltiplo de 9 mas não de

27, logo a maior potência de 3 que divide um número da forma 223 23 +− nn é 32 =9. Assim, k é no máximo 2.

SOLUÇÃO DO PROBLEMA 4Suponha que os dados estão numerados de 1 a n. A probabilidade de que somente odado No. 1 resulte em 2 é:

.6

5

6

5...

6

5

6

5

6

1 1

n

n−

=××

Analogamente, a probabilidade de que somente o dado k, (1 ≤ k ≤ n) resulte em 2 é

.6

5

6

5...

6

5

6

1

6

5...

6

5

6

5 1

n

n−

=×××××

Portanto, a probabilidade de obter exatamente um 2 é

.6

5

6

5...

6

5

6

5 1111

n

n

n

n

n

n

n

n

n nP−−−−

⋅=+++=

Agora observe que .5)1(566

5)1(

6

51

1

1 ≥⇔+≥⇔⋅+≥⋅⇔≥+

+ nnnnnPPn

n

n

n

nn

Para n = 5, ocorre a igualdade (P5 = P6), P5 = P6 > P7 > P8 > P9 >... eP1 < P2 < P3 < P4 < P5 = P6

E a probabilidade é máxima para n = 5 ou n = 6.

SOLUÇÃO DO PROBLEMA 5Sejam M e N os pontos médios de AC e BD e P o ponto médio do lado AB. EntãoPM é base média do ABC∆ e PN base média do .ABD∆ Segue que

.22

PNADBC

PM ===

Sendo X e Y as interseções da reta MN com BC e AD, temos então

NYAMNPNMPMXB ˆˆˆˆ === ou .ˆˆˆˆ NYAMNPNMPMXB =−=−= ππ

Page 24: Eureka 2003

Sociedade Brasileira de Matemática

EUREKA! N°16, 2003

24

A P

B

Y N M

X

D C

SOLUÇÃO ALTERNATIVA:

Provaremos que se, 2

CAM

+= e 2

DBN

+= então o vetor MN faz ângulos iguais

com AD e BC . Para isso, como ,BCAD = basta ver que os produtos internos

ADMN ⋅ e BCMN ⋅ têm o mesmo módulo.Temos

( ) =−−−⋅−

=−⋅

−−+=−⋅−=⋅

2

)()(

2)()(

2ADADBC

ADDBCA

ADMNADMN

BCMNBCNMBCCABDBCADBC

⋅−=−⋅−=−⋅−−+

=−−−⋅−

= )()(2

)()(

2

)()(2

SOLUÇÃO DO PROBLEMA 6Há 3n maneiras de colorir a fileira horizontal superior de palitos. O palito verticalmais à esquerda da primeira linha também pode ser colorido de 3 maneiras.

n

. . . . . .� � �. . .

. . .

. . . � �. . .

Uma vez definidas as cores dos palitos superior e mais à esquerda de umquadradinho, há duas maneiras de completá-lo segundo as condições do enunciado:se ambos têm mesma cor, há duas escolhas para a cor dos dois palitos restantes; seambos têm cores diferentes, há duas maneiras de colorir os dois palitos restantes comestas cores.Assim, para completar a primeira linha de quadrados há 3n ⋅ 3 ⋅ 2n maneirasDa mesma forma, a cor do palito vertical mais à esquerda da segunda linha dequadrados pode ser escolhido de 3 maneiras, e há 2n maneiras de colorir os demaispalitos desta linha. Assim, para m = 2, há 3n ⋅ 3 ⋅ 2n⋅ 3 ⋅ 2n colorações possíveis.

Analogamente, no caso geral, há nmmnmnn 23)23(3 ⋅=⋅⋅ + maneiras de realizar apintura pedida.

Page 25: Eureka 2003

Sociedade Brasileira de Matemática

EUREKA! N°16, 2003

25

XXIV OLIMPÍADA BRASILEIRA DE MATEMÁTICAProblemas e Soluções da Terceira Fase

PROBLEMAS – NÍVEL 1

PROBLEMA 1No quadriculado ao lado estão escritos todos osinteiros de 1 a 25. Considere todos os conjuntosformados por cinco desses números, de modo que,para cada conjunto, não existem dois números queestão na mesma linha ou na mesma coluna.

a) Apresente um conjunto cujo maior elemento é o23.

b) Apresente um conjunto cujo maior elemento é omenor possível.

2

15

14

3

20

13

1

12

25

19

16

9

21

22

6

11

7

24

18

5

23

10

8

4

17

PROBLEMA 2No desenho ao lado, a reta t é perpendicular aosegmento AB e passa pelo seu ponto médio M.Dizemos que A é o simétrico de B em relação à reta t(ou em relação ao segmento PQ ).Seja XYZ um triângulo retângulo de área 1m2.Considere o triângulo X'Y'Z' tal que X' é o simétricode X em relação ao lado YZ , Y' é o simétrico de Yem relação ao lado XZ e Z' é o simétrico de Z emrelação ao lado XY .Calcule a área do triângulo X'Y'Z'.

A

P

Q

M

B t

PROBLEMA 3Um parque tem a forma de um quadrilátero e possuioito portões de entrada: um em cada vértice doquadrilátero e um no meio de cada lado. Os portõesforam numerados de 1 a 8, de forma que a soma T dosnúmeros em cada lado é a mesma para os quatro lados.Apresente um exemplo de numeração dos pontos paracada um dos possíveis valores de T.

Page 26: Eureka 2003

Sociedade Brasileira de Matemática

EUREKA! N°16, 2003

26

PROBLEMA 4Sete moedas estão dispostas em círculo, com a coroavisível.

a) Mostre que é possível, virando-se cinco moedasconsecutivas de cada vez, fazer com que todasfiquem com a cara visível.

b) Mostre que não é possível, virando-se quatro moedasconsecutivas de cada vez, fazer com que todasfiquem com a cara visível.

PROBLEMA 5São dados um tabuleiro de xadrez (8 × 8) e palitinhos do tamanho dos lados dascasas. Dois jogadores jogam alternadamente e, em cada jogada, um dos jogadorescoloca um palitinho sobre um lado de uma casa do tabuleiro, sendo proibidosobrepor palitinhos.Vence o jogador que conseguir completar primeiro um quadrado 1 × 1 de palitinhos.Supondo que nenhum jogador cometa erros, qual dos dois jogadores tem a estratégiavencedora, ou seja, consegue vencer independentemente de como jogue seuadversário?

PROBLEMAS – NÍVEL 2

PROBLEMA 1Veja o problema No. 2 do Nível 1.

PROBLEMA 2Mostre que, entre dezoito inteiros consecutivos de três algarismos, sempre existealgum que é divisível pela soma de seus algarismos.

PROBLEMA 3São dados um tabuleiro quadriculado m × n e palitinhos do tamanho dos lados dascasas. Dois jogadores jogam alternadamente e, em cada jogada, um dos jogadorescoloca um palitinho sobre um lado de uma casa do tabuleiro, sendo proibidosobrepor palitinhos.Vence o jogador que conseguir completar primeiro um quadrado 1 × 1 de palitinhos.Supondo que nenhum jogador cometa erros, qual dos dois jogadores tem a estratégiavencedora, ou seja, consegue vencer independentemente de como jogue seuadversário?

Page 27: Eureka 2003

Sociedade Brasileira de Matemática

EUREKA! N°16, 2003

27

PROBLEMA 4Uma mistura possui os componentes A e B na razão 3 : 5, uma segunda misturapossui os componentes B e C na razão 1 : 2 e uma terceira mistura possui oscomponentes A e C na razão 2 : 3. Em que razão devemos combinar a 1a, 2a e 3a

misturas para que os componentes A, B e C apareçam na razão 3 : 5 : 2?

PROBLEMA 5Seja ABC um triângulo inscrito em uma circunferência de centro O e P um pontosobre o arco AB que não contém C. A perpendicular traçada por P à reta BOintersecta AB em S e BC em T. A perpendicular traçada por P a AO intersecta AB emQ e AC em R.Prove as duas afirmações a seguir:

a) PQS é um triângulo isósceles

b) STQRPQ ⋅=2

PROBLEMA 6

Seja n um inteiro positivo. Definimos nppp

nk

−⋅⋅

−⋅

−= 1

1...1

11

1)(21

ϕ ,

onde kppp ,...,, 21 são os fatores primos distintos de n. Prove que para todo m ≥ 1,existe n tal que !)( mn =ϕ .

Obs: mm ⋅⋅⋅= ...21! .

PROBLEMAS – NÍVEL 3

PROBLEMA 1Mostre que existe um conjunto A formado por inteiros positivos tendo as seguintespropriedades:a) A tem 2002 elementos.b) A soma de qualquer quantidade de elementos distintos de A (pelo menos um)

nunca é uma potência perfeita.

Obs: Uma potência perfeita é um número da forma ab, onde a e b são inteirospositivos e b ≥ 2.

Page 28: Eureka 2003

Sociedade Brasileira de Matemática

EUREKA! N°16, 2003

28

PROBLEMA 2ABCD é um quadrilátero convexo e inscritível e M é um ponto sobre o lado CD, talque o triângulo ADM e o quadrilátero ABCM têm a mesma área e o mesmoperímetro. Prove que ABCD tem dois lados de comprimentos iguais.

PROBLEMA 3Numeramos as casas de um tabuleiro quadriculado m × n, onde m, n ≥ 2, com osinteiros 1, 2, 3,...,mn de modo que, para todo i ≤ mn – 1, as casas i e i + 1 tenhamum lado em comum.Prove que existe i ≤ mn – 3 tal que as casas i e i + 3 têm um lado em comum.

PROBLEMA 4Definimos o diâmetro de um subconjunto não vazio de {1, 2,..., n} como a diferençaentre seu maior elemento e seu menor elemento (em módulo).Calcule a soma dos diâmetros de todos os subconjuntos não vazios de {1, 2,..., n}.

PROBLEMA 5Temos um número finito de quadrados, de área total 4. Prove que é possível arranjá-los de modo a cobrir um quadrado de lado 1.Obs: É permitido sobrepor quadrados e parte deles pode ultrapassar os limites doquadrado a ser coberto.

PROBLEMA 6Arnaldo e Beatriz se comunicam durante um acampamento usando sinais de fumaça,às vezes usando uma nuvem grande, às vezes uma pequena.No tempo disponível antes do café da manhã, Arnaldo consegue enviar umaseqüência de 24 nuvens. Como Beatriz nem sempre consegue distinguir uma nuvempequena de uma grande, ela e Arnaldo fizeram um dicionário antes de ir para oacampamento. No dicionário aparecem N seqüências de 24 tamanhos de nuvem(como por exemplo a seqüência PGPGPGPGPGPGGPGPGPGPGPGP, onde Gsignifica nuvem grande e P significa nuvem pequena). Para cada uma das Nseqüências, o dicionário indica seu significado. Para evitar interpretações erradas,Arnaldo e Beatriz evitaram incluir no dicionário seqüências parecidas. Maisprecisamente, duas seqüências no dicionário sempre diferem em pelo menos 8 das 24posições.Demonstre que 4096≤N .

Page 29: Eureka 2003

Sociedade Brasileira de Matemática

EUREKA! N°16, 2003

29

SOLUÇÕES – NÍVEL 1

PROBLEMA 1: SOLUÇÃO DE MÁRCIO H. MORAES FERNANDES (RIO DE JANEIRO - RJ)Das informações dadas pelo problema conclui-se a seguinte propriedade:Propriedade:Como dois números não podem ficar na mesma coluna ou na mesma linha, sendoque para formar um conjunto são precisos 5 números e sabendo que o quadriculadopossui 5 linhas e 5 colunas, cada número do conjunto tem que ocupar uma linha euma coluna e conseqüentemente, cada linha e cada coluna estarão ocupadas por umnúmero do conjunto a ser formado.

A) A resolução mais simples para que dois números não se encontrem na mesmalinha ou na mesma coluna são as diagonais. Na diagonal do número 23, apenas onúmero 25 é maior que este. Assim peguei todos os números da diagonal menoso 25 que tive que substituir pelo 3 (que estava numa coluna que ainda não usara)e assim não pude utilizar o 20 se não repetiria a coluna, dessa forma o últimonúmero foi o 19 que estava em linha e coluna que não utilizei. ConjuntoA = {23, 7, 21, 3, 19}. O conjunto A é a solução para o item a).

B) O menor número que pode ser maior no conjunto de 5 números é o 5. Assim, fuieliminando os números na seqüência. O número 5 pode ser descartado porque o4 e o 3 estão na mesma linha e para fazer o conjunto sendo 5 o maior, os doisteriam que ser utilizados. O número 6 pode ser descartado porque na 4a. colunada esquerda para a direita, o único número menor que 6 está na mesma linha queele. Se o número 7 for usado, o único número menor que ele na segunda coluna,estará na mesma linha dele. Com o 8, na quarta coluna poderá ser escolhido o 7ou 5, escolhendo qualquer um, outros números não poderão ser utilizados: O 1(na segunda coluna) ou o 6 (na terceira coluna). Com o 9 e o 10, se foremescolhidos, o número 1 não poderá ser utilizado por estar na mesma linha, nãorestando outro número na segunda coluna a ser utilizado. E com o 11 pode sefazer um conjunto obedecendo a propriedade. Conjunto B = {1, 3, 6, 8, 11}sendo 11 o menor número possível.

PROBLEMA 2Veja a solução do problema No. 1 do Nível 2.

PROBLEMA 3: SOLUÇÃO DE LUCIO ASSAOKA HOSSAKA (CURITIBA – PR)Os possíveis valores de T são 12, 13, 14 e 15 pois 8 < T < 16 = 1 + 8 + 7 (note que4 + 5 + 6 < 16 é a maior soma possível de números fora de {1, 7, 8}).9 não é o valor de T pois 8 deve existir e, usando 3 números, é impossível fazer comque a soma de seu lado seja 9. O mesmo acontece com 10, já que não se pode usar 1

Page 30: Eureka 2003

Sociedade Brasileira de Matemática

EUREKA! N°16, 2003

30

e 1. Não pode, igualmente, ser 11 o valor de T pois 7 e 8 devem existir. O único jeitode 7 chegar a 11 com mais dois números, é 7, 1, 3, pois não se pode repetir números.O único jeito de somar dois números a 8 com o resultado 11 é 8, 1 e 2. Fazemosentão uma ilustração:

1 8 2

7

3

Restam para colocar, os números 4, 5 e 6. É impossível somar 2 desses númeroscom 3 resultando 11. Os únicos valores para T, são: 12, 13, 14 e 15.

3 7 2

8

1

12 ⇒

5 6

4

5 2 6

7

1

13 ⇒

8 4

3

7 6 1

3

4

14 ⇒

2 8

5

7 2 6

5

3

15 ⇒

4 8

1

PROBLEMA 4: SOLUÇÃO DE DEBORAH DE SÁ PEREIRA BELFORT (RECIFE – PE)a) Para conseguir desviar as sete moedas, foi preciso desvirar as cinco primeiras

moedas, e depois desvira-se as próximas cinco, e algumas voltarão a estarviradas no lado Coroa. Continuo com este ciclo até chegar o resultado:

= coroa = cara

1 2 3 4

Page 31: Eureka 2003

Sociedade Brasileira de Matemática

EUREKA! N°16, 2003

31

5 6 7

b) De 4 em 4, que é um número par, não se consegue as sete moedas viradas.Virando as moedas de 4 em 4, a quantidade de caras vai ser sempre número par; e 7é ímpar.

PROBLEMA 5Veja a solução do problema No. 3 do Nível 2.

SOLUÇÕES – NÍVEL 2

PROBLEMA 1: SOLUÇÃO DE ANDRÉ L. RIBEIRO DOS SANTOS (PINDAMONHANGABA - SP)

Y

X

B

Z

C

A1

Y'

Z' α

A2

X'

. .

.

α

.

'XYZ X YZ≅� � ˆ ˆ( ) ' 'LAL YZ X YZX α⇒ = = ' 'XZ X Z⇒ =

Logo // ' 'XZ X Z (olhe os ângulos formados pela transversal 'ZZ ).

Marque os pontos B e C no segmento XZ , como mostra a figura.

Seja 1'Y A a altura do 'BY C� , em relação a 'Y . Prolongue A1 até encontrar o

segmento ' 'X Z , formando 90° em A2.

Page 32: Eureka 2003

Sociedade Brasileira de Matemática

EUREKA! N°16, 2003

32

Agora, note que 2'Y A é a altura do ' ' 'Z Y X� , em relação a 'Y .

Chame a medida de XZ de med( ) med( ' ')y XZ y X Z⇒ = = .

Chame a medida de 1YA de 1med( )h YA h⇒ =

1YA é a altura do ZYX� , em relação a Y; portanto 1 2h YA YA= = que é a altura

correspondente no ' '.Z YX�

Como Y é simétrico a Y' em relação a XZ , então 1 1'YA Y A h= =

Assim 1 1 2' ,Y A YA YA h= = =

Área do 1

2 2 2

XZ YAb h yhXYZ

⋅⋅= = =�

Logo 1 .2

yh =

Área do 2' ' '' ' '

2 2

X Z Y Ab hX Y Z

⋅⋅= = =�

( )1 1 2' ( ) 3

2 2 2

X Z Y A YA Y A y h h h yh⋅ + + ⋅ + += = =

De 31 3

2 2yh yh= ⇒ =

Área do 2' ' ' 3 .X Y Z m=�

PROBLEMA 2: SOLUÇÃO DE EDUARDO FISCHER (ENCANTADO - RS)Um número divisível por 18 cumpre a condição. Um número assim possui a somados algarismos igual a 9 ou a 18 (27 só com o 999, que não é par). Qualquer númerodivisível por 18 é divisível por 9 e 18. Como em cada 18 números inteirosconsecutivos um é divisível por 18 o problema está resolvido.Resp. Entre quaisquer 18 inteiros consecutivos, um é divisível por 18. A soma dosalgarismos de um múltiplo de 18 (com 3 algarismos) é 18 ou 9.Em qualquer caso, o número é divisível pela soma dos algarismos.

PROBLEMA 3: SOLUÇÃO DE ANDRÉ L. RIBEIRO DOS SANTOS (PINDAMONHANGABA - SP)Para preencher todos os quadrados do tabuleiro, precisamos de um número ímpar depalitos, se as paridades de m e n forem diferentes; ou de um número par de palitos, seas paridades forem iguais:

i) m e n são de paridades diferentes: o primeiro jogador coloca o primeiro palito naposiçào central do tabuleiro e imita espelhadamente *(em relação ao palito) as

Page 33: Eureka 2003

Sociedade Brasileira de Matemática

EUREKA! N°16, 2003

33

jogadas do adversário. Haverá uma hora em que todos os quadrados serão ocupadoscom 2 palitos e será a vez do segundo jogador. Este por sua vez preenche um dosquadradinhos com o terceiro palito e o primeiro jogador o completa em seguida,vencendo o jogo.

ii) m e n são de paridades iguais: o segundo jogador copia as jogadas do primeiro,espelhadamente*, quando sobram todos os quadrados preenchidos com 2 palitos é avez do primeiro jogador, este preenche um quadrado com o terceiro palito, e osegundo jogador o completa ganhando o jogo.*espelhadamente: como se estivesse olhando para um espelho, tem a mesmaprofundidade mas é invertido lateralmente. Exemplos:

A

C

B

D

par x par

A C

B D

ímpar x ímpar

central

A

C

B

D

ímpar x par

central

Em todos os casos A está espelhando a B e C está espelhando D.Se m e n tem a mesma paridade o segundo jogador ganha, se tem paridadesdiferentes o primeiro ganha.

PROBLEMA 4: SOLUÇÃO DE THOMÁS YOITI SASAKI HOSHINA (RIO DE JANEIRO – RJ)Temos na mistura:

I 3

8A⇒ e

5

8B⇒

II1

3B⇒ e

2

3C⇒

III 2

5A⇒ e

3

5C⇒

Queremos que na mistura

IV3

,10

A⇒ 1

2B e

1

5C

Se pegarmos x da I, y da II e z da III teremos:

Page 34: Eureka 2003

Sociedade Brasileira de Matemática

EUREKA! N°16, 2003

34

3 2 3

8 5 101 5 1

3 8 22 3 1

3 5 5

A x z

B y x

C y z

⇒ + = ∴

⇒ + = ∴

⇒ + = ∴

15 16 12

15 8 12

10 9 3

x z

x y

y z

+ =+ =+ =

2y z=

6

29y =

3

29z =

Teriamos que 6

29y = e

3,

29z = logo

20

29x =

: : 20 : 6 : 3.x y z =

PROBLEMA 5: SOLUÇÃO DE THOMÁS YOITI SASAKI HOSHINA (RIO DE JANEIRO – RJ)

β 90°– α

α – γ

α

α γ

γ α – β 90° –α

α – γ

γ

T

N

E

P A

M

α α

B

Q

R

O

C

δ

α – γ

X

S

a) Chamemos ˆPQS de α, logo ˆ 90QAM α= °− e sendo ABO� isósceles

ˆ 90ABO α= °− , então ˆPSQ α= .

Logo PSQ� é isósceles.

b) Agora chamemos ˆPAQ γ= e ˆOBC β= teremos então que, como

ˆˆ ˆ ,BPS BXP PAB γ= = = APQ PSB≅� � , logo AQ PQ

PQ PS AQ BSPS BS

= ∴ ⋅ = ⋅

Page 35: Eureka 2003

Sociedade Brasileira de Matemática

EUREKA! N°16, 2003

35

Como queremos provar que 2

,PQ QR ST= ⋅ e ,PQ PS=

Basta apenas provar AQ BS QR ST⋅ = ⋅ ou AQ ST

BSQR=

Pelo ,AQM� AM AM

sen AQsenAQ

αα

= ∴ =

Pelo ,AQR� cos

(90 )

QR AR ARQR

sen sen sen

ββ α α

⋅= ∴ =−

Logo cos

AQ AM

QR AR β=

cos( )AM

ARα β− =

Pelo ,BST� cos( )

(90 )

ST BT BTST

sen sen sen

α βα β α α

−= ∴ =°− +

Pelo ,BSN� BN BN

sen BSsenBS

αα

= ∴ =

Logo cos( )

;ST BT

BS BN

α β−= cosBN

BTβ =

cos( ) cos

cos cos( )

AM BT AM BT

AR BN AR BN

α β ββ α β

−= ⇔ = ⇔⋅ −

cos( ) cos

cos( ) cos

α β βα β β

− = ⇔−

1 1.=

PROBLEMA 6: SOLUÇÃO ADAPTADA DE GABRIEL BUJOKAS (SÃO PAULO - SP)Seja pi o i-ésimo primo positivo.

1 21 2( ... )nee e

np p pϕ ⋅ ⋅ ⋅ 21 1 111 2 1( 1)...( 1);n ne ee

n np p p p p− −−= ⋅ ⋅ − − com 1 *,n e +∈ � (isso

vem diretamente da fórmula). Então basta escrever M! da forma ao lado direito daigualdade. Para M pequeno é fácil.

01! 2 (2 1) (2)ϕ= ⋅ − =2! 2 (2 1) (4)ϕ= ⋅ − =

0 13! 2 3 (2 1)(3 1) (18)ϕ= ⋅ − − =

Page 36: Eureka 2003

Sociedade Brasileira de Matemática

EUREKA! N°16, 2003

36

2 14! 2 3 (2 1)(3 1) (72)ϕ= ⋅ − − =Agora utilizarei indução. Seja 5np ≥ o n-ésimo primo. Suponha que para todo

, !nk p k< possa ser escrito na forma acima utilizando apenas primos menores

que np na fatoração. Então 11 21 2 1( 2)! ( ... )nee e

n np p p pϕ −−− = ⋅ ⋅ ⋅ implica

11 22 21 2 1! ( 1)( 2)! ( ) ( 2)! ( ... ).nee e

n n n n n n n np p p p p p p p p pϕ ϕ −−= − − = ⋅ − = ⋅ ⋅ ⋅ Para os m

com 1,n np m p +< < m é um produto de primos menores ou iguais a np , donde

! ( 1)!m m m= ⋅ − também é da forma acima. Conclusão: Para todo M existe um N tal

que ! ( ).M Nϕ=

SOLUÇÕES – NÍVEL 3

PROBLEMA 1: SOLUÇÃO DE THIAGO MORELLO PERES (RIO DE JANEIRO - RJ)Por absurdo, suponhamos a inexistência da seqüência satisfazendo o item b.Seja p um número primo maior que 2005003. Seja uma seqüência a progressãoaritmética de primeiro termo p e a razão p:

{ ,2 ,3 ...,2002 }A p p p p=Assim qualquer soma é do tipo n ⋅ p com n < p até mesmo para a soma total:

(1 2002) 20022005003

2p p

+ ⋅⋅ = ⋅

Garante-se assim, que a soma não é potência perfeita, quaisquer que sejam asparcelas desta.Como este exemplo não confere com a suposição, esta é um absurdo e, portantoexistem seqüências satisfazendo os itens a e b simultaneamente. cqd.

PROBLEMA 2: SOLUÇÃO DE ELDER RODRIGO B. CAMPOS (RIO DE JANEIRO - RJ)

C e M d D

a

B b A

c

k

θ

π – θ

Page 37: Eureka 2003

Sociedade Brasileira de Matemática

EUREKA! N°16, 2003

37

• 2 ( ) 2 ( )p ADM p ABCM a b e k c d k c d a b e∆ = ⇔ + + + = + + ⇔ + = + +� (I)

•( )

( ) .2 2

d e c sen ab senS ABCD

θ θ+= +�

( )2

cd senS ADM

θ∆ = ⋅ ora, se ( ) ( )S ADM S ABCM∆ = � e

( ) ( ) ( ) ( ) 2 ( )S ADM S ABCM S ABCD S ABCD S ADM∆ + = ⇒ = ∆ ⇔� � �

( ) 2 d e c sen ab sen cd sen ec ab cdθ θ θ+ + = ⇔ + = ⇔(II) ab = c(d – e). De (I): b + a – c = d – e.

(I) em (II) 2( )ab c a b c ab ac bc c⇒ = + − ⇔ = + − ⇔( ) ( ) ou a b c c b c b c a c− = − ⇒ = =

Logo, ABCD� tem dois lados de mesmo comprimento. cqd.

PROBLEMA 3: SOLUÇÃO DE HENRIQUE CHOCIAY (PINHAIS - PR)A numeração da tabela pode ser comparada com o preenchimento de uma malha depontos, observe:Ex.: Tabela 3 × 4

11 12 3

10

9

4

5

6

2 1

8 7

Malha 3 × 4

Início

Fim

O preenchimento da tabela é análogo à tarefa de passar por todos os pontos da malhacom uma linha única (sem "quebras" ou bifurcações).A ocorrência de i ao lado de (i + 3), por sua vez, é análoga às figuras:

Page 38: Eureka 2003

Sociedade Brasileira de Matemática

EUREKA! N°16, 2003

38

i i + 1

i + 2 i + 3 , , ,

na malha.O problema torna-se, então, provar que é impossível preencher a tabela sem realizaruma dessas figuras. A malha é formada por (m – 1)(n – 1) quadrados de 4 pontospróximos, os quais terão alguns de seus lados preenchidos ao fim do preenchimento.Se houver quadrado com 3 lados pintados, haverá

i i + 3

Ou i ao lado de i + 3.O total de lados dos quadrados (com multiplicidade) é 4(m – 1) ⋅ (n – 1) = t

Para fazer a linha, efetuamos (mn – 1) riscos, que podem preencher lados de 1 ou de2 quadrados.• Se o risco for feito na lateral da malha, preencherá apenas 1 lado de quadrado.Exemplo:

• Se o risco for feito no "miolo" da malha, preencherá dois lados de quadrado.Exemplo:

Supondo a distribuição mais homogênea de lados preenchidos, cada quadrado tem omesmo número de seus lados preenchidos.Se o número de lados preenchidos por riscos for maior que a metade do total delados de quadrados, haverá com certeza um quadrado com 3 lados riscados.

Page 39: Eureka 2003

Sociedade Brasileira de Matemática

EUREKA! N°16, 2003

39

2o t

N > �

O número de lados preenchidos por p riscos laterais é ( p ⋅ 1) = pO número de lados preenchidos por (mn – 1) – p riscos no meio é 2(mn – 1) – 2pO número total de riscos é: [2(mn – 1) – 2p] + (p) = 2mn – p – 2 = No.O número máximo de riscos laterais é:

n pontos

n – 1 riscos/lados

m – 1 riscos

2(m – 1) + 2(n – 1) – 1

Pmax = 2m + 2n – 5

O número mínimo de lados preenchidos é 2mn – Pmax – 2 = 2mn – 2m – 2n + 3 == No min.

Se min2

o tN > � , fica provado que há (ou similar) e i ao lado de i + 3

4( 1)( 1)2 2 2 3

22 2 2 3 2 2 2 2

3 2

m nmn m n

mn m n mn m n

− −− − + >

− − + > − − +>

O número de lados preenchido é maior que a metade do total de lados.

Há e portanto há i ao lado de i + 3 para qualquer tabela.

PROBLEMA 4: SOLUÇÃO ADAPTADA DE RODRIGO KENDY YAMASHITA (SÃO PAULO - SP)Sejam m e M as somas dos elementos mínimos e máximos dos subconjuntos. Comoo diâmetro de um conjunto é definido como a diferença entre seu máximo e seumínimo, a soma desejada é igual a M – m. Note que podemos incluir os subconjuntosunitários, já que seus máximos e mínimos coincidem.

Page 40: Eureka 2003

Sociedade Brasileira de Matemática

EUREKA! N°16, 2003

40

O número , 1 ,k k n≤ ≤ é elemento mínimo dos subconjuntos da forma

{ } ,k A∪ sendo A um subconjunto de { 1; 2;...; }.k k n+ + Logo k é elemento mínimo

de 2n k− subconjuntos.

Conseqüentemente, 1

1 0

2 ( ) 2 .n n

n k k

k k

m k n k−

= == ⋅ = − ⋅∑ ∑

Contemos o número de subconjuntos de diâmetro k. Seja a o mínimo de um dessessubconjuntos. O seu máximo é, então, a + k. Assim, .a k n a n k+ ≤ ⇔ ≤ − Logopodemos escolher a de n – k maneiras. Como há k – 1 números entre a e a + k,

podemos escolher os demais elementos do subconjunto de 12k− maneiras. Logo há1( ) 2kn k −− ⋅ subconjuntos de diâmetro k. Como há, no total, 2 1n n− − subconjuntos

não vazios e não unitários,1 1 1

1 1 1

1 1 1

( ) 2 2 1 2 ( ) 2 2 2 2 ( ) 2n n n

k n k n k

k k k

n k n n k n n k− − −

− − +

= = =− ⋅ = − − ⇔ − ⋅ = − − ⇔ − ⋅ =∑ ∑ ∑

11 1 1

0

2 2 2 ( ) 2 2 2 2 2 2n

n k n n

k

n n k n n n−

+ + +

== − − ⇔ − ⋅ = − − + = − −∑Logo 12 2.nm n+= − −Para calcular M, basta observar que podemos associar cada conjunto

1 2{ ; ;...; }nA a a a= ao conjunto 1 2( ) { 1 ; 1 ;...; 1 },nf A n a n a n a= + − + − + − de

modo que se a = mín A então 1 max ( ).n a f A+ − = A função f é claramente uma

bijeção. Logo, como há 2 1n − subconjuntos não vazios,

( 1) (2 1) ( 1) 2 1 2n nM n m M m n n m M m= + ⋅ − − ⇔ − = + ⋅ − − − ⇔ − =1( 1) 2 1 2 (2 2) ( 3) 2 3.n n nn n n n n+= + ⋅ − − − ⋅ − − = − ⋅ + +

PROBLEMA 5: SOLUÇÃO DE RAFAEL DAIGO HIRAMA (CAMPINAS – SP)Podemos supor então que os quadrados têm lado menor que 1, caso contrário é sóposicionar o quadrado de 1 ou mais sobre o quadrado a se cobrir.Vamos classificar os quadrados como do tipo Qk tal que o lado do quadrado seja

menor que 1

1

2k− e maior ou igual a 1

2k.

Se tivermos um Q0 acabou, pois ele terá lado maior ou igual a 1 e pronto.Caso contrário vamos dividir o tabuleiro em 4 partes iguais. Cada uma tem lado 1/2,ou seja, é satisfatoriamente coberto por um Q1 cada um. Então se posiciona todos osdisponíveis. Se tiver pelo menos 4 Q1 acabou.

Page 41: Eureka 2003

Sociedade Brasileira de Matemática

EUREKA! N°16, 2003

41

Caso contrário os que sobraram devem ser divididos em 4 e preenchidos por quantosQ2 tiverem. E assim sucessivamente até preencher o tabuleiro.Exemplo:

Q1

Q1

Q2 Q2 Q2 Q2

Q2 Q2Q2 Q3 Q3

Q3

Q3

Agora, para provar que isso sempre é possível basta provar que a área total dosquadrados usados é menor que 4. Assim, já que o modo de preenchimento pede "usetantos do Qk quanto existirem", se sobrar buraco ou esqueceu-se de usar umquadrado em um passo anterior ou falta usar os quadrados menores.Para isso vamos ver o desperdício de cada quadrado, ou seja, quanto do quadradonão usamos para preencher a área de interesse (por exemplo, o desperdício de umquadrado Q3 ao ser colocado sobre um tabuleiro de lado 1/8 é o quanto do quadradoficará de fora desse tabuleiro, mesmo que esse resto esteja sobre outra parte dotabuleiro total ele vai ser contado como desperdício).

Vejamos, como sempre usamos Qk para cobrir um tabuleiro de lado 1

2k, a área do

Qk é 2 2

1

2 k− no máximo e 2

1

2 kno mínimo e a do tabuleiro é

2

1

2 k, logo o desperdício

é de 2 2 2 2

1 1 3

2 2 2k k k− − = no máximo, isso prova que o desperdício não passa de

2

2

32 3 vezes da área1

2

k

k

= preenchida, ou seja, é desperdiçado no total no máximo 3/4

da área dos quadrados utilizados, ou seja, 1/4 pelo menos é utilizado. Como o totalda área dos quadrados é 4, a área utilizada é pelo menos 1, o que termina oproblema.

Page 42: Eureka 2003

Sociedade Brasileira de Matemática

EUREKA! N°16, 2003

42

PROBLEMA 6: SOLUÇÃO DE FÁBIO DIAS MOREIRA (RIO DE JANEIRO - RJ)Definição: A distância entre duas palavras p e q é o número de posições em que duaspalavras diferem (símbolo: d(p, q)).

Teorema 1: d(p, q) + d(q, r) ≥ d(p, r).Prova: Seja αβ∆ o conjunto das posições em que α e β diferem. Então o teorema

equivale a #( ) #( ) #( ).pq qr pr∆ + ∆ ≥ ∆ Mas ( ) ( )pr pq qr pq qr∆ = ∆ ∪∆ − ∆ ∩∆ , pois

só há dois tipos de nuvens, logo p e r são iguais nas posições onde ambos diferem deq. Mas pq qr pq qr∆ ∩∆ ⊂ ∆ ∪ ∆ , logo #( ) #( ) #( )pr pq qr pq qr∆ = ∆ ∪∆ − ∆ ∩∆ =

#( ) #( ) 2#( )pq qr pq qr= ∆ + ∆ − ∆ ∩∆ , e nossa afirmação equivale a #( ) 0,pq qr∆ ∩∆ ≥obviamente verdadeiro.

Definição: A palavra real mais próxima a uma palavra q é a palavra p que:i) Pertence ao dicionário.ii) Minimiza a distância entre p e q.(se existir mais de uma palavra que atende i) e ii) todas elas são mais próximas).

Definição: A vizinhança de uma palavra p pertencente ao dicionário é o conjunto detodas as palavras mais próximas a p (símbolo: pε ).

Teorema 2: Toda vizinhança de uma palavra p contém todas as palavras cujasdistâncias até p sejam menores ou iguais a 4.Prova: Seja q tal que ( , ) 4d p q ≤ mas .pq ε∉ Então rq ε∈ para r pertencente ao

dicionário. Isso implica ( , ) ( , ) 4,d q r d p q< ≤ logo ( , ) ( , ) ( , ) 4 4 8d p r d p q d q r≤ + < + = ,absurdo, pois p e r não poderiam estar simultaneamente no dicionário.Teorema 3: Toda palavra p pertence a no máximo seis vizinhanças.Prova: Suponha que

1 2 3 4 5 6 7.q q q q q q qp ε ε ε ε ε ε ε∈ ∩ ∩ ∩ ∩ ∩ ∩ Como

( , ) ( , ) ( , ), ( , ) 8.i j i j i jd q q d p q d p q d q q≤ + ≤ Como ( , ) 8, ( , ) 8.i j i jd q q d q q≥ =Como ( , ) ( , ), ( , ) 4, {1,...,7}.i j id p q d p q d p q i= = ∀ ∈ Como cada palavra só tem

24 nuvens, existem duas palavras (q1 e q2, sem perda de generalidade) tais que

1 2.pq pq φ∆ ∩∆ ≠ Mas então, pelos argumentos do teorema 1,

1 2 1 21 2 1 2 1 2( , ) ( , ) ( , ) 2#( ) ( , ) 8 2#( ) 8 2 1 6pq pq pq pqd q q d p q d p q d q q= + − ∆ ∩∆ ⇒ = − ∆ ∩∆ ≤ − ⋅ =absurdo, pois q1 e q2 não poderiam pertencer simultaneamente ao dicionário.

N é máximo quando todas as palavras distam no máximo quatro da palavra dodicionário mais próxima a ela e todas as palavras que distam exatamente quatro da

Page 43: Eureka 2003

Sociedade Brasileira de Matemática

EUREKA! N°16, 2003

43

palavra do dicionário mais próxima pertencem a seis vizinhanças, já que issocaracteriza a formação mais densa possível, devido ao seguinte teorema:

Teorema 4: , ( , ) ( , ) 4.q rp q r d p q d p rε ε∈ ∩ ≠ ⇒ = =Prova: Suponha que ( , ) ( , ) 4d p q d p r= < (a igualdade é obrigatória pela definição

de vizinhança). Então ( , ) ( , ) ( , ) 4 4 8d q r d p q d q r≤ + < + = , absurdo, pois as duaspalavras não poderiam pertencer simultaneamente ao dicionário.Porquê isso valida nossa afirmação acima? Porque nenhum ponto que dista três oumenos ao ponto mais próximo pertence a mais de uma vizinhança. Assim, o arranjodescrito acima é o mais denso, pois todas as palavras que não pertencem aodicionário estão sempre cercadas por palavras do dicionário (pertencem sempre aonúmero máximo de vizinhanças).Nas circunstancias acima descritas #( ) #( )p qε ε= para todo p e q (pois

0 1 2 3 424 24 24 24 24C C C C C+ + + + é constante e igual a #( )pε por (1)). Além disso,

#( )pp D

ε∈∑ , onde D é o dicionário, seria 242 , mas não, é pois as palavras que distam

quatro de uma palavra no dicionário são contadas seis vezes. Vamos achar então 'pn ,

com um fator de correção apropriado:

' 0 1 2 3 424 24 24 24 24pn C C C C C= + + + +

d = 0(própriapalavra)

d =1 d =2 d =3

d =4

Vamos contar só uma vez!

6

' 106261 24 276 2024 4096

6pn = + + + + = (que coincidência!)

Mas até agora consideramos o melhor caso – há algum desperdício de palavrasenvolvido. Logo algumas vizinhanças são maiores do que são no caso ideal. Por isso,

' 4096pn ≥ em geral.

Assim 24 12 122 4096 2 2 4096.N N N≥ = ⇒ ≤ =

Page 44: Eureka 2003

Sociedade Brasileira de Matemática

EUREKA! N°16, 2003

44

XXIV OLIMPÍADA BRASILEIRA DE MATEMÁTICAProblemas e Soluções da Primeira Fase – Nível Universitário

PROBLEMA 1A função �→+∞− ),1(:f é contínua e derivável.Sabe-se que f(0) = 0, f '(0) = a e que f(x + 1) = e f(x) para todo x > – 1.Calcule f '(3).

PROBLEMA 2Seja A a matriz real n × n

.

...

...

+

++

=

yxxx

xyxx

xxyx

A

����

Diga para que valores de x e y a matriz A é inversível e calcule A– 1.

PROBLEMA 3

Calcule ∫− +++

−++1

1 2

2

.11

11dx

xx

xx

PROBLEMA 4Determine todos os valores inteiros positivos de m para os quais o polinômio(x + 1)m + xm + 1 é divisível por (x2 + x + 1)2.

PROBLEMA 5Jogamos 10 dados comuns (com 6 faces equiprováveis numeradas de 1 a 6).Calcule a probabilidade de que a soma dos 10 resultados seja igual a 20.

PROBLEMA 6

Considere a curva .16643),( }{ 322 +−=∈= xxyyxC �

a) Seja Q = (a, b) um ponto de C. Suponha que a reta tangente a C no ponto Qintersecte C num único outro ponto, Q'. Determine as coordenadas de Q'.

b) Seja P0 = (3, 8). Para cada inteiro não negativo n, definimos Pn +1 = nP' , oponto de interseção de C com a reta tangente a C em Pn. Determine P2002.

Page 45: Eureka 2003

Sociedade Brasileira de Matemática

EUREKA! N°16, 2003

45

SOLUÇÃO DO PROBLEMA 1

Derivando a equação )()1( xfexf =+ temos .)(')1(' )( xfexfxf ⋅=+

Assim ,1)1( 0 == ef aeff f =⋅= )0()0(')1('

,)2( 1 eef == aeeff f =⋅= )1()1(')2('(2) 1'(3) '(2) .f ef f e ae += ⋅ =

SOLUÇÃO DO PROBLEMA 2Se y = 0 todas as linhas são iguais e a matriz não é inversível. Se nx + y = 0 a somadas n linhas é 0 e portanto a matriz novamente é não inversível. Vamos mostrar quese nenhuma destas duas condições ocorre a matriz é inversível.

Se

1 1 ... 1

1 1 ... 1

1 1 1

B

=

� � � �

temos B2 = nB e A = yI + xB.

Tome .)(

1B

ynxy

xI

yC

+−=

( ) IBynxy

xI

yxByIAC =

+

−+=)(

1

Comentário (não faz parte da solução)

Encontramos C conjecturando que .1 vBuIA +=−

E resolvendo um sistema para encontrar u e v. Pode-se demonstrar antes de tentarresolver o sistema que A–1, se existir, deve ter a forma acima: A– 1 é uma funçãoanalítica de A, logo um polinômio em A, logo um polinômio em B. Comoobservamos que B2 é um múltiplo escalar de B segue que todo polinômio em B é daforma uI + vB.

SOLUÇÃO ALTERNATIVAVamos resolver o sistema

=⋅+++⋅+⋅

=⋅++++⋅=⋅++⋅++

nn

n

n

bayxaxax

baxayxax

baxaxayx

)(...

...)(

...)(

21

221

121

Page 46: Eureka 2003

Sociedade Brasileira de Matemática

EUREKA! N°16, 2003

46

Somando todas as equações, obtemos ),...()...)(( 11 nn bbaaynx ++=+++ donde

)...()...( 11 nn bbynx

xaax ++

+=++ , caso nx + y ≠ 0.

Diminuindo essa igualdade da j-ésima equação, obtemos

)...( 1 njj bbynx

xbay ++

+−=⋅ e, caso

y ≠ 0, .)(

...)(

)1(...

)( 1 njj bynxy

xb

ynxy

yxnb

ynxy

xa

+−−

++−+−

+−=

Assim, .

)(

)1(

)(

)(

)( ...

)(

)1(

)(

)( ...

)(

)(

)1(

1

++−

+−

+−

+−

++−

+−

+−

+−

++−

=−

ynxy

yxn

ynxy

x

ynxy

x

ynxy

x

ynxy

yxn

ynxy

x

ynxy

x

ynxy

x

ynxy

yxn

A

���

Note que, se nx + y = 0, o sistema não tem solução se b1 +...+ bn ≠ 0, e, se y = 0, o

sistema não tem solução se 0)...( 1 ≠+++

− nj bbynx

xb para algum j . Em nenhum

desses casos A é invertível.

SOLUÇÃO DO PROBLEMA 3

Seja .11

11)(

2

2

+++

−++=xx

xxxf Racionalizando, temos

,2

11

)1()1(

11)11()(

22

2

22

22

x

xx

xx

xxxxxf

−+=

+−+

−−+−++= logo ),()( xfxf −=− para

todo x, e portanto, ∫− =1

1.0)( dxxf

SOLUÇÃO ALTERNATIVA

Vamos achar uma primitiva de f: Em dxxx

xx∫+++

−++

11

112

2

fazemos x = tan θ, dx =

sec2θ dθ , e, como θθ sec1tan 2 =+ (para ),22

πθπ <<− obtemos

∫ ⋅++−+ θ

θθθθ 2sec

1tansec

1tansec =θd

Page 47: Eureka 2003

Sociedade Brasileira de Matemática

EUREKA! N°16, 2003

47

∫ ++−+

.)cos1(cos

cos12

θθθθ

θθd

sen

sen

Fazendo ,2

tan z=θ,

1

22z

dzd

+=θ ,

1

1cos,

1

22

2

2 z

z

z

zsen

+−=

+= θθ obtemos

22 2 2

2 2 2 2

2 2 1 2 2 (1 ).

2 2 1 1 (1 )

z z z dz z z dz

z z z z

+ + +⋅ = + − + − ∫ ∫

Buscando A, B, C, D tais que ,)1(

)1(2

)1(1)1(1 22

2

22 z

zz

z

D

z

C

z

B

z

A

−+=

−+

−+

++

+ obtemos (A

+ B + Az)(1 – z)2 + (C + D – Cz)(1+z)2 = 2z(1 + z2), donde A – C = 2, B – A + D – C= 0, –A –2B + C + 2D = 2, A + B + C + D = 0. Assim, D = –B, C = – A, logo A = 1,C = –1, D = 1, B = –1.

Assim, ∫ −+−=

−+−+

+++=

−+

.1

2)1ln(

1

1)1ln(

1

1)1ln(

)1(

)1(22

2

22

2

zz

zz

zzdz

z

zz

Quando x varia entre – 1 e 1, θ varia entre 4

π− e

4

π, donde z varia entre

8tan

π

e

8tan

π. Temos 24

tan 2 1,8 2 21 cos

4

senπ

ππ

= = = − + +

donde z varia entre 21− e

.12 − Assim, a integral é ,021

12

1

2)1ln(

2

2 =−

−−

+−z

z pois .)21()12( 22 −=−

SOLUÇÃO DO PROBLEMA 4

Seja 2

3

2

1 i+−=ω uma raiz de x2 + x + 1. Para que (x2 + x + 1)2 divida

(x + 1)m + xm + 1 = P(x), devemos ter P(ω) = 0 e P'(ω) = 0.

Assim, 1)1( −=++ mm ωω e .0))1(( 11 =++ −− mmm ωω

Temos que 2

3

2

11

i+=+ω é tal que ωω =+ 2)1( e 1)1( 3 −=+ω . Como ω e ω + 1

são raízes sextas da unidade, o comportamento se repetirá com período 6.

Assim, 0)1( 11 =++ −− mm ωω equivale a )1(2311 )1()1( −+−− +=−=+ mmm ωωω , ou seja

,1)1( 2 =+ +mω o que equivale a m ≡ –2 (mod 6). Nesse caso, temos

,1)1()1( 24 −=+=++=++ ωωωωωω mm donde as duas condições sãosatisfeitas. Assim, os números que satisfazem o enunciado são os inteiros positivosda forma 6k – 2.

Page 48: Eureka 2003

Sociedade Brasileira de Matemática

EUREKA! N°16, 2003

48

SOLUÇÃO DO PROBLEMA 5Devemos encontrar o número de soluções de ,20... 1021 =+++ aaa .61 ≤≤ ia

O número de soluções de ,20... 1021 =+++ aaa 1≥ia é claramente

9

19.

Devemos agora descontar as soluções para as quais apenas um dentre os ai é ≥ 7 poiscaso contrário tal soma seria ≥ 7 + 7 + 8 ⋅ 1 = 22. Assim, basta descontar 10 vezes onúmero de soluções de ,20... 1021 =+++ aaa 1 7a ≥ , 1≥ia ,

ou de ,14...~1021 =+++ aaa 1, 1,ia a ≥� que é

9

13 . Assim

=

9

1310

9

19N e a

probabilidade pedida é 106

9

1310

9

19

=p .

SOLUÇÃO DO PROBLEMA 6

a) De ,1664332 +−= xxy temos ,4332 2 −= xdx

dyy donde .

2

433 2

y

x

dx

dy −=

A equação da reta tangente a C passando por (a, b) é

.2

433

2

433 22

−−+

−= ab

abx

b

ay Substituindo em 1664332 +−= xxy temos

,2

4332

2

43316643

2322

3

+−+

−=+−b

aabx

b

axx que terá uma raiz dupla em x = a,

e cuja soma das raízes é .2

43322

−b

a Assim, o outro ponto terá primeira coordenada

igual a ab

a2

2

43322

− , e, substituindo na equação da reta, segunda coordenada

igual a

+−+

−=+−+

−−

−b

aab

b

a

b

aab

b

aa

b

a

2

12992

2

433

2

4332

2

4332

2

433 323232232

.

b) Usando a fórmula acima, obtemos P1 = (–5, 16), P2 = (11, 32), P3 = (3, – 8), P4 =(–5, – 16), P5 = (11, – 32) e P6 = (3, 8). Assim, a seqüência (Pn) é periódica deperíodo 6, logo )16,5(42002 −−== PP .

Observação: No item b), o fato de P3 diferir de P0 apenas por uma troca de sinal dasegunda coordenada já é suficiente para concluir que a seqüência é periódica deperíodo 6.

Page 49: Eureka 2003

Sociedade Brasileira de Matemática

EUREKA! N°16, 2003

49

XXIV OLIMPÍADA BRASILEIRA DE MATEMÁTICAProblemas e Soluções da Segunda Fase – Nível Universitário

PROBLEMA 1Seja y = P(x) um polinômio de grau 4. Mostre que se existe uma reta (em �2) quecorta o gráfico de P em 4 pontos então existe uma reta que corta o gráfico em 4pontos igualmente espaçados.

PROBLEMA 2

)( ijaA = uma matriz real simétrica nn× tal que 1=iia e ∑=

<n

jija

1

2 , para todo

},...,2,1{ ni ∈ . Prove que .1det0 ≤< A

PROBLEMA 3

Sejam },...,2,1{,...,, 21 nAAA k ⊂ conjuntos com 2

nAi ≥ e

4

nAA ji ≤∩ para

todo i, j com ji ≠ . Prove que 1

.1

k

ii

kA n

k=≥ ⋅

+�

PROBLEMA 4

Determine todas as soluções reais da equação .222 xx +−+=

PROBLEMA 5Dado x ∈ �, definimos xx =)(ln 0 e, para cada k ∈ �, se 0)(ln >xk , definimos

)),(ln(ln)(ln 1 xx kk =+ onde ln é o logaritmo natural.

Dado n inteiro positivo, definimos k(n) como o maior k tal que 1)(ln ≥nk , e an como( )

( )0

ln ( ) ln( ) ln ln( ) ... ln ( ).k n

j k nj

n n n n n=

= ⋅ ⋅ ⋅ ⋅∏

Diga se a série ∑∞

=1

1

n na converge ou diverge.

PROBLEMA 6Considere duas elipses no plano �2 que se intersectam em 4 pontos. Nestes 4 pontostrace as retas tangentes às duas elipses, obtendo assim 8 retas.Prove que existe uma elipse (ou circunferência) tangente a estas 8 retas.

Page 50: Eureka 2003

Sociedade Brasileira de Matemática

EUREKA! N°16, 2003

50

SOLUÇÕES – NÍVEL UNIVERSITÁRIO

PROBLEMA 1: SOLUÇÃO DE FABRÍCIO SIQUEIRA BENEVIDES (FORTALEZA – CE)

Seja 4 3 2( )P x ax bx cx dx e= + + + + e seja ( )x kx q= +� a reta que o intersectaem 4 pontos.Ou seja, ( ) ( ) ( )Q x P x x= −� tem quatro raizes.Queremos mostrar que existe ( )r x tx s= + tal que ( ) ( ) ( )S x P x r x= − temquatro raizes igualmente espaçadas.

4 3 2

4 3 2

( ) ( ) ( )

' '.

S x ax bx cx d t x e s

ax bx cx d x e

= + + + − + −= + + + +

Note que nosso problema é equivalente a dados a, b, c achar d', e' tais queS(x) acima tenha 4 raizes igualmente espaçadas.Primeiro, mostraremos que é possível escolher d' de modo que S(x) seja

simétrico em relação à reta ,4

bx

a

−= isto é, ,4 4

b bS k S k k

a a

− − − = + ∀ ∈ � .

Para escrever menos seja 4

bu

a

−= .

4 3 2( ) ( ) ( ) ( ) ( ) '( ) 'S u k S u k a u k b u k c u k d u k e− = + ⇔ − + − + − + − + =4 3 2( ) ( ) ( ) '( ) 'a u k b u k c u k d u k e= + + + + + + + + ⇔

4 3 2 2 3 4 3 2 2 3 2 2( 4 6 4 ) ( 3 3 ) ( 2 )a u u k u k uk k b u u k uk k c u uk k dk⇔ − + − + + − + − + − + − =4 3 2 2 3 4 3 2 2 3 2 2( 4 6 4 ) ( 3 3 ) ( 2 ) 'a u u k u k uk k b u u k uk k c u uk k d k+ + + + + + + + + + + + =

3 3 2 38 8 6 2 4 2 ' 0au k auk bu k bk cuk d k⇔ + + + + + =3 38 2

4

bu auk bk

a

− = ⇒ = − 3 2(4 3 2 ') 0au bu cu d k⇔ + + + ⋅ =

Basta então tomar 3 2' 4 3 2d au bu cu= − − −O fato de já existir uma reta que intersecta o P(x) inicial em 4 pontos, nos dizque a, b, c nos foram dados de modo que S(x) tenha 3 pontos de máx/mínlocais. (Senão o gráfico de S(x) seria convexo ou côncavo, e qualquer reta ointersectaria em no máximo 2 pontos).Logo, o gráfico de S(x) é algo do tipo:

Page 51: Eureka 2003

Sociedade Brasileira de Matemática

EUREKA! N°16, 2003

51

ou

4

bx

a

−=

Finalmente mudar e' significa transladar o gráfico de S para cima ou para baixo.Claramente podemos escolher um e tal que S tenha 4 raízes 1 2 3 4, , , .e e e ex x x x

s

r

f

Para cada e' desses considere as funções

2 1( ) e ef e x x= −

3 2( ) e eg e x x= − , para

(ver gráfico)

r e s≤ ≤

4 3( ) e eh e x x= −Pela escolha de d', S é simétrico e f(e) = h (e).

( ) 0, ( ) 0f r g r= >( ) 0, ( ) 0f s g s> =

Pelo T.V.I. existe ' ( , )e r s∈ tal que ( ') ( ').f e g e=Neste caso, ( ') ( ') ( ')f e g e h e= = , e este é o nosso tão procurado e'.

PROBLEMA 2: SOLUÇÃO DE HUMBERTO SILVA NAVES (S.J. DOS CAMPOS - SP)Temos que a matriz A é diagonalizável, pois é simétrica, ou seja:A = H ⋅ D ⋅ H –1 onde H T = H –1 e D é uma matriz diagonal formada pelos auto-valores de A.Obs. As matrizes D e H são reais.Claramente det. A = det. D.Primeiramente vamos provar que todos os auto-valores de A são positivos:Seja X um auto-vetor de A, i.e., X é uma matriz n × 1, não nula, tal que

Page 52: Eureka 2003

Sociedade Brasileira de Matemática

EUREKA! N°16, 2003

52

AX Xλ=1

n

x

X

x

=

Seja i, tal que 1 20 max{ ; ;...; }i nx x x x< = . Se fosse 0λ ≤

teríamos: 1 1 2 2 ...i i i in nx a x a x a xλ = + + + ⇔

1 1 2 2 1 1 1 1( 1) ... ...i i i ii i ii i in nx a x a x a x a x a xλ − − + +− = + + + + + +

Temos ( )1 i ix xλ − ≥ e

1 1

1 ,ij ij j ij i j i

j n j n

a a x x≠ ≠≤ ≤ ≤ ≤

< ⇒ <∑ ∑ pois

1max{ ;...; },i nx x x= logo ( )1 1

1i i ij j ij j ij i j i

j n j n

x x a x a x xλ≠ ≠≤ ≤ ≤ ≤

≤ − = ≤ <∑ ∑ um

absurdo! Logo det D > 0 ⇒ det A > 0.Claramente um auto-valor de A é uma raiz de P(x) = det (A – xI).

O coeficiente de 1nx − de P(x) é a soma da diagonal principal de A multiplicada por

( ) 11

n−− , ou seja, ( ) 11

nn

−− . Logo a soma das raízes de P(x) (com suas respectivas

multiplicidades) é ( ) 1

1.

( 1)

n

n

nn

−− −=

−Temos: 1 2 ... n nλ λ λ+ + + = onde os λi's são os auto-valores de A, com 0 i iλ ≥ ∀ ;

1 ≤ i ≤ n. Pela desigualdade das médias, temos:

1 21 2 1 2 3

...1 ... ... 1n n

n nn

λ λ λ λ λ λ λ λ λ λ+ + += ≥ ⇒ ≤

Mas det 1 2... 1 det 1.nD Aλ λ λ= ≤ ⇒ ≤

PROBLEMA 3: SOLUÇÃO DE CARLOS STEIN NAVES DE BRITO (S.J. DOS CAMPOS - SP)

Seja 1

k

ii

A U=

=�

Seja 1 2

k

ii

nS A k

== ≥ ⋅∑ (I)

Por absurdo, suponha que 1

kU n

k< ⋅

+ (II)

Page 53: Eureka 2003

Sociedade Brasileira de Matemática

EUREKA! N°16, 2003

53

Seja ai quantas vezes os elementos de Ai aparecem nos outros ( )jA j i≠ , por

exemplo, se 1 {1,2,3}A = , 2 {1,2,5}A = e 3 {1,3,6}A = , temos que a1 = 4. Temos

1

k

i i jjj i

a A A=≠

= ∩∑

Para cada 1

k

ii

t A=

∈ � , o seja bt o número de Ai's que contém t.

Lema:

1

1

( 1).k

ii

k

i j ji

j A

a b b

=

=∈

= −∑ ∑�

Prova: Temos que ai é quantas vezes aparece cada it A∈ em outros ( )jA j i≠ , logo

cada t aparece em ( 1)tb − outros Aj, pois bt conjuntos contém t, tirando o Ai, logo

temos ( 1)tb − outros que contém t. Assim 1 1

( 1) ( ( 1)).i i

k k

i t i tt A i i t A

a b a b∈ = = ∈

= − ⇒ = −∑ ∑ ∑ ∑

Para cada 1

k

ii

t A=

∈ � , existem bt conjuntos que contém t, logo cada parcela 1tb −

aparece bt vezes em 1

( 1)i

k

ti t A

b= ∈

∑ ∑ , logo

1

1

( 1).k

ii

k

i t ti

t A

a b b

=

=∈

= −∑ ∑�

cqd.

Logo

1 1 1

2 2

1 k k k

i i ii i i

k

i t t t ti

t A t A t A

a b b b b

= = =

=∈ ∈ ∈

= − = −∑ ∑ ∑ ∑� � �

(note que

1

1 2k

ii

k

t ii

t A

nb A k

=

=∈

= ≥ ⋅∑ ∑�

, pois

estamos contando quantas vezes aparece cada elemento). Por Cauchy-Schwarz:

1

1

2

22 2 2 2

212

1

( 1)4 4( )4

1

k

ii

k

ii

kt

t A ii

t k

t A ii

bk n k nA

S k k nb

kU U U nAk

=

=

∈=

=

⋅ ⋅ + ⋅ ≥ = = ≥ > =

⋅+

∑∑

∑�

� �

2

1

( 1) ( 1)( 1 2)

4 2 4 4

k

i t ti

k kn kn kn k n ka b b k

=

+ ⋅ −→ = − > − = + − =∑ ∑ ∑

Page 54: Eureka 2003

Sociedade Brasileira de Matemática

EUREKA! N°16, 2003

54

Pelo princípio das casas dos pombos, ja∃ , tal que ( 1)

4j

k n k

ak

⋅ ⋅ −

> (senão

( 1)

4i

kn ka

−≤∑ , absurdo).

Logo ( 1),

4j j

n ka a

−∃ > . De novo pela casa dos pombos, como 1

,k

j j iii j

a A A=≠

= ∩∑

existe um p tal que

( 1)

41 1 4 4

jj p j p

n ka n n

A A A Ak k

∩ ≥ > = ⇒ ∩ >− −

, absurdo.

Logo 1

kU n

k≥ ⋅

+.

PROBLEMA 4: SOLUÇÃO DE MÁRCIO AFONSO ASSAD COHEN (RIO DE JANEIRO - RJ)É, tá difícil…vamos tentar uma idéia:

Seja 2cosx α= , com (0, / 3)α π∈ (Ok, pois já sei que 1 < 2≤ x 2 2≤ + < 2).

Obs: 2 2cos 2 2cos 1 1 2senθ θ θ= − = −22 2(1 cos ) 4cos ( / 2) 2 2cos( / 2)x xα α α+ = + = → + =

(pois / 2 (0, / 6)α π∈ logo cos( / 2) 0α > ).

2 2cos( / 2) 2 2 2 2cos( / 2) 2(1 cos( / 2))x xα α α+ = → − + = − = − =22 2 ( / 4) 2 2 2 ( / 4)sen x senα α= ⋅ → − + =

(pois ( / 4) 0sen α > ).

Logo, 2 2 ( / 4) O .: ( / 4) cos( / 2 / 4)x sen bs senα α π α= + → = −

[ ] 22 2 ( /4) 2 2cos( /2 /4) 2 1 cos( /2 /4) 2 2 cos ( /4 /8)sen α π α π α π α + = + − = ⋅ + − = ⋅ − Portanto, tirando raiz: 2cos( / 4 / 8), . .x i eπ α= −2cos 2cos( / 4 / 8) cos cos( / 4 / 8)α π α α π α= − → = −

/ 4 / 8 9 /8 / 4 2 / 9α π α α π α π→ = − → = → =Logo, 2cos 2 / 9x π= é a única solução real da equação.

Page 55: Eureka 2003

Sociedade Brasileira de Matemática

EUREKA! N°16, 2003

55

PROBLEMA 5: SOLUÇÃO DA BANCA

Sejam b0 = 1, kbk eb =+1 para todo k ∈ �.

Para 1+<≤ kk bnb temos k(n) = k.

Como a derivada de 1ln ( )k x+ é 0

1: ( ),

ln ( )

k

kj j

g xx=

=∏ temos, para cada n,

1

1 1ln ( 1) ln ( ) ( ) ( ),n

k k k knn n g x dx g n

+

+ ++ − = ≤∫ para todo n ≥ bk, pois gk é decrescente.

Assim, ( ) ( )1 1

1 1 1

1( ) ln ln .

k k k k

k k k k kb n b b n bn

g n b ba

+ +

+ + +≤ < ≤ <

= ≥ − ∑ ∑

Como ( ) ( ) ( )1 1 1

1 1ln ln 1 ln

2 2k k k k k kb b b+ + +< + < + = para todo k ≥ 1 e

( ) ( )1 1 1 1ln ln 1k k k kb b+ + + +≥ = , temos ∑+<≤

>1

2

11

kk bnb na para todo k, donde

11 0

1

k k

nn k b n b n

aa

+

∞ ∞

= = ≤ <

=

∑ ∑ ∑ diverge.

PROBLEMA 6: SOLUÇÃO DA BANCAVamos utilizar coordenadas projetivas (Ref.: "Aplicações de planos projetivos emTeoria dos Números e Combinatória" de Carlos Yuzo Shine - Eureka! No. 15).

Consideremos as duas cônicas do problema inseridas no plano projetivo 2�� . O fato

de serem elipses significa que essas cônicas não cortam a "reta do infinito"z = 0.Lema: Se, A, B, C e D são tais que 3 quaisquer não são colineares, existeum sistema de coordenadas projetivas no qual A = [1, 0, 0], B = [0, 1, 0],C = [0, 0, 1] e D = [1, 1, 1]

Demonstração: Sejam , , ,a b c d� �� �

vetores de 3� representando as classes de

equivalência de A, B, C e D respectivamente. Como A, B e C não são

colineares, , e a b c�� �

são l.i., logo existem reais k, l, m tais que

.d ka lb mc= + +� �� �

Como d não pertence às retas AB, BC ou AC, k, l, m são

diferentes de zero. Assim, se considerarmos a base , ,ka lb mc�� �

de 3� teremos

A = [1, 0, 0], B = [0, 1, 0], C = [0, 0, 1], D = [1, 1, 1], o que prova o lema.

Page 56: Eureka 2003

Sociedade Brasileira de Matemática

EUREKA! N°16, 2003

56

Sejam A, B, C e D os pontos de interseção entre as duas elipses. Usando olema, podemos realizar uma mudança de coordenadas que leve os pontos[–1, 1, 1], [–1, –1, –1], [1, –1, 1] e [1, 1, 1] em [1, 0, 0], [0, 1, 0], [0, 0, 1] e[–1, 1, 1] como toda mudança de coordenadas é invertível, usando o lemapodemos realizar uma mudança de coordenadas que leve os pontos A, B, C eD em [1, 1, 1], [–1, 1, 1], [–1, –1, 1], [1, –1, 1].Podemos escolher esse novo sistema de coordenadas de modo que as duascônicas continuem sendo elipses, em relação à reta do infinito z = 0.De fato a família de cônicas no plano que passam por (–1, –1), (–1, 1), (1, –1)e (1, 1) é dada pelas equações 2 2(1 ) 1tx t y+ − = , t ∈ � ( se {0,1}t ∈ a cônica sedegenera num par de retas). No novo sistema de coordenadas temos duascônicas dessa família. Se uma delas é uma hipérbole, digamos

2 2(1 ) 1tx t y+ − = , com t > 1, podemos aplicar a mudança de coordenadas

projetivas que leva [X, Y, Z] em [Y, Z, X] (e, no plano, leva (x, y) em 1

,y

x x

):

a imagem de Q = {(–1, –1), (–1, 1), (1, –1), (1, 1)} ainda é Q e a imagem da

hiperbole ( 2 2(1 ) 1tx t y+ − = ) é a elipse ( ) 2 21

1 .t x y

t t

−+ =

Assim, temos agora duas cônicas que passam pelos pontos de Q tais que, seuma delas é uma hiperbole então a outra é uma elipse. Assim, ou essascônicas são duas elipses ou qualquer reta no plano intersecta uma dessascônicas. O segundo caso não é possível, pois nesse caso as cônicas nãopoderiam ser imagem de duas elipses por uma mudança de coordenadasprojetivas, dado que a imagem da reta do infinito, que continua sendo umareta, sempre intersecta uma dessas cônicas.

Agora, temos duas elipses que passam pelos pontos de Q. Suponhamos quesuas tangentes no ponto (1, 1) sejam as retas (ax + by = 1) e (cx + dy = 1),com a, b, c, d > 0, a + b = c + d = 1. Após aplicarmos uma mudança decoordenadas afim do tipo T(x, y) = ( / , )x yλ , com 0,λ > obtemosduas outras elipses cujas retas tangentes em T (1, 1) = (1/ ,1)λ são( 1) e ( 1).a x by c x dyλ λ+ = + = As distâncias dessas retas à origem são,

respectivamente, 2 2 2

1

a bλ + e

2 2 2

1.

c dλ +

Page 57: Eureka 2003

Sociedade Brasileira de Matemática

EUREKA! N°16, 2003

57

Temos 2 2

2 2 2 2 2 2 22 2

d ba b c d

a cλ λ λ −+ = + ⇔ =

−, que é positivo, pois

( 0 0).a b c d a c d b+ = + ⇒ − > ⇔ − >

Assim, tomando 2 2

2 2

d b

a cλ −=

−, e aplicando ( , ) ( / , )T x y x yλ= às nossas duas

elipses, obtemos duas elipses que se intersectam em quatro pontos de modoque todas as 8 retas tangentes às duas elipses nesses pontos estão a umamesma distância da origem (por simetria), e logo existe uma círculo tangentea todas elas, o qual está contido na união dos interiores dessas elipses, eportanto não intersecta a imagem da reta do infinito pela mudança decoordenadas projetivas que leva as elipses originais nestas, e logo é imagemde uma elipse por essa mudança de coordenadas. Essa elipse é tangente às 8retas do enunciado. Isso resolve o problema.

Nota: Os enunciados dos problemas 3 e 5 da segunda fase do Nível Universitário

saíram com alguns erros na prova: no problema 3, aparecia 1

n

ii

A=� em vez de

1

k

ii

A=� , e , no problema 5, aparecia

( )

( )0

ln ( ) ln( ) lnln( ) ... ln ( )k n

j k nj

x x x x x=

= ⋅ ⋅ ⋅ ⋅∏

em vez de ( )

( )0

ln ( ) ln( ) lnln( ) ... ln ( ).k n

j k nj

n n n n n=

= ⋅ ⋅ ⋅ ⋅∏

♦♦♦

Agradecemos a Okakamo Kokobongo Matsubashi pela revisão deste número.

♦♦♦

Errata: No artigo "Reciprocidade Quadrática", de Carlos Gustavo Moreira e Nicolau Saldanha(publicado na Eureka! No. 15), onde está "símbolo de Lagrange" deveria ser "símbolo deLegendre".

Page 58: Eureka 2003

Sociedade Brasileira de Matemática

EUREKA! N°16, 2003

58

XXIV OLIMPÍADA BRASILEIRA DE MATEMÁTICAResultado – Nível 1 (5a. e 6a. Séries)

MEDALHA DE OUROHenrique Ponde de Oliveira Pinto Salvador - BACamila Alves Pereira Gloria do Goitá - PECássio Kendi Takamori São José dos Campos - SPJéssica Guerra Caldato Santo André - SPVinícius Marques Regitano Piracicaba - SP

MEDALHA DE PRATAMário Henrique Mendonça Castilho São João da Boa Vista - SPBernardo de Oliveira Veiga Rio de Janeiro - RJRafael Tupynambá Dutra Belo Horizonte - MGGabrielle Collato Marcelino Santo André - SPMaria Fernanda Petri Beto São Paulo - SPGuilherme Philippe Figueiredo Fortaleza - CECristiano Peres Guimarães Mendonça- SPGustavo Henrique dos Santos Figueiredo Santo André - SPLarissa Lais de Sá São Paulo - SPRafael Augusto da Silva Gonçalves Salvador - BA

MEDALHA DE BRONZEDiogo Bonfim Moraes Morant de Holanda Rio de Janeiro - RJFernanda Sá Leal de Moura Teresina - PILuísa Dias Barbosa Alves Recife - PEDavid Francisco dos Santos Serra - ESMarcos Coppa Gomes Filho Natal - RNAnderson Vasconcelos Maciel Fortaleza - CESamuel Carvalho Lima Holanda Fortaleza - CERodolfo de Andrade Marinho Silva Campina Grande - PBFranz Biondi Siemon Vitória - ESRafael Sampaio de Rezende Fortaleza - CEAndré Vasconcelos Barros Natal - RNGuilherme Silva Moura Jequié - BA

MENÇÃO HONROSADaniel Luna de Menezes João Pessoa - PBLaís Moutinho Medeiros Recife - PERafael Moura e Sucupira Fortaleza - CEVinicius de Souza Lima e Oliveira Rio de Janeiro - RJWeslen Costa Timoteo Paulista - PEFilipe Alves Tomé Fortaleza - CELays Cardoso Tatagiba Itaperuna - RJMarlon Vieira de Lima Júnior Fortaleza - CE

Lukas Carmona Macedo de Souza São Paulo - SPAlessandro Wagner Palmeira Guarulhos - SPNathália Pereira Gonçalves Rio de Janeiro - RJRenan Magri Itaporã - PRRafael Alcoforado Domingues João Pessoa - PBLetícia Duarte Ferrari Rio de Janeiro - RJLucio Eiji Assaoka Hossaka Curitiba - PRFlavia Contartesi São Carlos - SPOdair Dutra Santana Júnior Botuporanga - SPMayara Franco Rodrigues Araraquara - SPArtur de Almeida Losnak São Paulo - SPThaísa Giorno Dantas Rabaneda Lopes Atibaia - SPCássio dos Santos Araújo Recife - PEAmanda Yumi Iseri Uberaba - MG

Tiago Madeira Itajaí - SCMatheus Mello Asunção Belém - PALuísa Castro Noronha Valinhos - SPRafael Ellis Reuben São Paulo - SP

Page 59: Eureka 2003

Sociedade Brasileira de Matemática

EUREKA! N°16, 2003

59

Resultado – Nível 2 (7a. e 8a. Séries)

MEDALHA DE OUROThomás Yoiti Sasaki Hoshina Rio de Janeiro - RJAndré Lucas Ribeiro dos Santos Pindamonhangaba - SPVitor Humia Fontoura Salvador - BAGabriel Tavares Bujokas São Paulo - SP

MEDALHA DE PRATAGuilherme Rodrigues Nogueira de Souza São Paulo - SPDouglas Bokliang Ang Cunha São José dos Campos - SPHector Kenzo Horiuti Kitahara São Paulo - SPGuilherme Rohden Echelmeier Itajaí - SC

Enzo Haruo Hiraoka Moriyama São Paulo - SPLuty Rodrigues Ribeiro Fortaleza - CEEduardo Fischer Encantado - RSRafael Kitayama Shiraiwa São Paulo - SPThaís Viveiro São Paulo - SP

MEDALHA DE BRONZECaio dos Santos Pereira Gazzola Belo Horizonte - MGRodrigo Augusto Santana Belém - PARodrigo Viana Soares Fortaleza - CEAndré Linhares Rodrigues Fortaleza - CEFábio Eigi Imada São José dos Campos - SPRafael Montezuma Pinheiro Cabral Fortaleza - CEPedro Paulo Gondim Cardoso Salvador - BARhamon Barroso de Sousa Fortaleza - CELucas Magalhães Pereira Castello Branco Fortaleza - CEMax Douglas Peixoto da Silva Fortaleza - CERenata Mayer Gukovas São Paulo - SPMilena Pinheiro Martins Teresina - PI

Anderson Hoshiko Aiziro São Carlos - SPMENÇÃO HONROSA

Daniel Yoshio Futenma da Silva São Paulo - SPLanderson Bezerra Santiago Maracanaú - CEJosé Armando Barbosa Filho Fortaleza - CEDanilo Eiki Yokoyama São Paulo - SPFernando Mizoguchi Gorgoll São Paulo - SPPedro Thiago Ezequiel de Andrade Fortaleza - CEJosé Robério Xavier dos Santos Júnior Fortaleza - CE

Erick Vizolli Curitiba - PRCamila Vasconcelos de Oliveira Fortaleza - CERaphael Rodrigues Mata Salvador - BAAdriano César Braga Borges Contagem - MGGustavo Eidji Camarinha Fujiwara São Paulo - SPHenrique Kenji Formagio Noguchi São Paulo - SPAndré Ikeda Cantão Curitiba - PRPaulo André Carvalho de Melo Rio de Janeiro - RJFábio Queiroz Vasconcelos Cunha Salvador - BA

Flaviano Ramos Pereira Junior Belém - PAMauro Cardoso Lopes São Paulo - SPLuiz Müller Vitória - ESTiago Nery Vasconcelos São Paulo - SPThiago de Azevedo Pinheiro Hoshino São Paulo - SP

Page 60: Eureka 2003

Sociedade Brasileira de Matemática

EUREKA! N°16, 2003

60

Resultado – Nível 3 (Ensino Médio)

MEDALHA DE OURO

Guilherme Issao Camarinha Fujiwara São Paulo - SPFábio Dias Moreira Rio de Janeiro - RJRafael Daigo Hirama Campinas - SP

MEDALHA DE PRATA

Yuri Gomes Lima Fortaleza - CEThiago da Silva Sobral Fortaleza - CEAlex Corrêa Abreu Niterói - RJHenrique Chociay Curitiba - PRAntonio Carlos Maldonado Silveira A. Munhoz Rio de Janeiro - RJHenry Wei Cheng Hsu São Paulo - SPSamuel Barbosa Feitosa Fortaleza - CELarissa Cavalcante Queiroz de Lima Fortaleza - CEBernardo Freitas Paulo da Costa Rio de Janeiro - RJDavi Máximo Alexandrino Nogueira Fortaleza - CEThiago Costa Leite Santos São Paulo - SP

MEDALHA DE BRONZE

Einstein do Nascimento Júnior Fortaleza - CEEduardo de Moraes Rodrigues Poço São Paulo - SPRafael Tajra Fonteles Teresina - PIFelipe Rodrigues Nogueira de Souza São Paulo - SPMurilo Vasconcelos Andrade Maceió - ALThiago Braga Cavalcante Fortaleza - CEPaulo Ribeiro de Almeida Neto Ananindeua - PAGermanna de Oliveira Queiroz Fortaleza - CEJuliana Gomes Varela Fortaleza - CERodrigo Aguiar Pinheiro Fortaleza - CEIsrael Franklim Dourado Carrah Fortaleza - CEDaniel Pessoa Martins Cunha Fortaleza - CERenato Seiji Tavares São Paulo - SPCarlos Augusto David Ribeiro Fortaleza - CELetícia Rosa dos Santos Rio de Janeiro - RJ

MENÇÃO HONROSA

Rafael Marini Silva Vila Velha - ESTelmo Luis Correa Junior São Paulo - SPDiego Alvarez Araujo Correia Fortaleza - CEVitor Gabriel Kleine Mogi das Cruzes - SPFrancisco Bruno de Lima Holanda Fortaleza - CEDiogo dos Santos Suyama Belo Horizonte - MGAnderson Torres São Paulo - SPLarissa Rodrigues Ribeiro Fortaleza - CEMarina Lima Medeiros Fortaleza - CE

Antonia Taline de Souza Mendonça Fortaleza - CERodrigo Angelo Muniz Cariacica - ESEduardo Paiva Costa Teresina - PIEduardo Monteiro Nicodemos Rio de Janeiro - RJThiago Morello Peres Rio de Janeiro - RJElder Rodrigo Barbosa Campos Rio de Janeiro - RJThiago Luís Viana de Santana Rio de Janeiro - RJFilipe Rodrigues de Souza Moreira Rio de Janeiro - RJRodrigo Kendy Yamashita São Paulo - SPJoão Marcos da Cunha Silva Fortaleza - CELyussei Abe São Paulo - SP

Page 61: Eureka 2003

Sociedade Brasileira de Matemática

EUREKA! N°16, 2003

61

Resultado – Nível Universitário

MEDALHA DE OURO

Carlos Yuzo Shine São Paulo - SPHumberto Silva Naves São José dos Campos - SPMarcio Afonso Assad Cohen Rio de Janeiro - RJ

MEDALHA DE PRATA

Thiago Barros Rodrigues Costa Fortaleza - CECarlos Stein Naves de Brito São José dos Campos - SPRodrigo Villard Milet Rio de Janeiro - RJDaniel Massaki Yamamoto São Paulo - SPGiuliano Boava Florianópolis - SCFabrício Siqueira Benevides Fortaleza - CEEduardo Famini Silva Rio de Janeiro - RJ

MEDALHA DE BRONZE

Tertuliano Franco Santos Franco Salvador - BARodrigo Roque Dias São Paulo - SPLucas de Melo Pontes e Silva São Paulo - SPThiago Afonso de André São Paulo - SPSergio Alvarez Araujo Correia Fortaleza - CEDaniel Nobuo Uno São Paulo - SPEvandro Makiyama de Melo São Paulo - SPLeonardo Augusto Zão Nilópolis - RJBruno Fernandes Cerqueira Leite São Paulo - SP

Daniel Mourão Martins Rio de Janeiro - RJDaniele Véras de Andrade Rio de Janeiro - RJLucas Heitzmann Gabrielli São Paulo - SPDiogo Diniz P.S. Silva Campina Grande - PB

Diêgo Veloso Uchôa Teresina - PIMarcelo Handro Maia São José dos Campos - SP

MENÇÃO HONROSA

Gilberto Kirk Rodrigues Rio de Janeiro - RJDiogo Luiz Duarte Rio de Janeiro - RJCamilo Marcantonio Junior Rio de Janeiro - RJMarcio Miranda de Carvalho Teresina - PIMarcio Paiva Reis Vitória - ESArnaldo João do Nascimento Junior Duque de Caxias - RJ

���� � ����� � ���� ����� ����� ��� � ����

���� � � � ��� ���

�������������������������

Page 62: Eureka 2003

Sociedade Brasileira de Matemática

EUREKA! N°16, 2003

62

AGENDA OLÍMPICA

XXV OLIMPÍADA BRASILEIRA DE MATEMÁTICA

NÍVEIS 1, 2 e 3Primeira Fase – Sábado, 7 de junho de 2003

Segunda Fase – Sábado, 13 de setembro de 2003Terceira Fase – Sábado, 18 de outubro de 2003 (níveis 1, 2 e 3)

Domingo, 19 de outubro de 2003 (níveis 2 e 3 - segundo dia de prova).

NÍVEL UNIVERSITÁRIOPrimeira Fase – Sábado, 13 de setembro de 2003

Segunda Fase – Sábado, 18 e Domingo, 19 de outubro de 2003

IX OLIMPÍADA DE MAIO10 de maio de 2003

XIV OLIMPÍADA DE MATEMÁTICA DO CONE SUL23 a 30 de maio de 2003

Ica – Peru

♦XLIV OLIMPÍADA INTERNACIONAL DE MATEMÁTICA

07 a 19 de julho de 2003Tóquio – Japão

X OLIMPÍADA INTERNACIONAL DE MATEMÁTICA UNIVERSITÁRIA25 a 31 de julho de 2003

Universidade Babes-Bolyai, Cluj-Napoca, Romênia

♦XVIII OLIMPÍADA IBEROAMERICANA DE MATEMÁTICA

13 a 20 de setembro de 2003Argentina

♦VI OLIMPÍADA IBEROAMERICANA DE MATEMÁTICA UNIVERSITÁRIA

8 de novembro de 2003

♦♦♦

Page 63: Eureka 2003

CONTEÚDO

AOS LEITORES 2

IX OLIMPÍADA DE MAIO 3Enunciados e Resultado Brasileiro

XIV OLIMPÍADA DE MATEMÁTICA DO CONE SUL 6Enunciados e Resultado Brasileiro

XLIV OLIMPÍADA INTERNACIONAL DE MATEMÁTICA 9Enunciados e Resultado Brasileiro

X OLIMPÍADA INTERNACIONAL DE MATEMÁTICA PARA ESTUDANTES UNIVERSITÁRIOS 11Enunciados e Resultado Brasileiro

XVIII OLIMPÍADA IBERO-AMERICANA DE MATEMÁTICA 14Enunciados e Resultado Brasileiro

GEOMETRIA COM CONTAS 17Carlos Yuzo Shine

A ENUMERABILIDADE DE � x � E O CHÃO TRIANGULAR 36José Paulo Carneiro

COMO É QUE FAZ? 41

SOLUÇÕES DE PROBLEMAS PROPOSTOS 45

PROBLEMAS PROPOSTOS 59

AGENDA OLÍMPICA 61

COORDENADORES REGIONAIS 62

Page 64: Eureka 2003

Sociedade Brasileira de Matemática

EUREKA! N°17, 2003

2

AOS LEITORES

Caros leitores,

Neste número a Eureka! publica várias provas de 2003 de competiçõesinternacionais nas quais o Brasil participa: Olimpíada de Maio, Olimpíada doCone Sul, Olimpíada Internacional, Olimpíada Ibero-americana e OlimpíadaInternacional Uhniversitária (IMC). Este foi o primeiro ano em que uma delegaçãobrasileira participa desta última competição, resultado de uma parceria da OBMcom as Universidades em que estudam os membros da equipe brasileira.Começamos bem: obtivemos três segundos prêmios, três terceiros prêmios e duasmenções honrosas.Neste número excepcionalmente não publicamos a seção "Olimpíadas ao redor domundo", que volta no próximo. Por outro lado, criamos a seção "Como é quefaz?", onde vamos resolver problemas sugeridos pelos leitores.A idéia é que os leitores enviem problemas que não conseguem resolver, quetenham aparecido em alguma Eureka! (por exemplo em algum artigo ou provapublicada) ou alguma competição (ou de qualquer outra origem), que tentaremosresolver e publicar os mais interessantes na nossa seção.

Os editores

Page 65: Eureka 2003

Sociedade Brasileira de Matemática

EUREKA! N°17, 2003

3

IX OLIMPÍADA DE MAIOEnunciados e Resultado Brasileiro

PRIMEIRO NÍVELDuração da prova: 3 horas

PROBLEMA 1Pedro escreve todos os números de quatro algarismos diferentes que podem serarmados com dígitos a, b, c, d que cumprem as seguintes condições:

a ≠ 0 ; b = a + 2; c = b + 2 ; d = c + 2.

Calcule a soma de todos os números que Pedro escreveu.

PROBLEMA 2O triângulo ABC é retângulo em A e R é o ponto médio da hipotenusa BC. Sobreo cateto maior AB se marca o ponto P tal que CP = BP e sobre o segmento BP semarca o ponto Q tal que o triângulo PQR é equilátero. Se a área do triângulo ABCé 27, calcule a área do triângulo PQR.

PROBLEMA 3Determine o menor número inteiro positivo que termina em 56, é múltiplo de 56 etem a soma de seus dígitos igual a 56.

PROBLEMA 4Célia escolhe um número n e escreve a lista dos números naturais de 1 até n:

1, 2, 3, 4, …, n – 1, n.

Em cada passo, troca a lista: copia o primeiro número ao final e apaga os doisprimeiros.Depois de n – 1 passos ficará escrito um único número.Por exemplo, para n = 6 os cinco passos são:

1, 2, 3, 4, 5, 6 → 3, 4, 5, 6, 1 → 5, 6, 1, 3 → 1, 3, 5 → 5, 1 → 5

e ficará escrito o número 5.

Célia escolheu um número n entre 1000 e 3000 e depois de n – 1 passos ficou onúmero 1.

Page 66: Eureka 2003

Sociedade Brasileira de Matemática

EUREKA! N°17, 2003

4

Determine todos os valores de n que Célia pode ter escolhido.Justifique porque estes valores servem e os demais não.

PROBLEMA 5Temos um tabuleiro quadriculado 4 × 4. Definimos a separação entre duas casascomo o menor número de movimentos que deve empregar um cavalo de xadrezpara ir de uma casa a outra (utilizando movimentos do cavalo). Três casas A, B, Cformam um trio bom se as três separações entre A e B, entre A e C e entre B e Csão iguais. Determine um número de trios bons que se formam no tabuleiro.

OBSERVAÇÃO:Em cada movimento o cavalo se desloca 2 casas em direção horizontal mais umacasa em direção vertical ou se desloca 2 casas em direção vertical mais uma casaem direção horizontal.

SEGUNDO NÍVELDuração da prova: 3 horas

PROBLEMA 1São escolhidos quatro dígitos a, b, c, d diferentes entre si e diferentes de zero e seescreve a lista de todos os números de quatro algarismos que se obtém trocando delugar os dígitos a, b, c, d.Que dígitos deve-se escolher para que a lista tenha a maior quantidade possível denúmeros de quatro algarismos que sejam múltiplos de 36?

PROBLEMA 2Seja ABCD um retângulo de lados AB = 4 e BC =3. A perpendicular à diagonalBD traçada por A corta BD no ponto H. Chamamos de M o ponto médio de BH ede N o ponto médio de CD. Calcule a medida do segmento MN.

PROBLEMA 3Encontre todos os pares de números inteiros positivos (a, b) tais que 8b + 1 émúltiplo de a e 8a + 1 é múltiplo de b.

Page 67: Eureka 2003

Sociedade Brasileira de Matemática

EUREKA! N°17, 2003

5

PROBLEMA 4Beto marcou 2003 pontos verdes no plano, de maneira que todos os triânguloscom seus três vértices verdes têm área menor que 1.Demonstre que os 2003 pontos verdes estão contidos num triângulo T de áreamenor que 4.

PROBLEMA 5Uma formiga, que está numa aresta de um cubo de lado 8, deve realizar umpercurso pela superfície do cubo e regressar ao ponto de partida. Seu caminhodeve conter pontos interiores das seis faces do cubo e deve visitar só uma vez cadaface do cubo. Encontre o comprimento do caminho mais curto que a formiga poderealizar e justifique porque é o caminho mais curto.

RESULTADOS

PRIMEIRO NÍVEL

Lucio Eiji Assaoka Hossaka Medalha de Ouro Curitiba - PRMarlon Vieira de Lima Junior Medalha de Prata Fortaleza - CEHenrique Pondé de Oliveira Pinto Medalha de Prata Salvador - BAGuilherme Philippe Figueiredo Medalha de Bronze Fortaleza - CEFernanda Sá Leal de Moura Medalha de Bronze Teresina - PIRégis Prado Barbosa Medalha de Bronze Fortaleza - CETiago Madeira Medalha de Bronze Itajaí - SCMateus Faitanin Yin Menção Honrosa Vitória - ESDiogo Bonfim Moraes Morant de Holanda Menção Honrosa Rio de Janeiro - RJRonaldo Rozenbaum Paiva Menção Honrosa Rio de Janeiro - RJ

SEGUNDO NÍVEL

Telmo Luis Correa Junior Medalha de Ouro São Paulo - SPLuty Rodrigues Ribeiro Medalha de Prata Fortaleza - CEPedro Thiago Ezequiel de Andrade Medalha de Prata Fortaleza - CEGabriel Tavares Bujokas Medalha de Bronze São Paulo - SPJuliana G. Cavalcante Medalha de Bronze Fortaleza - CELeandro Farias Lima Medalha de Bronze Fortaleza - CEEduardo Fischer Medalha de Bronze Encantado - RSAndré Linhares Rodrigues Menção Honrosa Fortaleza - CERodrigo Viana Soares Menção Honrosa Fortaleza - CERaphael Rodrigues Viana Menção Honrosa Salvador - BA

Page 68: Eureka 2003

Sociedade Brasileira de Matemática

EUREKA! N°17, 2003

6

XIV OLIMPÍADA DE MATEMÁTICA DO CONE SULEnunciados e Resultado Brasileiro

A XIV Olimpíada de Matemática do Cone Sul foi realizada na cidade deIca, Peru, no período de 23 a 30 de maio de 2003. A equipe brasileira foi lideradapelos professores Paulo José Bonfim Gomes Rodrigues e Emanuel de SouzaCarneiro, ambos de Fortaleza – CE. Novamente a equipe brasileira obteve a maiorpontuação entre os países participantes.

RESULTADOS DA EQUIPE BRASILEIRA

BRA1 Fábio Dias Moreira PrataBRA2 Henry Wei Cheng Hsu PrataBRA3 Thiago Costa Leite Santos PrataBRA4 Rodrigo Aguiar Pinheiro Ouro

PROBLEMA 1Em um torneio de futebol entre quatro equipes, A, B, C e D, cada equipe joga comcada uma das outras exatamente uma vez.

a) Decidir se é possível que, ao finalizar o torneio, as quantidades de golsmarcados e sofridos pelas equipes sejam:

A B C DGols marcados 1 3 6 7Gols sofridos 4 4 4 5

Se a resposta é afirmativa, dê um exemplo com os resultados das seis partidas; emcaso contrário, justifique.

b) Decidir se é possível que, ao finalizar o torneio, as quantidades de golsmarcados e sofridos pelas equipes sejam:

A B C DGols marcados 1 3 6 13Gols sofridos 4 4 4 11

Page 69: Eureka 2003

Sociedade Brasileira de Matemática

EUREKA! N°17, 2003

7

Se a resposta é afirmativa, dê um exemplo com os resultados das seis partidas; emcaso contrário, justifique.

PROBLEMA 2Considere a seqüência {an} definida da seguinte maneira:

a1 = 1 a2 = 3 an+2 = 2an+1 an

+ 1, para todo inteiro n ≥ 1.

Provar que a máxima potência de 2 que divide a4006 − a4005 é 22003.

PROBLEMA 3Seja ABC um triângulo acutângulo tal que o ângulo B mede 60o . A circunferênciade diâmetro AC intersecta as bissetrizes internas de A e C nos pontos M e Nrespectivamente (M ≠ A, N ≠ C). A bissetriz interna do ângulo B intersecta MN eAC nos pontos R e S, respectivamente. Demonstrar que BR ≤ RS.

PROBLEMA 4No triângulo acutângulo ABC, os pontos H, G e M encontram-se sobre o lado BC,de modo que AH, AG e AM são altura, bissetriz e mediana do triângulo,respectivamente. Sabe-se que HG = GM, AB = 10 e AC = 14. Determinar a área dotriângulo ABC.

PROBLEMA 5Seja 13 += kn , onde k é um inteiro, 1≥k . Constrói-se um arranjo triangularde lado n formado por círculos de mesmo raio como o mostrado na figura para

7=n .

Page 70: Eureka 2003

Sociedade Brasileira de Matemática

EUREKA! N°17, 2003

8

Determinar, para cada k, o maior número de círculos que podem ser coloridos devermelho de tal modo que não existam dois círculos vermelhos tangentes entre si.

PROBLEMA 6

Demonstrar que existe uma seqüência de inteiros positivos ,...x,...,x,x n21 que

satisfaz as duas condições seguintes:i) contém exatamente uma vez cada um dos inteiros positivos,

ii) para cada ,...2,1=n a soma parcial nxxx +++ ...21 é divisível por nn .

Page 71: Eureka 2003

Sociedade Brasileira de Matemática

EUREKA! N°17, 2003

9

XLIV OLIMPÍADA INTERNACIONAL DE MATEMÁTICA07 a 19 de julho, Tóquio - Japão

A XLIV Olimpíada Internacional de Matemática foi realizada em Tóquio, Japão,no período de 07 a 19 de julho de 2003. A equipe brasileira foi liderada pelosprofessores Nicolau Saldanha, do Rio de Janeiro – RJ e Élio Mega, de São Paulo –SP.

RESULTADOS DA EQUIPE BRASILEIRA

BRA1 Alex Corrêa Abreu Medalha de BronzeBRA2 Samuel Barbosa Feitosa Medalha de BronzeBRA3 Rafael Daigo Hirama Menção HonrosaBRA4 Larissa Cavalcante Queiroz de Lima Menção HonrosaBRA5 Fábio Dias Moreira Medalha de PrataBRA6 Davi Máximo Alexandrino Nogueira Medalha de Bronze

PROBLEMA 1Seja A um subconjunto do conjunto }1000000,,2,1{ �=S com exatamente 101

elementos. Demonstre que existem números 10021 ,,, ttt � em S tais que osconjuntos

}|{ AxtxA jj ∈+= , para 1, 2, ,100,j = �

são disjuntos dois a dois.

PROBLEMA 2Determine todos os pares de inteiros positivos ),( ba tais que

12 32

2

+− bab

a

é um inteiro positivo.

PROBLEMA 3Considere um hexágono convexo tal que para cada quaisquer dois lados opostosverifica-se a seguinte propriedade: a distância entre os seus pontos médios é igual

a 23 vezes a soma dos seus comprimentos. Demonstre que todos os ângulos do

hexágono são iguais.

Page 72: Eureka 2003

Sociedade Brasileira de Matemática

EUREKA! N°17, 2003

10

(Um hexágono convexo ABCDEF tem três pares de lados opostos: AB e DE ,BC e EF , CD e FA ).

PROBLEMA 4Seja ABCD um quadrilátero convexo cujos vértices estão sobre umacircunferência. Sejam P , Q e R os pés das perpendiculares às retas BC , CA eAB , respectivamente, passando por D . Demonstre que QRPQ = se e só se as

bissetrizes dos ângulos ABC∠ e ADC∠ se intersectam sobre a reta AC .

PROBLEMA 5

Sejam n um inteiro positivo e nxxx ,,, 21 � números reais tais que

nxxx ≤≤≤ �21 .

(a) Demonstre que

2 22

1 1 1 1

2( 1)( ) .

3

n n n n

i j i ji j i j

nx x x x

= = = =

−− ≤ − ∑∑ ∑∑

(b) Demonstre que a igualdade é válida se e só se nxxx ,,, 21 � formam umaprogressão aritmética.

PROBLEMA 6Seja p um número primo. Demonstre que existe um número primo q tal que,

para todo inteiro n, o número pn p − não é divisível por q.

Page 73: Eureka 2003

Sociedade Brasileira de Matemática

EUREKA! N°17, 2003

11

X OLIMPÍADA INTERNACIONAL DE MATEMÁTICA PARAESTUDANTES UNIVERSITÁRIOS

25 a 31 de Julho, Cluj - Napoca, Romênia

A X Olimpíada Internacional de Matemática para estudantesuniversitários foi realizada na cidade de Cluj-Napoca, Romênia, no período de 25a 31 de Julho de 2003. A equipe brasileira foi liderada pelo professor LucianoCastro, do Rio de Janeiro – RJ.

RESULTADOS DA EQUIPE BRASILEIRA

Márcio Afonso Assad Cohen IME Medalha de PrataHumberto Silva Naves ITA Medalha de PrataRodrigo Villard Milet UFRJ Medalha de PrataCarlos Stein Naves de Brito ITA Medalha de BronzeDaniel Yamamoto ITA Medalha de BronzeGiuliano Boava UFSC Medalha de BronzeEduardo Famini Silva IME Menção HonrosaThiago Barros Rodrigues Costa UNICAMP Menção Honrosa

PRIMEIRO DIA

PROBLEMA 1

a) Seja 1 2, , , ,...na a a� uma seqüência de números reais tais que 1 1a = e

1

3, .

2n na a n+ > ∀

Prove que a seqüência 1

32

nn

a−

tem um limite finito ou tende a infinito.

b) Prove que para todo 1α > existe uma seqüência 1 2, , , ,...na a a� com as

mesmas propriedades, tal que 1lim .32

nnn

a α−→∞=

Page 74: Eureka 2003

Sociedade Brasileira de Matemática

EUREKA! N°17, 2003

12

PROBLEMA 2

Sejam 1 2 51, , ,a a a� elementos não nulos de um corpo. Simultaneamentetrocamos cada elemento pela soma dos outros 50. Desta forma a nova seqüência

1 2 51, , ,b b b� é uma permutação da anterior. Quais são os possíveis valores dacaracterística do corpo?

PROBLEMA 3Seja A uma matriz quadrada n × n tal que 3A3 = A2 + A + I. Prove que (Ak )k ∈ �

converge a uma matriz idempotente B (i.e., a uma matriz B tal que B2 = B).

PROBLEMA 4Determine o conjunto de todos os pares (a, b) de inteiros positivos para os quais oconjunto dos inteiros positivos pode ser decomposto em dois conjuntos A e B taisque .a A b B⋅ = ⋅

PROBLEMA 5

Sejam :[0,1]g → � uma função contínua e : (0,1]nf →� a seqüência de

funções definida por 0 ( ) ( )f x g x= e 1 0

1( ) ( ) , (0,1], 0.

x

n nf x f t dt x nx+ = ∀ ∈ ≥∫

Determine lim ( )nnf x

→∞para todo (0,1]x ∈ .

PROBLEMA 6

Seja 1

1 1 0( ) ...n nn nf z a z a z a z a−

−= + + + + um polinômio com coeficientesreais. Prove que se as raízes de f estão no semi-plano esquerdo

{ | Re( ) 0}z z∈ <� então 3 1 2k k k ka a a a+ + +< para todo k = 0, 1,…, n – 3.

Page 75: Eureka 2003

Sociedade Brasileira de Matemática

EUREKA! N°17, 2003

13

SEGUNDO DIA

PROBLEMA 1Sejam A e B matrizes reais n × n tais que AB + A + B = 0. Prove que AB = BA.

PROBLEMA 2

Calcule o seguinte limite: 2

0lim

mx

nxx

sen tdt

t+→ ∫ (m, n naturais dados).

PROBLEMA 3Seja A um subconjunto fechado de �n e seja B o conjunto de todos os pontos b de

�n tais que existe exatamente um ponto a0 em A tal que 0 inf

a Aa b a b

∈− = − .

Prove que B é denso em �n.

PROBLEMA 4Encontre todos os inteiros positivos n para os quais existe uma família F desubconjuntos de três elementos de S ={1, 2, …,n} que satisfaz as seguintescondições:(i) para quaisquer elementos distintos a, b ∈ S existe exatamente um A ∈ F

tal que a, b ∈ A.(ii) Se a, b, c, x, y, z são tais que {a, b, x}, {a, c, y}, {b, c, z} ∈ F então {x, y, z} ∈ F.

PROBLEMA 5a) Mostre que para toda função :f × →� � � existe uma função :g →� � tal

que ( , ) ( ) ( ), ,f x y g x g y x y≤ + ∀ ∈� .

b) Encontre uma função :f × →� � � para a qual não existe :g →� � tal

que f(x, y) ≤ g(x) + g(y), ∀x, y ∈ �.

PROBLEMA 6

Seja 0 1, ,..., ,...na a a a seqüência definida por 0 1a = , 10

1.

1 2

nk

nk

aa

n n k+=

=+ − +∑

Calcule 0 2

kk

k

a∞

=∑ (se existir).

Page 76: Eureka 2003

Sociedade Brasileira de Matemática

EUREKA! N°17, 2003

14

XVIII OLIMPÍADA IBEROAMERICANA DE MATEMÁTICA13 a 20 de setembro, Mar del Plata - Argentina

A XVIII Olimpíada Iberoamericana de Matemática foi realizada na cidade de Mardel Plata, Argentina, no período de 13 a 20 de setembro de 2003. A equipebrasileira foi liderada pelos professores Augusto C. de Oliveira Morgado, do Riode Janeiro – RJ e Luzinalva Miranda de Amorim, de Salvador – BA.

RESULTADOS DA EQUIPE BRASILEIRA

BRA1 Alex Corrêa Abreu Medalha de OuroBRA2 Davi Máximo Alexandrino Nogueira Medalha de PrataBRA3 Fábio Dias Moreira Medalha de OuroBRA4 Samuel Barbosa Feitosa Medalha de Bronze

PRIMEIRO DIA

PROBLEMA 1

a) Têm-se duas sucessões, cada uma de 2003 inteiros consecutivos, e umtabuleiro de 2 linhas e 2003 colunas

………………

Decida se é sempre possível distribuir os números da primeira sucessão naprimeira linha e os da segunda sucessão na segunda linha, de modo que osresultados obtidos ao somar os dois números de cada coluna formem uma novasucessão de 2003 números consecutivos.

b) E se trocássemos 2003 por 2004?Tanto em a) como em b), se a resposta for afirmativa, explique como distribuiriaos números, e se for negativa, justifique o porquê.

Page 77: Eureka 2003

Sociedade Brasileira de Matemática

EUREKA! N°17, 2003

15

PROBLEMA 2Sejam C e D dois pontos da semicircunferência de diâmetro AB tais que B e Cestão em semiplanos distintos em relação à reta AD. Denotemos por M, N e P ospontos médios de AC, DB e CD, respectivamente. Sejam OA e OB os circuncentrosdos triângulos ACP e BDP. Demonstre que as retas OA OB e MN são paralelas.

PROBLEMA 3Pablo copia o seguinte problema:

Considere todas as sucessões de 2004 números reais 0 1 2 2003( , , ,..., ),x x x xtais que

0 1,x =

1 00 2 ,x x≤ ≤

2 10 2 ,x x≤ ≤�

2003 20020 2 .x x≤ ≤Entre todas estas sucessões, determine aquela para a qual a expressão seguinteassume o seu maior valor: S = … .Quando Pablo ia copiar a expressão S, apagaram o quadro. Só conseguia lembrar-se de que S era da forma

1 2 2002 2003... ,S x x x x= ± ± ± ± +onde o último termo, 2003x , tinha coeficiente +1, e os anteriores tinham coeficiente+1 ou –1. Demonstre que Pablo, apesar de não ter o enunciado completo, podedeterminar com certeza a solução do problema.

SEGUNDO DIA

PROBLEMA 4Seja M ={1, 2,…,49} o conjunto dos primeiros 49 inteiros positivos. Determine omaior inteiro k tal que o conjunto M tenha um subconjunto de k elementos em quenão haja 6 números consecutivos. Para esse valor máximo de k, encontre aquantidade de subconjuntos de m, de k elementos, que tenham a propriedademencionada.

Page 78: Eureka 2003

Sociedade Brasileira de Matemática

EUREKA! N°17, 2003

16

PROBLEMA 5No quadrado ABCD, sejam P e Q pontos pertencentes aos lados BC e CDrespectivemante, distintos dos extremos, tais que BP = CQ. Consideram-se pontosX e Y, X ≠ Y, pertencentes aos segmentos AP e AQ respectivamente. Demonstreque, quaisquer que sejam X e Y, existe um triângulo cujos lados têm oscomprimentos dos segmentos BX, XY e DY.

PROBLEMA 6

Definen-se as sucessões 0 0( ) ,( )n n n na b≥ ≥ por:

0 01, 4a b= = e2001

1 + ,n n na a b+ = 2001

1 + an n nb b+ = para 0.n ≥

Demonstre que 2003 não divide nenhum dos termos destas sucessões.

Page 79: Eureka 2003

Sociedade Brasileira de Matemática

EUREKA! N°17, 2003

17

GEOMETRIA COM CONTASCarlos Yuzo Shine, Colégio Etapa

♦ Nível Avançado

Às vezes precisamos de mais elementos para resolver problemas degeometria. Pode-se traçar novos elementos na figura que possam ajudar ou fazeralgumas contas. Mostraremos algumas técnicas para fazer algumas contas queajudam (e até resolvem!).

Em geral, pode-se pensar em problemas de geometria seguindo esses passos:

(i) Faça a figura do problema (praticamente nenhum problema vem comfigura), bem grande e com certa precisão (ou seja, use a régua e ocompasso, mas não é necessário muito rigor).

(ii) Mexa um pouco com os elementos da figura. Algo que é sempre útil éfixar um certo número de ângulos (de preferência, o menor númeropossível, de modo que os ângulos marcados determinem a figura - a nãoser, é claro, que acrescentar algum outro ângulo adicione alguma simetriaalgébrica útil) e calcular todos os outros ângulos possíveis (se os ângulosque você escolheu determinam a figura, é possível calcular todos osoutros, de um jeito ou de outro). Procure quadriláteros inscritíveis paraajudar. Se necessário, faça conjecturas (é para isso que você fez umdesenho bem feito!). Alguns problemas de geometria já são resolvidosnesse passo!

(iii) Se o problema ainda não foi resolvido, é hora de elaborar uma estratégiapara resolver o problema, ou seja, determinar quais cálculos devem serfeitos. Nada de fazer cálculos sem planejá-los!

(iv) Execute sua estratégia. Lembre-se sempre de ter uma meta em mente(algo do tipo "precisamos calcular tal ângulo") e, se você estiver numaprova, de controlar seu tempo e o tamanho da conta (não deixe a contacrescer muito; a falta de controle é um fermento muito poderoso paracontas.)

É claro que esses passos não são precisos e que, para dominá-los, é preciso muitotreino e, por que não, aprender algumas técnicas.

Page 80: Eureka 2003

Sociedade Brasileira de Matemática

EUREKA! N°17, 2003

18

TRIGONOMETRIAMuitos problemas de geometria podem ser resolvidos com o auxílio datrigonometria. As fórmulas que você deve saber são basicamente essas quatro:

sen( ) sen cos sen cosa b a b b a+ = +sen( ) sen cos sen cosa b a b b a− = −cos( ) cos cos sen sena b a b a a+ = −cos( ) cos cos sen sena b a b a b− = +

A partir dessas você pode deduzir essas outras, que na verdade são as mais úteispara nós e que tornam a trigonometria tão poderosa.

Transformando produtos em somas

1sen sen (cos( ) cos( ))

21

cos cos (cos( ) cos( ))21

sen cos (sen( ) sen( ))2

a b a b a b

a b a b a b

a b a b a b

= − − +

= − + +

= − + +

Transformando somas em produtos

sen sen 2sen cos2 2

sen sen 2sen cos2 2

cos cos 2cos cos2 2

cos cos 2sen sen2 2

x y x yx y

x y x yx y

x y x yx y

x y x yx y

+ − + = − + − =

+ − + = + − − = −

Por fim, relembramos a lei dos senos e a lei dos co-senos. No triângulo ABC, sejaAB = c, AC = b, BC = a, ∠A = α, ∠ B = β e ∠C = γ. O circunraio de ABC é R.

2 2 2

2 2 2

2 2 2

2sen sen sen

2 cos

2 cos

2 cos

a b cR

a b c bc

b a c ac

c a b ab

α β γαβγ

= = =

= + −

= + −

= + −

A lei dos senos, por envolver proporções (que são mais simples) e elementosadicionais do triângulo (o circunraio), é particularmente útil.Vamos resolver alguns problemas e mostrar algumas técnicas de cálculo.

Page 81: Eureka 2003

Sociedade Brasileira de Matemática

EUREKA! N°17, 2003

19

CONVENÇÃOSempre que houver um triângulo ABC, α, β e γ são as medidas dos ângulos∠BAC, ∠ABC e ∠ACB, respectivamente.

UM COMEÇO E O TRUQUE DA CO-TANGENTE

Exemplo(Prova de Seleção para a IMO) Seja Γ uma circunferência de centro O tangenteaos lados AB e AC do triângulo ABC nos pontos E e F. A reta perpendicular aolado BC por O intercepta EF no ponto D. Mostre que A, D e M (ponto médio deBC) são colineares.

ResoluçãoPrimeiro, um bom desenho, com todos os ângulos que pudermos marcar (a técnicado arrastão é bastante útil - é por isso que você deve fazer um desenho grande!!).Note que os ângulos do triângulo ABC já determinam os ângulos toda a figura(para perceber isso, note que se construir ABC todos os outros ângulos da figura jáestão determinados).

A

B C

E

F D

O

M

β

γ

Γ

γ

βα2

α2

π2

α2

π2

α2

P

Page 82: Eureka 2003

Sociedade Brasileira de Matemática

EUREKA! N°17, 2003

20

É sempre bom justificar os cálculos. Seja P a interseção de BC e da retaperpendicular a BC por O. Como BEO∠ e BPO∠ são retos, o quadriláteroBPOE é inscritível, de modo que .DOE EBM β∠ = ∠ = Analogamente,

.DOF γ∠ =A reta AO é bissetriz de  e AOEF é inscritível, logo / 2.OEF OFE α∠ = ∠ =Mas, como provar que A, D e M estão alinhados? Uma maneira é provar que

BAD BAM∠ = ∠ , por exemplo. Para isso, é só calcular os dois ângulos.Como calcularemos ?BADφ = ∠ Veja o triângulo ADE. Sendo r o raio de Γ, comuma lei dos senos calculamos DE. AE pode ser facilmente calculado. Como jáconhecemos AED∠ (viu como é bom fazer o arrastão?), temos elementossuficientes para calcular φ.Para calcular ,BAMθ = ∠ usaremos o triângulo BAM, da qual conhecemos BM,

AB, e .ABM∠Já temos uma estratégia. Vamos executar o plano!

O

D E

A

β r

π α 2 2

α 2

φ

No triângulo ODE,sen

sensen sen

2 2

DE r rDE

βα αβ β β

= ⇔ = + +

(note que ( / 2)ODE π β α∠ = − + - utilizamos o fato de que

sen sen( )x xπ= − para todo x real; utilizaremos bastante esse fato e o fato

sen( / 2 ) cos )x xπ − =Sendo o triângulo AEO retângulo em E, obtemos cotg( / 2).AE r α=

Page 83: Eureka 2003

Sociedade Brasileira de Matemática

EUREKA! N°17, 2003

21

No triângulo ADE,

sencos

2

DE AEαφ φ

= +

(*)

Quando temos uma equação do tipo

( ) ,sen sen

a b

x x δ=

+e queremos determinar x, utilizamos o truque da co-tangente:

sen( ) sen cos sen cos

sen sen( ) sen sen

cos sen cotg

a b x b x x

x x x a x

b bx

a a

δ δ δδ

δ δ

+ += ⇔ = ⇔ =+

= ⇔ + =

e podemos isolar cotg x.Voltemos a (*). Substituindo DE e AE e utilizando o truque da co-tangente, temos

cotg sen2 2

cos cotg sen2 2 sen

α αβα αφ

β

+ + = ⇔

2cos sen sen sen2 2 2

cotgsen sen cos

2 2

α α αβ βφ

α αβ

+ − ⇔ =

22sen cos 2sen sen2 2 2

cotgsen sen

α α αβ βφ

α β

+ − ⇔ =

2sen( ) sen 2sen sen2

cotgsen sen

αα β β βφ

α β

+ + − ⇔ =

2sen( ) sen 1 2sen2

cotgsen sen

αα β βφ

α β

+ + − ⇔ =

Page 84: Eureka 2003

Sociedade Brasileira de Matemática

EUREKA! N°17, 2003

22

sen( ) sen cos

cotgsen sen

α β β αφα β

+ +⇔ =

Calculemos θ. Uma prática normal em trigonometria é adotar o circunraio dealgum triângulo igual a 1/2, de modo que, pela lei dos senos, seus lados sejamiguais aos senos dos seus respectivos ângulos opostos. Podemos fazer isso porqueestamos só fixando o tamanho da figura. É claro que só podemos fazer isso umavez só em cada problema.

β

B M

A

θ

sen γ

sen α 2

Nesse caso, façamos isso com .ABC∆ Temos 1

/ 2 sen2

BM BC α= = e

sen sen( ).AB γ α β= = + No triângulo ABM,

2sen( )sen cotg cos cotg

sen sen( ) sen

2sen( ) sen cos

sen sen

BM AB α ββ θ β θθ θ β α

α β α βα β

+= ⇔ + = ⇔ =+

+ −=

Puxa, os resultados de cotgφ e cotgθ são diferentes! Na verdade, não são. Nuncaperca a fé!

Page 85: Eureka 2003

Sociedade Brasileira de Matemática

EUREKA! N°17, 2003

23

cotg cotg sen( ) sen cos 2sen( ) sen cosφ θ α β β α α β α β= ⇔ + + = + −sen( ) sen cos sen cos ,α β α β β α⇔ + = +

que é sempre verdade.

ALGUMAS IDENTIDADESSuponha que o circunraio do triângulo ABC é R = 1/2. Então, c = AB = senγ, b =AC = senβ e a = BC = senα.Além disso, por exemplo,

• O perímetro do triângulo é 2 4cos cos cos ;2 2 2

pα β γ =

• A área do triângulo é sen sen sen / 2;S α β γ=

• O inraio do triângulo é 2sen sen sen ;2 2 2

rα β γ =

• cos cos cos 1 / ;r Rα β γ+ + = +

• 2cos sen sen .2 2 2

p aα β γ − =

Exercício: Prove todas as identidades acima.

Exemplo:(IMO) Sejam AH1, BH2 e CH3 as alturas de um triângulo acutângulo ABC. Acircunferência inscrita no triângulo ABC é tangente aos lados BC, CA, AB em T1,T2 e T3, respectivamente. Considere a reta simétrica da reta H1H2 relativamente àreta T1T2, a reta simétrica da reta H2H3 relativamente à reta T2T3, a reta simétricada reta H1H3 relativamente à reta T1T3. Prove que estas retas simétricasdeterminam um triângulo cujos vértices pertencem à circunferência inscrita notriângulo ABC.

Resolução:Esse é o problema 6 da IMO de 2001.Primeiro, uma boa, e bem grande, figura. Vamos só desenhar a reta simétricarelacionada a T2T3. H é o ortocentro de ABC.

Page 86: Eureka 2003

Sociedade Brasileira de Matemática

EUREKA! N°17, 2003

24

B β

H1

β γ C

H

H3

T3

γ

X3π – α – γ = β2 2

π α γ− −

2 2

π α γ− −

P

A

2

π β−

2

π γ−

2

π β− 2 2

π α−

H2 β

T2

1�

Façamos o arrastão: veja que 2 3AH HH é inscritível, logo 3 2 .AH H γ∠ = Seja P a

interseção de 2 3T T e 2 3H H (só não podemos escolher duas retas i jTT e i jH H

concorrentes quando o triângulo ABC é equilátero; tal caso é trivial). Como

2 3AT AT= , os ângulos 2 3AT T∠ e 3 2AT T∠ medem ambos / 2 / 2.π α− Assim,

3 3 3 2 3 3 / 2H PT AT T PH T π α γ∠ = ∠ −∠ = − − e, sendo 1l a reta simétrica da reta

2 3H H relativamente à reta 2 3T T , o ângulo entre 1l e 2 3T T é igual também a

/ 2 .π α γ− − Logo o ângulo entre 1l e AB é 2( / 2 / 2 )π α γ γ π α γ β− − + = − − = ,

ou seja, 1l e BC são paralelos.

Definindo analogamente 2l e 3l , temos 2 //l AC e 3 // .l AB

Com isso, já sabemos que o triângulo determinado por 1l , 2l e 3l é semelhante a

ABC, e com lados homólogos paralelos. Temos, então, dois candidatos a taltriângulo:

Page 87: Eureka 2003

Sociedade Brasileira de Matemática

EUREKA! N°17, 2003

25

C

C'

B

B'

A'

A

C A' B

C'

A

B'

Estudando um caso particular (o triângulo equilátero, por exemplo), vemos que ocandidato mais indicado é o da direita. Podemos, então calcular a distância entrelados homólogos nessa situação e compararmos com a distância entre BC e 1.l

Assuma que o circunraio de ABC é 1/2, para termos sen ,BC α= senCA β= e

sen .BC α=Vamos calcular a distância entre BC e 1l . Seja 3X a interseção de 1l e AB. A

distância de A a 1l é 3sen .AX β E a distância desejada é 1 3sen .AH AX β− Bom,

1AH é fácil de calcular: 1 sen sen sen .AH AB β γ β= = E 3 ?AX 3AH é fácil de

calcular, 3AT também. Podemos calcular 3 3 3 3H T AT AH= − e usar a lei dos senos

no triângulo 3 3,PH X com a ceviana 3.PT Mãos à obra!!

Para começar, 3 cos sen cosAH AC α β α= = e 3 sen ,AT p α= − sendo p o

semiperímetro de ABC. Portanto 3 3 sen sen cos .H T p α β α= − −Pela lei dos senos no triângulo 3 3,PH T

3 3 3

3 3sen sen

PT H T

H PTγ=

∠No triângulo 3 3,PT X

3 3 3

3 3sen sen

PT X T

X PTβ=

∠Dividindo as duas últimas equações e tendo em vista que 3 3 3 3,H PT X PT∠ = ∠

obtemos 3 3 3 3

sen sen sen sen sen sen cos

sen sen

pX T H T

γ γ γ α γ β αβ β

− −= =

Page 88: Eureka 2003

Sociedade Brasileira de Matemática

EUREKA! N°17, 2003

26

Da lei dos co-senos no ABC∆ (ela também é útil de vez em quando!),2 2 2sen sen sen

sen sen cos2

β γ αγ β α + −=

Logo, substituindo sen sen sen

,2

pα β γ+ +=

2 2 2 2

3 3

sen sen sen sen sen sen sen sen

2senX T

α γ β γ γ β γ αβ

− + + − − +=

2 2sen sen sen sen sen sen

2sen

α γ β γ β αβ

− + − +=

Enfim, podemos calcular 3 3 3 3.AX AT X T= + Veja que

3

sen sen sensen .

2AT p

α β γα − + += − =

2 2

3

sen ( sen sen sen ) sen sen sen sen sen sen

2senAX

β α β γ α γ β γ β αβ

− + + − + − +=

2sen sen 2sen sen sen sen sen

2sen

α β β γ α γ αβ

− + − +=

Enfim, a distância entre 1l e BC é2

1 3

sen sen 2sen sen sen sen sensen sen sen

2AH AX

α β β γ α λ αβ β γ − + − +− = −

sen ( sen sen sen )

2

α α β γ− + +=

Na seção de identidades, você deve provar que

2cos sen sen2 2 2

p aα β γ − =

Logo a distância entre 1l e BC é (ufa!)

2cos sen sen sen2 2 2

dα β γ α =

Agora calculemos a distância entre os lados homólogos dos triângulos ABC e o delados respectivamente paralelos aos lados de ABC.

Page 89: Eureka 2003

Sociedade Brasileira de Matemática

EUREKA! N°17, 2003

27

A

C'

α

α r

I

B'

r

α

B C A'

Seja I o incentro do triângulo ABC. A distância de I a BC é igual ao inraio r e adistância de I a B'C' é r cosα.Assim, a distância entre BC e B'C' é:

2' cos (1 cos ) 2 cos2

d r r r rαα α = + = + =

Você tem outra identidade para provar:

2sen sen sen2 2 2

rα β γ =

Logo

2' 2 2sen sen sen cos2 2 2 2

dα β γ α = ⋅

2 2sen cos sen sen cos2 2 2 2 2

α α β γ α = ⋅ ⋅

2cos sen sen sen2 2 2

dα β γ α = =

Page 90: Eureka 2003

Sociedade Brasileira de Matemática

EUREKA! N°17, 2003

28

Conseqüentemente, 1l contém B' C'. Analogamente (ou você acha que eu faria

todas as contas de novo?), 2l contém A' C' e 3l contém A'B'.

Às vezes traçar novos elementos na figura também ajuda.

Exemplo(IMO) Seja P um ponto interior ao triângulo ABC tal que

APC ABC APB ACB∠ − ∠ = ∠ −∠Sejam D e E os incentros dos triângulos APB e APC, respectivamente. Prove queas retas BD, CE e AP passam por um ponto em comum.

ResoluçãoSeja .APC ABC APB ACBθ = ∠ −∠ = ∠ − ∠

γ + θ β + θ

P

A

B

α1 α2

C

Veja que podemos "separar" θ de β e γ. Note que se θ ficar "para baixo" obtemosum quadrilátero inscritível, então faremos isso.

Page 91: Eureka 2003

Sociedade Brasileira de Matemática

EUREKA! N°17, 2003

29

A

α1 α2

F β

α1 + γ

B

θ

γ β

θ P

G γ

α1 + β

C

O quadrilátero AFPG é inscritível, logo AFG β∠ = , ou seja, // .FG BCO problema pede, na verdade, para provarmos que as bissetrizes de ACP∠ e

ABP∠ se encontram sobre AP. Sejam Q e R as interseções de BD e CE com AP.Devemos ter Q = R. Do teorema das bissetrizes,

AQ AB

QP BP= e

AR AC

RP CP=

Como

,AQ AR AQ AR

Q R AQ ARAP AQ AP AR QP RP

= ⇔ = ⇔ = ⇔ =− −

é suficiente demonstrarmos queAB AC

BP CP=

Vamos, então, calcular BP e CP. Sendo FG paralela a BC, temos FB = k ⋅ AB.Aplicando a lei dos senos ao triângulo BFP, temos

( ) ( )1

1 1

sen( ) sen

sen sen sen senk ABBP FB AB

BPBP k

α γ θα γ θ θ α γ

⋅ += ⇔ = ⇔ =+ +

Page 92: Eureka 2003

Sociedade Brasileira de Matemática

EUREKA! N°17, 2003

30

Analogamente,

( )2

sen

senAC

CP k

θα β

=+

Como 1α γ+ e 2α β+ somam π , o resultado está demonstrado.

GEOMETRIA ANALÍTICAQuando aparecem problemas com muitos ângulos retos e que envolvam só retas,geometria analítica às vezes é indicada.

Exemplo(IMO) No quadrilátero convexo ABCD, as diagonais AC e BD são perpendicularese os lados opostos AB e CD não são paralelos. Sabemos que o ponto P, onde seintersectam as mediatrizes de AB e CD, está no interior de ABCD. Prove queABCD é um quadrilátero cíclico se, e somente se, os triângulos ABP e CDP têmáreas iguais.

ResoluçãoEsse problema é perfeito para se resolver com geometria analítica: é muito fácilcolocar as coisas nos eixos (tome como eixos as diagonais); tudo é muito fácil decalcular analiticamente (mediatrizes e áreas); e , por fim, a única condição quepoderia complicar, que é saber quando ABCD é cíclico, pode ser facilmentetransformada na potência da interseção das diagonais em relação ao seucircuncírculo.

y

x C

D

O

B

P A

(0 ; b)

(c ; 0)

(0 ; d)

(a ; 0)

Page 93: Eureka 2003

Sociedade Brasileira de Matemática

EUREKA! N°17, 2003

31

Sejam, então, A = (a; 0), B = (0, b), C = (c, 0) e D = (0; d). O quadrilátero ABCD éinscritível se, e somente se, .OA OC OB OD ac bd⋅ = ⋅ ⇔ = Fácil não?

Seja P = (x; y). Como P pertence às mediatrizes de AB e CD, temos PA = PB e PC= PD.

PA = PB ⇔ (x – a)2 + (y – 0)2 = (x – 0)2 + (y – b)2 ⇔ 2ax – a2 = 2by – b2

Analogamente, PC = PD ⇔ 2cx – c2 = 2by – b2. Resolvendo o sistema obtido,temos

2 2 2 2

2 2

2 2 2 2 2 2

( ) ( )

2 2 2( )

2 2 ( ) ( )

2( )

a b d c d bx

ax by a b ad bc

cx dy c d a b c c d ay

ad bc

− − −=− = − −

⇔− = − − − −=

Tudo bem com os denominadores pois, como AB e CD não são paralelos,/ / / / 0OA OB OC OD a b c d ad bc≠ ⇔ ≠ ⇔ − ≠ (nunca se esqueça de verificar

quando os denominadores são nulos; essa verificação às vezes faz você perceberque tem que estudar alguns casos em separado).

A área do triângulo PAB é igual a |D|/2 em que

1

0 1

0 1

x y

D a ay bx ab

b

= = − − +

Da mesma forma, a área do triângulo PCD é igual a ' / 2,D em que

1

' 0 1

0 1

x y

D c cy dx cd

d

= = − − +

Assim, devemos ter

Page 94: Eureka 2003

Sociedade Brasileira de Matemática

EUREKA! N°17, 2003

32

ay bx ab cy dx cd− − + = − − +

Seria muito bom nos livrarmos do módulo. O sinal de D depende da ordem emque colocamos as coordenadas no determinante. Se os pontos correspondentesestão dispostos no sentido anti-horário, D é positivo; se estão no sentido horário, énegativo. Como P pertence ao interior de ABCD, PAB e PCD têm a mesmaorientação, de modo que realmente podemos nos livrar do módulo. Logo, tirandoo módulo e substituindo x e y, temos que as áreas de PAB e PCD são iguais se, esomente se,

2 2 2 2 2 2 2 2( ) ( ) ( ) ( )( ) ( )

2( ) 2( )

a b c c d a a b d c d ba c b d ab cd

ad bc ad bc

− − − − − −− + − = −− −

(**)

Nada de abrir tudo com pressa! Queremos ac = bd, e isso significa queprovavelmente em algum momento fatoraremos a equação com ac – bd como umdos fatores.

(**)2 2 2 2 2 2 2 2( )( ) ( )( ) 2( )( )a b ac bd c d c d ac bd a b ab cd ad bc⇔ − + − − + − + − − = − −

2 2 2 2 2 2 2 2 2 2 2 2 2 2 2 2 2 2 2 2( )( )ac bd a c b d a c a d b c b d a c b c a d b d⇔ + + − − − − + + − − + +2 2 2 22( )a bd ab c acd bc d= − − +

2 2 2 2 2 2 2 2 2 2 2 2( ) ( ) 2( )ac a c bd b d acb acd bda bdc a c b d⇔ + − + − − + + − −

2 2 2 22( )a bd ab c acd bc d= − − +

2 2 2 2 2 2 2 2 2 2 2 2( ) ( ) ( ) 2( ) 0ac a c bd b d acb acd bda bdc a c b d⇔ + − + − − − + + − − =

2 2 2 2 2 2 2 2( ) ( ) ( ) ( ) 2( )( ) 0ac a c bd b d ac b d bd a c ac bd ac bd⇔ + − + + + − + − − + =2 2( )(( ) ( ) ) 0ac bd a c b d⇔ − − + − =

ac bd⇔ = ou ( e )a c b d= =

Não é possível termos a = c e b = d pois já vimos que ad bc≠ . Logo as áreas dePAB e PCD são iguais se, e somente se, ac = bd.

Page 95: Eureka 2003

Sociedade Brasileira de Matemática

EUREKA! N°17, 2003

33

A geometria analítica tem uma pequena desvantagem: não passa de aplicaçõesextensivas do teorema de Pitágoras.

Apesar de Pitágoras resolver problemas como o que acabamos de ver, mesclar umpouco as contas com trigonometria e números complexos pode vir a calhar.

Agora, alguns problemas para você pensar.

PROBLEMAS

01. Seja ABC um triângulo acutângulo, M o ponto médio do segmento BC, P oponto sobre o segmento AM tal PM = BM, H o pé da perpendicular de P a BC,Q o ponto de interseção entre o segmento AB e a reta que passa através de H eé perpendicular a PB e, finalmente, R o ponto de interseção entre o segmentoAC e a reta que passa através de H e é perpendicular a PC. Mostre que ocircuncírculo do triângulo QHR é tangente a BC no ponto H.

02. No triângulo ABC, AB = AC. D é um ponto sobre o lado BC tal que BD = 2CD. Se P é o ponto de AD tal que ,ABP PAC∠ = ∠ prove que

2 .DPC BAC∠ = ∠

03. Um quadrilátero convexo está inscrito em uma circunferência de raio unitário.Demonstre que a diferença entre seu perímetro e a soma das diagonais é maiordo que zero e menor do que 2.

04. (IMO) O prolongamento da bissetriz AL do triângulo acutângulo ABCintercepta a circunferência circunscrita no ponto N. A partir do ponto Ltraçam-se perpendiculares LK e LM aos lados AB e AC, respectivamente.Prove que a área do triângulo ABC é igual a área do quadrilátero AKNM.

05. (Ibero) A circunferência inscrita no triângulo ABC é tangente aos lados BC,CA e AB nos pontos D, E e F, respectivamente. AD corta a circunferência numsegundo ponto Q. Demonstrar que a reta EQ passa pelo ponto médio de AF se,e somente se, AC = BC.

06. (IMO) Seja I o incentro do triângulo ABC. A circunferência inscrita notriângulo ABC é tangente aos lados BC, CA e AB nos pontos K, L e M,respectivamente. A reta que passa por B, paralela ao segmento MK, interceptaas retas LM e LK nos pontos R e S, respectivamente. Prove que o ângulo ∠RISé agudo.

Page 96: Eureka 2003

Sociedade Brasileira de Matemática

EUREKA! N°17, 2003

34

07. (Vietnã) Seja ABC um triângulo e A', B', C' pontos médios dos arcos BC, AC eAB do circuncírculo de ABC, respectivamente. As retas A'B' e A'C'interceptam o lado BC em M e N, respectivamente. Defina os pares de pontosP, Q e R, S analogamente. Prove que MN = PQ = RS se, e somente se, ABC éequilátero.

08. (IMO) Seja ABC um triângulo acutângulo com circuncentro O. Seja PA umaaltura do triângulo com P no lado BC.

Considere que 30 .BCA ABC∠ ≥ ∠ + °

Prove que 90 .CAB COP∠ + ∠ < °

09. (IMO) Num triângulo ABC, seja AP a bissetriz de BAC∠ com P no lado BC, eseja BQ a bissetriz de ABC∠ com Q no lado CA.

Sabemos que 60BAC∠ = ° e que AB + BP = AQ + QB.

Quais são os possíveis valores dos ângulos do triângulo ABC?

10. (Coréia) Sejam R e r o circunraio e o inraio, respectivamente, do triânguloABC, e R' e r' o circunraio e o inraio, respectivamente, do triângulo A'B'C'.Prove que se 'C C∠ = ∠ e ' 'Rr R r= então os triângulos são semelhantes.

11. (Turquia) Sejam AC e PC a área e o perímetro, respectivamente, doquadrilátero cíclico C. Se a área e o perímetro do quadrilátero cujos lados sãotangentes ao circuncírculo de C são AT e PT , respectivamente, prove que

2

C C

T T

A P

A P

12. (EUA) Seja ABCD um trapézio isósceles com AB // CD. O incírculo dotriângulo BCD toca CD em E. Seja F um ponto da bissetriz de DAC∠ tal que

.EF CD⊥ O circuncírculo do triângulo ACF corta a reta CD em C e G.

Mostre que o triângulo AFG é isósceles.

Page 97: Eureka 2003

Sociedade Brasileira de Matemática

EUREKA! N°17, 2003

35

13. (Balcânica, adaptado) Seja ABC um triângulo acutângulo e M, N e P asprojeções ortogonais do baricentro de ABC sobre seus lados. Prove que

2 [ ] 1

9 [ ] 4

MNP

ABC< ≤

([XYZ] é a área do triângulo XYZ)

14. (Ibero) Dados dois círculos 1ω e 2ω , dizemos que 1ω bissecta 2ω quando se

intersectam e a corda comum é um diâmetro de 2ω . Se 1ω e 2ω são idênticas,

dizemos que 1ω e 2ω bissectam-se mutuamente. Considere dois círculos fixos

e não concêntricos 1ω e 2ω .

(a) Mostre que há infinitos círculos ω que bissectam tanto 1ω como 2ω .

(b) Encontre o lugar geométrico do centro de ω .

15. (Ibero) Seja ABC um triângulo acutângulo com circuncírculoω centrado emO. Seja AD, BE e CF as alturas de ABC. A reta EF corta ω em P e Q.

(a) Prove que .AO PQ⊥

(b) Se M é o ponto médio de BC, prove que 2 2AP AD OM= ⋅

16. (São Petersburgo) Seja AL uma bissetriz interna do triângulo ABC, com Lsobre BC. As retas paralelas 1l e 2l passam por B e C, respectivamente, e são

equidistantes de A. Os postos M e N pertencem a 1l e 2l , respectivamente, e

são tais que os pontos médios de LM e LN pertencem a AB e AC,respectivamente. Prove que LM = LN.

17. (IMO) No plano, considere uma circunferência C, uma reta L tangente àcircunferência e M um ponto da reta L. Encontre o lugar geométrico dos pontos Pcom a seguinte propriedade: existem dois pontos Q, R da reta L tais que M é oponto médio de QR e C é a circunferência inscrita no triângulo PQR.

Page 98: Eureka 2003

Sociedade Brasileira de Matemática

EUREKA! N°17, 2003

36

A ENUMERABILIDADE DE ��

× ��

E O CHÃO TRIANGULARJosé Paulo Carneiro

♦ Nível Intermediário

Uma das maneiras mais conhecidas de mostrar que o conjunto ×� � éenumerável, isto é, que existe uma bijeção entre ×� � e � , (onde � = {0; 1; 2;…} é o conjunto dos números naturais) é exibir uma bijeção de ×� � sobre � ,inspirada na figura:

(0; 0) (0; 1) (0; 2) (0; 3) …(1; 0) (1; 1) (1; 2) (1; 3) …(2; 0) (2; 1) (2; 2) (2; 3) …(3; 0) (3; 1) (3; 2) (3; 3) …

… … … …

a qual sugere a enumeração:

(0; 0); (1; 0); (0; 1); (2; 0); (1; 1); (0; 2); (3; 0); (2; 1); (1; 2); (0; 3);… Ou seja,colocamos, sucessivamente, os pares (a; b) tais que a soma a + b assuma osvalores 0; 1; 2; 3; …, e dentro da cada grupamento que tenha a + b constante(correspondente, na figura, a uma das diagonais indicadas), ordenamos os parespela ordem natural de sua segunda componente.Obtém-se então a bijeção:

f : ×� �→ �(0; 0) → 0(1; 0) → 1(0; 1) → 2(2; 0) → 3(1; 1) → 4(0; 2) → 5………….

Onde f(x; y) é o lugar que ocupa (x; y) nesta enumeração. (Como estamosincluindo 0 em N, é preciso começar a contar a partir do 0-ésimo lugar).Uma questão interessante é produzir uma fórmula explícita para esta função eutilizar esta fórmula para provar que f é realmente uma bijeção.

Page 99: Eureka 2003

Sociedade Brasileira de Matemática

EUREKA! N°17, 2003

37

Para isto, seja (x; y) ∈ ×� � . Observando a figura, vê-se que se (x; y) for tal quex + y = s > 0, então o par (x; y) é precedido, pelo menos, por todos os pares (u; v)tais que u + v = 0; 1; 2;…;s – 1. Existe um par que tem soma 0, dois que têm soma1, e assim por diante, até s pares que têm soma s – 1, de modo que esses pares são

em número de ( 1)

1 ...2

s ss

++ + = . Além disto, já na sua diagonal, o par (x; y) é

precedido por y pares.

Portanto, 2( )( 1) ( ) 3

( ; )2 2

x y x y x y x yf x y y

+ + + + + += + = . Finalmente,

constata-se diretamente que esta fórmula também é válida se (x; y) = (0; 0).Podemos então concluir que f é dada pela fórmula:

:f × →� � �

2( ) 3( ; )

2

x y x yf x y

+ + +=

Pode ser útil ao leitor testar esta fórmula para pares específicos.Imaginemos agora que seja apresentado ao leitor, sem nenhuma menção a suaorigem, o seguinte problema: provar que a função definida por

2( ) 3( ; )

2

x y x yf x y

+ + += é uma bijeção de ×� � sobre � .

Naturalmente, o leitor iniciará tentando provar que f é injetiva, mostrando quef(a; b) = f(c; d) implica (a; b) = (c; d). Mas da equação

2 2( ) 3 ( ) 3

2 2

a b a b c d c d+ + + + + += não é imediato concluir que (a; b) = (c; d), como

o leitor pode experimentar.

No entanto, se lembrarmos a maneira pela qual criamos a fórmula definidora de f,podemos raciocinar do seguinte modo. Suponhamos primeiro que a + b = c + d(ou seja, (a; b) e (c; d) estão na mesma diagonal). Neste caso, f(a; b) = f (c; d) ⇒(a + b)2 + a + 3b = (c + d)2 + c + 3d ⇒ a + b + 2b = c + d + 2d ⇒ 2b = 2d ⇒ b = de, portanto, a = c.

Por outro lado; se a + b ≠ c + d, podemos supor, sem perda de generalidade, quea + b > c + d. Mas então (a + b)2 > (c + d )2 e, como f (a; b) = f(c; d), obtém-se(a + b)2 – (c + d)2 = 3d + c – (3b + a) > 0, mas (a + b)2 – (c + d )2 =

Page 100: Eureka 2003

Sociedade Brasileira de Matemática

EUREKA! N°17, 2003

38

(a + b – c – d)(a + b + c + d) ≥ a + b + c + d, e logo 3d + c–(3b +a) ≥ a + b + c +d,donde 2d ≥ 2a + 4b, e portanto c + d ≥ d ≥ a + 2b ≥ a + b, absurdo.Logo, não pode ocorrer o caso a + b > c + d, e a conclusão final é que f é injetiva.Provar a sobrejetividade de f, sem apelar para a forma pela qual criamos f, nãoparece fácil. O que queremos provar é que, dado n ∈ � , existe um par (x; y) tal

que 2( ) 3

( ; ) .2

x y x yf x y n

+ + += = É aconselhável que o leitor primeiro tente

fazê-lo, para sentir a dificuldade de calcular x e y em função de n.

Para começar por um exemplo concreto, se n = 50, quem é (x; y) = f – 1(50)? Ouseja, calcule x e y naturais (e já sabemos, pela injetividade, que serão únicos!) tais

que 2( ) 3

50.2

x y x y+ + + = Se apelarmos para o esquema das diagonais, vamos

iniciar perguntando em que diagonal está o 50o. par. Para isto, vamos descobrirprimeiro quais números da forma 1; 1 + 2 = 3; 1 + 2 + 3 = 6, etc., precedem 50.Calculando "no braço", vemos que 50 é precedido por 1; 3; 6; 10; 15; 21; 28; 36;45 = 1 + 2 + 3 + 4 + 5 + 6 + 7 + 8 + 9 (o próximo já seria 55). Isto significa que f – 1(50) é precedido pelos 45 pares (u; v) que têm somas u + v = 0; 1; 2;…;8,sendo o último deles igual a (0; 8) = f – 1(44) (não esqueça de contar a partir do 0).A diagonal seguinte começa com o par (9; 0) = f – 1(45). Para chegar ao 50,precisamos andar mais cinco pares sobre esta diagonal: (8; 1); (7; 2); (6; 3); (5; 4);(4; 5), chegando até o (4; 5) = f – 1(50). Este par é o 50o. da seqüência (nuncaesquecendo de contar a partir do 0) e, portanto, é o único que satisfaz a f(x; y) =50.Para abordar o caso geral, vamos primeiro lembrar que os números T1 = 1; T2 = 1

+ 2 = 3, T3 = 1 + 2 + 3,…, ( 1)

1 ...2k

k kT k

+= + + = , são chamados números

triangulares (ver [1]). Dado n (vamos considerar somente n > 0, já que f – 1(0) =(0; 0) é imediato), para determinar (x; y) = f – 1(n) pelo método usado acima, énecessário primeiro determinar o maior número triangular Tk tal que kT n≤ , que

vamos chamar de chão triangular de n. Por exemplo, como vimos, o chãotriangular de 50 é 45.A existência do chão triangular de n, no caso geral, decorre do seguinte: aseqüência T1; …; Tm ;… é ilimitada superiormente; logo, algum Tm é maior doque n; pelo Princípio da Boa Ordenação dos números naturais (ver [2]), seja Tj omenor dos Tm que são maiores do que n; então Tj–1 é o chão triangular de n; defato, 1j jT n T− ≤ < .

Page 101: Eureka 2003

Sociedade Brasileira de Matemática

EUREKA! N°17, 2003

39

Para mostrar a unicidade do chão triangular de n, suponha (por absurdo) que

1p pT n T +≤ < e 1q qT n T +≤ < com p < q (sem perda de generalidade). Como a

seqüência (Tm) é estritamente crescente, 1 ,p qn T T n+< ≤ ≤ absurdo.

Uma vez então determinado Tk, o chão triangular de n, o par (x; y) = f – 1(n) estarána diagonal de soma k, formada pelos pares (k; 0); (k – 1; 1);…;(0; k). Nestadiagonal, f(k; 0) = Tk; f(k – 1; 1) = Tk + 1;…; f(0; k) = Tk + k = Tk + 1 – 1, ou seja, deum modo geral: f(k – j; j) = Tk + j. Quando j = n – Tk, termos

( ; )k kf k n T n T n− + − = . Logo: 1( ; ) ( ) ( ; ).k kx y f n k n T n T−= = − + − Por exemplo:1(50) (9 50 45;50 45) (4;5).f − = − + − =

Como conseqüência deste raciocínio, fica claro que f é injetiva e sobrejetiva e quese pode determinar de modo único f – 1(n) para cada n pela fórmula:

1 0; se 0( )

( ; ); se 0k k

nf n

k n T n T n− =

= − + − >

onde kT é o chão triangular de n.

Mas permanece um problema mais sutil; existe alguma fórmula fechada quecalcule k tal que kT seja o chão triangular de um dado n?

Para responder a isto, observe que a solução positiva de ( 1)

2x x

n+ = é

0

8 1 1

2n

x+ −= e que a função

( 1)( )

2x x

f x+= é crescente em [0; [.+∞ Sendo, pois

0k x= (onde 0x simboliza o chão inteiro de 0x , ou a parte inteira de 0x ),

temos 0 1k x k≤ < + e, portanto: 0 01

( 1)( 1) ( 1)( 2).

2 2 2k k

x xk k k kT n T +

++ + += ≤ = < =

Page 102: Eureka 2003

Sociedade Brasileira de Matemática

EUREKA! N°17, 2003

40

Conclusão: uma expressão explícita para k tal que kT seja o chão triangular de n

é:

8 1 1.

2n

k + −=

Por exemplo, para n = 50, temos: 401 1

9,51... 9.2

k −= = =

Referências:

[1] Carneiro, J.P.; Contar de duas maneiras, para generalizar; Eureka! No. 6; pp. 15-17.[2] Carneiro, J.P; O Princípio da Descida Infinita de Fermat; Revista do Professor de Matemática,No. 32; 3o. quadrimestre de 1996; pp. 39-44.

Page 103: Eureka 2003

Sociedade Brasileira de Matemática

EUREKA! N°17, 2003

41

COMO É QUE FAZ?

PROBLEMA 1PROPOSTO POR ASDRUBAL SANTOS (BOTUCATU – SP)

Encontre todas as funções :f + +→� � tais que:

i) ( ) 0,f x x +≥ ∀ ∈�ii) ( ( ) ) 2 ,f f x x x− = para todo x +∈� .

SOLUÇÃO DE GUILHERME RODRIGUES NOGUEIRA DE SOUZA (SÃO PAULO - SP)

Seja a função :g + +→� � tal que ( ) ( ) ,g x f x x= − para todo x +∈�Logo:

( ( )) 2f g x x=( ) ( ( )) 2g x g g x x⇔ + =( ( )) ( ) 2g g x g x x⇔ = − + (I)

Seja: 0 1( ) , ( ) ( ( )), para todo .n ng x x g x g g x n+= = ∈�Logo, por (I):

( ) ( ) ( )( ( ( ))) ( ( )) 2 ( )n n ng g g x g g x g x= − +( 2) ( 1) ( )( ) ( ) 2 ( )n n ng x g x g x+ +⇔ = − +

Seja ( ) ( ),nna g x= com x fixo.

Então chegamos na seguinte recursão:

2 1 2n n na a a+ += − +Resolvendo essa recursão chegamos em:

( )( ) 2 ( )1 2

3 3nn

n

g x x g x xa

+ − = ⋅ − −

( )( ) ( ) 2 ( )( ) 2

3 3nn g x x g x x

g x+ − ⇔ = − −

Para n suficientemente grande temos

( ) 2 ( )2 ,

3 3ng x x g x x+ −< desde que ( ) .g x x≠

Suponha, por absurdo que ( )g x x> para algum x +∈�

Page 104: Eureka 2003

Sociedade Brasileira de Matemática

EUREKA! N°17, 2003

42

Tome n par. Temos que ( ) ( )ng x é negativo se n é suficientemente grande, o que é

absurdo, pois g é de + +→� � .

Suponha agora que ( ) .g x x< Também por absurdo, tomando n ímpar, concluímos

que ( ) ( )ng x é negativo, e como já vimos antes isso é um absurdo.

Portanto, para todo x positivo, ( )g x x> e ( )g x x< são falsos.

Logo g(x) = x, para todo x positivo. Então f(x) – x = x ⇔ f(x) = 2xVamos testar f(x) = 2x.Temos: ( ( ) ) 2f f x x x− = (2 ) 2f x x x⇔ − = ( ) 2f x x⇔ = 2 2 ,x x⇔ =então a única função que satisfaz o enunciado é ( ) 2 .f x x=

PROBLEMA 2PROPOSTO POR ASDRUBAL SANTOS (BOTUCATU – SP)

Mostre que não existem funções :f →� � tais que 2( ( )) 2, .f f x x x= − ∀ ∈�

SOLUÇÃO DE TELMO LUIS CORREA JÚNIOR (SÃO PAULO – SP)Vamos determinar os possíveis valores de x para que f ( f (x)) = x:

2 2 2x x x= − ⇔ = ou x = – 1.Agora, os valores tais que ( ( ( ( )))) :f f f f x x=

2( ( 2))f f x x⇔ − =2 2( 2) 2x x⇔ − − =

4 24 2 0x x x⇔ − − + =Mas 2 e –1 devem ser raízes dessa equação, pois

( ( )) ( ( ( ( ))))f f x x f f f f x x= ⇒ = .Fatorando, obtemos:

2( 2)( 1)( 1) 0x x x x− + + − =

1 5 1 5( 2)( 1) 0

2 2x x x x

− + − −⇔ − + − − =

Logo os únicos valores tais que ( ( ( ( ))))f f f f x x= são 2, –1, 1 5

2

− + e

1 5.

2

− −

Considere os valores , ( ), ( ( )) e ( ( ( ))).x f x f f x f f f x

Page 105: Eureka 2003

Sociedade Brasileira de Matemática

EUREKA! N°17, 2003

43

f(x) f f

x f(f (x))

f f

f(f (f (x))

Para 1 5

, ( ( ( ( )))) .2

x f f f f x x− += =

Considere os possíveis valores de f(x): como( ( ( ( ( ))))) ( ),f f f f f x f x= ( )f x pode ser

apenas 2, –1, 1 5 1 5

, .2 2

− − − +

De fato, se 1 5

( ) ,2

f x− += então ( )f x x= ,

logo ( ( )) ( ).f f x f x= Absurdo, pois temos2

1 5 1 5 1 52 .

2 2 2f f

− + − + − += − ≠

2 f f

1 5

2

− + f (2)

f f

2

Se f(x) = 2, como f (f (2)) = 2, então f( f ( f (x))) = 2,

logo f(2) = 1 5

2

− +, absurdo.

Se f(x) = –1, de modo análogo ao caso f(x) = 2chegamos a um absurdo.

Se f(x) = 1 5

2

− −, os possíveis valores para f( f(x))

são 1 5 1 5

,2 2

− + − −, 2 ou –1.

f f

1 5

2

− + f (f (x))

f f

1 5

2

− −

f (f (f (x)))

Se f (f (x)) = 1 5

2

− +, então x = f (f (x)), absurdo.

Se f (f (x)) = 1 5

2

− −, então

1 5 1 5,

2 2f − − − −=

Page 106: Eureka 2003

Sociedade Brasileira de Matemática

EUREKA! N°17, 2003

44

Logo 1 5 1 5 1 5

( ( )) ,2 2 2

f f f − + − − − −= =

absurdo.

Se f ( f(x)) = 2, f ( f (f ( f (x)))) = 2 ⇒ 2 = 1 5

2

− +, absurdo.

Se f ( f(x)) = –1, f ( f ( f ( f (x)))) = –1 ⇒ –1 = 1 5

2

− +, absurdo.

Como em qualquer caso f(f(x)) = x2 – 2 é um absurdo, não existe função f nessascondições.

PROBLEMA 3PROPOSTO POR DAVI MÁXIMO ALEXANDRINO NOGUEIRA (FORTALEZA - CE)Prove que é possível decompor o conjunto {1, 2, 3,…,2n} em dois subconjuntos Ae B não contendo nenhuma progressão aritmética de tamanho 2n.

SOLUÇÃO:Vamos escolher o conjunto A aleatoriamente:Cada elemento de U = {1, 2,…,2n} é colocado em A (ou em B = U \ A) comprobabilidade 1/2, de forma independente. Agora, se fixarmos uma PA com 2n

termos, a probabilidade de ela estar contida em A ou em B é 2 2 1

1 12 .

2 2n n−⋅ = Por

outro lado, podemos estimar o número de progressões aritméticas de tamanho 2n

em U da seguinte forma: temos no máximo 2

2 1

n

n − possibilidades para a razão e

no máximo 2n possibilidades para o termo inicial, e logo temos no máximo 22

2 1

n

n −tais PA's.Como, para cada uma dessas PA's, a probabilidade de ela estar contida em A ou

em B é 2 1

1

2 n− , a probabilidade de alguma dessas PA's estar contida em A ou em B

é, no máximo, 2

2 1

2 1 21

2 1 2 2 1

n

nn n−⋅ = <− −

, para todo 2,n ≥ donde há exemplos

(de fato a grande maioria, se n é grande) de decomposições U = A ∪ B onde nemA nem B contêm progressões aritméticas de tamanho 2n (para n = 1 podemostomar o exemplo A ={1} e B ={2}).

Page 107: Eureka 2003

Sociedade Brasileira de Matemática

EUREKA! N°17, 2003

45

SOLUÇÕES DE PROBLEMAS PROPOSTOS� Publicamos aqui algumas das respostas enviadas por nossos leitores.

68. Seja ABC um triângulo de lados inteiros e área racional. Prove que existempontos X, Y, Z com coordenadas inteiras no plano �2 tais que o triângulo XYZ écongruente ao triângulo ABC.

SOLUÇÃO DE ZOROASTRO AZAMBUJA NETO (RIO DE JANEIRO – RJ)Sejam a, b, c as medidas dos lados do triângulo, respectivamente opostos aosvértices A, B e C. Podemos supor que (0,0)A = , ( ,0)B c= e ( , )C m h= . Temos

que c é inteiro e 2 2 2 2 2 2, ( ) ,m h b m c h a+ = − + = com a e b inteiros. Daí obtemos2 2 2

2

b a cm

c

− += e logo 2 2 2 2 2 2

22

4 ( ).

4

b c b a ch

c

− − += Como a área 2

ch de ABC é

racional, segue que h é racional, e logo 2 2 2 2 2 24 ( )b c b a c− − + é um quadrado

perfeito. Isso implica que 2 2 2b a c− + é par (senão 2 2 2 2 2 24 ( )b c b a c− − + seriacongruente a 3 módulo 4, absurdo).

Assim, r

mc

= e s

hc

= com r, s inteiros, e portanto, 2 2 2 2r s b c+ = (pois

2 2 2m h b+ = ).Vamos agora mostrar que existem , , ,x y u v∈� tais que 2 2 2x y c+ = , 2 2 2u v b+ = e( )( ) .x yi u vi r si+ + = + Isso resolve o problema, pois o número complexo

x yi

c cϕ = − tem módulo 1, e portanto a multiplicação por α é uma rotação em 2

que leva A, B, e C em ( ) ( )

(0,0), ( , ) e ( )x yi r si

X Y x y Z m hic c

ϕ− +

= = − = + = ⋅ =

2

( )( )( )( , ).

x yi x yi u viu vi u v

c

− + += = + = Assim, XYZ é congruente a ABC e

seus vértices têm coordenadas inteiras.Vamos mostrar a existência dos x, y, u, v: queremos mostrar que se

2 2 2 2( ) ,N r si r s b c+ = + = com b, c, r, s ∈ � então existem x, y, u, v ∈ � com

( )( ) ,x yi u vi r si+ + = + 2 2 2( )N x yi x y c+ = + = e 2 2 2( ) .N u vi u v b+ = + =Vamos provar isso por indução em ( )N r si+ (se ( ) 1N r si+ = o resultado é

óbvio), usando existência e unicidade de fatoração em [ ] { , , }i a bi a b= + ∈� �

(vejam o artigo de Guilherme Fujiwara na Eureka! No. 14).

Page 108: Eureka 2003

Sociedade Brasileira de Matemática

EUREKA! N°17, 2003

46

Se 1,c = tomamos x = 1, y = 0, u = r e v = s. Se 1,c > tome α um fator

irredutível de c. Temos que 2 2 ( )( ),b c r si r siα = + − e logo r siα + ou r siα − (e,

nesse caso r siα + ). Suponha então, sem perda de generalidade, que r siα + .

Temos então [ ]r si

iα+ ∈� e

( ) ( ).

r si r sir siN

α α α+ −+ = ⋅

Se ( )N α é um

quadrado perfeito (o que nesse caso equivale a α ser o produto de um elemento de� por um elemento de {1, i}), existem, por hipótese de indução, , , ,x y u v ∈� � �

com ( )( ),r si

x yi u viα+

= + +� �

22 2 2 2 2, ,

( )

cx y u v b

N α+ = + =� � e basta tomar ( ) ( )x yi x yiα+ = +� �

para concluir. Caso contrário, como | , | ,c cα α e logo | cαα (pois α é irredutível e

[ ]).iαα

∉�

� Assim, 2 2 2 2 2| | ( )( ),c b c r si r siα α = + − e portanto | r siαα + ou

2 | .r siα + No primeiro caso, 2

22

( ),

( ) ( )

r si N r si cN b

N Nαα αα α + + = = ⋅

e podemos

escrever ( )( )r si

x yi u viαα+

= + +� � com 2

2 2 2 2 22

, ,( )

cx y u v b

N α+ = + =� � e basta

tomar ( ).x yi x yiαα+ = +� � O segundo caso é análogo:2

22 2 2

( ),

( ) ( )

r si N r si cN b

N Nα α α + + = = ⋅

e podemos escrever

2( )( )

r six yi u vi

α+

= + +� � com 2

2 2 2 2 22

, ,( )

cx y u v b

N α+ = + =� � e basta tomar

2 ( ).x yi x yiα+ = +� �

69. Sejam a e b inteiros positivos tais que 1na − divide 1nb − para todo inteiropositivo n.Prove que existe k ∈ � tal que kb a= .

SOLUÇÃO DE ZOROASTRO AZAMBUJA NETO (RIO DE JANEIRO – RJ)Suponha por absurdo que b não seja uma potência de a.Então existe k ∈ � tal que 1.k ka b a +< < Consideremos a seqüência

1, 1.

1

n

n n

bx n

a

−= ∈ ∀ ≥−

� Como 2

1

1 1 1 1... ,

1n n n jnja a a a

=

= + + =− ∑ temos

Page 109: Eureka 2003

Sociedade Brasileira de Matemática

EUREKA! N°17, 2003

47

21 1

1 1... .

( 1) 1

n n nn n

n jn jn k kn n nj j

b b b b bx

a a a a a a a a

∞ ∞

= =

= − = + + + − − − ∑ ∑ Note que

como ( )1

( 1) 1

nkn

kn n n

b ab

a a a

+

−=− −

e 1

1na − tendem a 0 quando n cresce, se definimos

21

... ,n n n nk

n k jj

b b b by

a a a a=

= + + + = ∑ temos que

1

( 1) 1

n

n n kn n n

bx y

a a a− = −

− −tende a 0 quando n tende a infinito. Por outro lado, como ny é uma soma de k

progressões geométricas de razões jb a , 1 j k≤ ≤ , ny satisfaz a equação de

recorrência 0 1 1 ... 0,n k n k k nC y C y C y+ + −+ + + = 0 ,n∀ ≥ onde

1 ( 1) / 20 1 1 2

... ...k k k kk k k

b b bC x C x C x C a x x x

a a a− +

− + + + + = − − −

(vejam o artigo "Equações de recorrência" na Eureka! No. 9).Note que todos os Ci são inteiros. Note também que

0 1 1 0 1 1 1... ( ) ( ) ... ( )n k n k k n n k n k n k n k k n nC x C x C x C x y C x y C x y+ + − + + + − + −+ + + = − + − + + −tende a 0 quando n tende a infinito, pois n j n jx y+ +− tende a 0 para todo j com

0 j k≤ ≤ (e k está fixo). Como os iC e os nx são todos inteiros, isso mostra que

0 1 1 ... 0n k n k k nC x C x C x+ + −+ + + = para todo n grande.

Agora, como 1 ( 1)

1,

( 1) 1

n n

n n k k n n n

b bx y

a a a a+ + = + + − − −

temos

0 1 1 10

... ,n k jk

n k n k k n j n k jkj

bC x C x C x C z

a

+ −

+ + − + −+=

+ + + = + ∑ onde

( )1

1.

1( 1)

m

m mk m m

bz

aa a+= −

−−

Note que 1 1 1

0

n k j nk

k k k kj

b b bC P

a a a

+ −

+ + +=

= ⋅ ∑ , onde

1 ( 1) / 20 1 1 2

( ) ... ... ,k k k kk k k

b b bP x C x C x C x C a x x x

a a a− +

− = + + + + = − − −

donde

10.

k

bP

a + ≠

Por outro lado, para todo j com

Page 110: Eureka 2003

Sociedade Brasileira de Matemática

EUREKA! N°17, 2003

48

0 ,j k≤ ≤( )

( )( )1

1

1,

1

k jkn

n k j nk n k j k j n k

b abz

a a a a b a

−+

+ − + + − − −

= − − − que tende a 0

quando n tende a infinito, donde 1

0

nk

n j n k j kj

bw C x

a+ − +=

= ∑ tende a

10

k

bP

a + ≠

, o que é um absurdo, pois, como vimos antes, wn é igual a 0 para

todo n grande.

71. Considere três circunferências, tangentes duas a duas. Prove que há apenasduas circunferências tangentes às três simultaneamente, e mostre como construí-las.

SOLUÇÃO DE ANTONIO CAMINHA MUNIZ NETO (FORTALEZA – CE)

Prova Analisemos o caso em que as três circunferências, α, β, γ digamos, sãotangentes exteriormente duas a duas (a análise dos demais casos é completamenteanáloga).

A

α

β

γ

Seja A o ponto de tangência das circunferências β e γ. Aplicando àconfiguração mostrada na figura acima a inversão I de pólo A e módulo igual àpotência de A em relação a α, a circunferência α permanece fixa, ao passo que ascircunferências β e γ são respectivamente transformadas em retas distintas r e s,perpendiculares à reta que une seus centros. Como inversões preservam tangênciade curvas, segue que r e s são ainda tangentes a α, de modo que a situação éexatamente a da figura abaixo (observe que r e s podem ser construídas com régua

Page 111: Eureka 2003

Sociedade Brasileira de Matemática

EUREKA! N°17, 2003

49

e compasso: elas são as tangentes a α que são perpendiculares à reta dos centrosde β e γ):

α

s r

Seja δ uma circunferência qualquer que tangência α, β e γ. É claro que δ nãocontém A, donde concluímos que sua inversa δ’ ainda é uma circunferência,ademais tangente às inversas de α, β e γ, quer dizer, tangente a α, r e s. Portanto,segue imediatamente da figura acima que só há duas possibilidades para δ’,digamos δ’1 e δ’2. Tais circunferências podem ser facilmente construídas comrégua e compasso, basta para tanto observar que, sendo R o raio e O o centro de α,os centros de δ’1 e δ’2 são os pontos situados sobre a paralela a r por O, distando2R de O. Portanto, há exatamente duas circunferências tangentes simultaneamenteα, β e γ e tais circunferências são as inversas δ1 e δ2 de δ’1 e δ’2 por I (as quaistambém podem ser facilmente construídas).

73. Prove que, dado um inteiro positivo n, existe uma progressão aritméticacrescente formada por n inteiros positivos cujas somas dos dígitos tambémformam uma progressão aritmética crescente, mas não existe uma progressãoaritmética infinita de inteiros positivos cujas somas dos dígitos formem umaprogressão aritmética crescente.

SOLUÇÃO DE JOSÉ DE ALMEIDA PANTERA (RIO DE JANEIRO – RJ)Vamos mostrar, por indução em k, que para quaisquer inteiros positivos 2k ≥ e Ne todo 0ε > existe uma progressão aritmética crescente de inteiros positivos( )j N j k Nx − ≤ ≤ + tal que se ( )js x é a soma dos dígitos de jx então 1( ( ))j N js x − ≤ ≤ é

constante, 1( ( ))j j ks x ≤ ≤ é uma progressão aritmética crescente de razão r e

Page 112: Eureka 2003

Sociedade Brasileira de Matemática

EUREKA! N°17, 2003

50

1( ) ( )k i k is x s x rε+ + −− < para 1 i N≤ ≤ . De fato, para k = 2, podemos tomar2

1 2 110 10 , 10 1,m m mx x x= − − = − onde m é grande o suficiente para que m N≥ (e

logo 10 m N> ) e 1

.m

ε<

De fato, ( ) 9is x m= , para 2 10 1m i− ≤ ≤ [pois, para 1 10 , (10 1)m ma a≤ ≤ − =( 1) 10 10 1 ( 1),m ma a− ⋅ + − − − donde ( (10 1)) ( 1) 9 ( 1) 9 )ms a s a m s a m⋅ − = − + − − = ,

( (10 1))ms a ⋅ − = 2( ) 18s x m= e, para j > 2, digamos j = 2 + a,

2 210 10 1 10 ( 1) 10 10 1m m m m mjx a a a a= ⋅ + − − = + − ⋅ + − − , e, como

1 10 ,ma N+ ≤ < 1( ) ( ) 1 ( ) 9 ( 1) (1 ( 1) 9 ( ))j js x s x s a m s a s a m s a+ − = + + − + − + − + − =

2 ( ) ( 1) ( 1) 9 (9 ).s a s a s a m mε= − − − + < < ⋅Vamos agora conseguir uma tal progressão aritmética com k = 1 elementos. Para

isso, seja 1 1( )j N j k Nx − − ≤ ≤ + + uma progressão como acima associada a , 1, .3

k Nε +

Podemos supor que N k≥ e 1

2N

ε< . Seja � um inteiro tal que 110 k Nx + +>� .

Considere a seqüência 1( )j N j k Ny − ≤ ≤ + + , 110 ,j j j ky x t − += + ⋅� onde

210 1 (10 1)m mst s= − + − e 1( ( ) ( ))

(1 )9

k kr s x s xm Nε+− −

= ≥ − , onde

2 1( ) ( )r s x s x= − (note que m é inteiro pois ( ) (mod 9)j js x x≡ para todo j).

Assim, 1( ) ( ) ( ).j j j ks y s x s t − −= + Temos então 1( ( ))j N js y − ≤ ≤ constante e

1( ) ( ) 9j js y s y m+ − = , para 1 .j k≤ ≤ De fato, para 1 1,j k≤ ≤ − isso segue do fato

análogo para 1( ) ,j j kx ≤ ≤ e de 1 1( ( ))j k j ks t − − ≤ ≤ ser constante, e para j = k,

1 1( ) ( ) 9 ( )j k j k k ks t s t m r x x− − − +− = = − − , donde 1 2 1( ) ( ) ( ) ( ).k ks y s y r s x s x+ − = = − O

fato de termos 1( ) ( )k i k is y s y rε+ + +− < para 1 i N≤ ≤ segue dos fatos análogos para

1( )k is x + + e ( ).is ySuponha agora que exista uma progressão aritmética infinita de inteiros positivos

1( )j jx ≥ cujas somas dos dígitos 1( ( )j js x ≥ ) formam uma progressão aritmética

crescente. Tome um inteiro positivo k tal que 2 110 ,k x x> > e note que

1

11 2 1 1 2 1 1 2 110 1 10 1

( ) ( 10 ( )) ( ) ( ) ( 10 ( )) ( )k k

k ks x s x x x s x s x x s x x x s x ++

+ += + − = + − = + − = ,

absurdo.

Page 113: Eureka 2003

Sociedade Brasileira de Matemática

EUREKA! N°17, 2003

51

74. Ache todas as funções �� →:f tais que:

,cos)(2)()( yxfyxfyxf ⋅=−++ ., �∈∀ yx

SOLUÇÃO DE MARCÍLIO MIRANDA DE CARVALHO (TERESINA – PI)

Faça 2

yπ= e ( ) ( ) 0

2x t f t f t

π π= + ⇒ + + = (I)

Faça x = 0 e y = t ⇒ f(t) + f(–t) = 2a cos t, onde a = f(0). (II)

Faça 2

xπ= e ( ) ( ) 2 cos 2 sen

2 2y t f t f t b t b t

π ππ = + ⇒ + + − = + = − , (III)

onde .2

b fπ =

Somando (I) com (II) e subtraindo de (III) chegamos a f(t) = a cost + b sent,.t∀ ∈� É fácil verificar, apenas substituindo, que todas as funções f(t) desta

forma funcionam.

75. Seja Tn um triângulo retângulo cujos lados medem 2 4 4(4 , 4 1, 4 1)n n n⋅ ⋅ − ⋅ + ,

onde n é um número inteiro positivo. Seja nα a medida do ângulo oposto ao lado

de medida 24 n⋅ . Mostre que, se n varia dentro dos inteiros positivos,

.90... 0321 =+++ ααα

SOLUÇÃO DE ANTONIO CAMINHA MUNIZ NETO (FORTALEZA – CE)Seja Tn um triângulo retângulo cujos lados medem 4n2, 4n4 – 1 e 4n4 + 1, onde n éum número inteiro positivo. Seja αn a medida, em graus, do ângulo oposto ao ladode medida 4n2. Mostre que α1 + α2 + α3 + .... = 90°.

Solução: Sabemos que tg αn = 14

44

2

−n

n. Portanto, sendo αn = 2βn temos

14

4

1

24

2

2 −=

− n

n

tg

tg

n

n

ββ

.

Resolvendo a equação acima para tg βn obtemos tg βn = 22

1

n para todo n ≥ 1.

Assim, basta provar que β1 + β2 + β3 + .... = 45°. Sendo bn = tg (β1 + β2 + ... + βn)para n ≥ 1, temos então de provar que bn → 1 quando n → + ∞.

Page 114: Eureka 2003

Sociedade Brasileira de Matemática

EUREKA! N°17, 2003

52

Temos b1 = tg β1 = 2

1 e, pela fórmula para a tangente da soma,

2

2

1 22( 1)1

1 21 2( 1)

2( 1) 1.

1 2( 1)1 n

n nn n nn b

n n nn

bb tg n bb

b tg n b

ββ

+++

+ +

++ + += = =− + −−

Provemos, por indução sobre k ≥ 1, que (*) 1+

=k

kbk , o que terminará a

demonstração. A relação (*) é trivialmente verdadeira para k = 1. Suponha que jáprovamos que ela é verdadeira para 1 ≤ k ≤ n, onde n ≥ 1 é um inteiro. Então

( )( )

.2

1

)2)(122(

)1)(122(

)1(2

)1](1)1(2[

)1(2

1)1(2

2

2

31

21

2

1

++=

++++++=

−++++=

−+++

=+

++

n

n

nnn

nnn

nn

nnn

n

nb

nn

nn

n

76. Mostre que um polígono qualquer pode ser recortado e os recortesreorganizados, sem superposição, de tal jeito que formem um quadrado.

SOLUÇÃO DE ANDERSON TORRES (SÃO PAULO – SP)Este problema é tão legal que dá para generalizar! Vamos demonstrar que se doispolígonos têm mesma área podemos fatiar um deles e reorganizar as fatias demodo a produzir um polígono congruente ao segundo.Lema 1: Um retângulo pode ser convertido em um novo retângulo do qualconhecemos um dos lados (por exemplo, qualquer retângulo de área x pode serdecomposto para formar um retângulo em que um dos lados mede 1 (e o outromedirá x neste caso).Prova: Basta exibir uma maneira de fazer isto. Veja este exemplo:

K

D

I

2

A B

J

C

1

3

H

E

N

2'

3'

F

M

L

1'

G

Page 115: Eureka 2003

Sociedade Brasileira de Matemática

EUREKA! N°17, 2003

53

1) Faça com que as bases dos retângulos estejam alinhadas. Prolongue o lado HGaté cortar o outro retângulo no ponto I.

2) Trace DJ//IB. Seja KI //AB e K ∈ DJ. Agora é só deslizar o triângulo DCJ atéDC se alinhar com IK. E depois encaixa DIK no espaço abaixo, em MLF.

E acabou!

Esta construção só vale se 2.HE ≥ DI. Caso não seja possível, basta fatiar damaneira mais primitiva: fatia em partes iguais até cair no caso anterior:

Neste caso nem precisou!

Agora é só escrever. Dado um polígono qualquer, recortamos o desdito emtriângulos quaisquer. Cada um deles pode ser fatiado para virar um paralelogramo(parte pela base média) e cada paralelogramo pode ser fatiado nun retângulo, é sócortar em uma altura. Podemos transformar cada um destes retângulos em fatiasde lado 1, empilhá-los pelo lado comum e transformar isto tudo num quadrado(oras, um quadrado é um retângulo equilátero) E fim!

77. Prove que as distâncias entre um ponto sobre uma circunferência e os quatrovértices de um quadrado nesta inscrita não podem ser todos números racionais.

SOLUÇÃO DE CLAUDIO BUFFARA (SÃO PAULO – SP)Seja o quadrado inscrito ABCD de lado L e suponhamos, sem perda degeneralidade, que o ponto P se encontra no arco AB.

Se P coincide com B, então m(PA) = m(PC) = L e m(PD) = L ⋅ 2 . Assim, se L éracional, então m(PD) é irracional e se L é irracional, então m(PA) = m(PC) éirracional. De forma análoga pode-se provar que se P coincide com A, entãom(PB), m(PD) ou m(PD) será irracional.Suponhamos agora que P não coincide com A nem com B e que m(PA), m(PB),m(PC) e m(PD) sejam todos racionais.O quadrilátero PBCD é inscritível. Assim, pelo Teorema de Ptolomeu, teremos:m(PD) ⋅ m(BC) + m(PB) ⋅ m(CD) = m(BD) ⋅ m(PC).

Page 116: Eureka 2003

Sociedade Brasileira de Matemática

EUREKA! N°17, 2003

54

Mas m(BC) = m(CD) = L e m(BD) = L ⋅ 2 . Portanto:

m(PD) + m(PB) = m(PC) ⋅ 2 .Mas:m(PD) e m(PB) são racionais ⇒ m(PD) + m(PB) é racional e

m(PC) é racional ⇒ m(PC) ⋅ 2 é irracional

Ou seja, um número racional é igual a um número irracional.

Temos, portanto, uma contradição, a qual ocorreu em virtude da hipótese feitainicialmente de serem m(PA), m(PB), m(PC), m(PD) todos racionais.

Conclusão: pelo menos um destes quatro segmentos tem de ter comprimentoirracional.

Observação: se tomarmos o quadrilátero inscritível PADB, o Teorema dePtolomeu nos dará:

m(PA)⋅ m(BD) + m(PB) ⋅ m(AD) = m(PD) ⋅ m(AB), ou seja:

m(PA) ⋅ 2 + m(PB) = m(PD), e mais uma vez cairemos na contradição de ter umnúmero racional igual a um número irracional.

78. Seja ABCD um quadrilátero convexo não trapézio, de diagonais AC e BDiguais. Tomamos sobre os lados AB e CD, respectivamente, pontos P e Q tais que:

BC

AD

QC

DQ

PB

AP ==

Mostre que os pontos P e Q são colinares com o ponto de interseção dasmediatrizes dos lados AD e BC.

SOLUÇÃO DE MARCOS FRANCISCO FERREIRA MARTINELLI (RIO DE JANEIRO – RJ)Considere o quadrilátero ABCD AC = BD, onde O é o ponto de encontro dasdiagonais e O' o ponto de encontro das mediatrizes de AD e BC.

Page 117: Eureka 2003

Sociedade Brasileira de Matemática

EUREKA! N°17, 2003

55

A

P'

D

Q

O

B

β β O'

C

Provarei que os quadriláteros AOO'D e BOO'C são inscritíveis (i).

Seja ˆ 'ADOα = ∆ . Como DO' = O'A ⇒ ˆ 'DAO α∆ =Observe que os 'ACO∆ e 'BDO∆ são congruentes (L.L.L), mais uma vez porque

' 'O B O C= e, do enunciado, AC = BD.

⇒ ˆ ˆˆ ˆ' ' e ' ' )O AO ODO DBO ACO i= = ⇒ está provado.

Como o quadrilátero AOO'D é inscritível, ˆ ˆ' 'DOO DAO α= =E ainda, ˆ ˆ' 180 ' 180 (180 )O OC O OA α α= ° − = ° − ° − =

Como o quadrilátero BOO'C é inscritível ˆ ˆˆ, ' ' 'O OC O BC O CBα= = = (ii)De ii), observe que o ' 'AO D O BC∆ ∆ ⇒�

' ' '

'

AO BO AD AO

AD BC BC BO= ⇒ = (iii)

Seja Q ∈ CD, DQ AD

QC BC= , bem como P' a interseção de QO' com AB. Se

provarmos que '

, teremos ' .'

AP ADP P

P B BC= ≡

Como, por hipótese, DQ AD

QC BC= e de (iii), temos

' '

' '

DQ AO DO

QC BO CO= = , e conclui-se

que O'Q é bissetriz do ' .DO C∆

Page 118: Eureka 2003

Sociedade Brasileira de Matemática

EUREKA! N°17, 2003

56

Seja ˆ ˆ' ' ' ' ' 180 ( 180 2 ) 2DO Q Q O C AO Pβ β α α β= = ⇒ = ° − + ° − = − e' ' 180 ( 180 2 ) 2P O B β α α β= ° − + ° − = −

' 'O P⇒ é bissetriz do ' '

'' '

AP AO ADAO B

P B O B BC∆ ⇒ = = ⇒ está provado que ' .P P≡

Portanto, P, O'e Q estão sobre uma mesma reta. (c.q.d.)

79. Temos uma fileira infinita de copos, cada um deles associado a um inteiro k, eum número finito de pedras distribuídas de alguma maneira por esses copos. Se hápelo menos duas pedras no copo k podemos pular uma pedra para o copo k – 1e outra para o copo k + 1.Prove que fazendo movimentos desse tipo um número suficientemente grande devezes, chega-se necessariamente a uma situação onde não é possível fazer nenhummovimento desse tipo (i.e., onde há no máximo uma pedra em cada copo), e que aconfiguração final não depende da escolha dos movimentos durante o processo.

SOLUÇÃO:Sejam 1,..., nx x ∈� as posições das n pedras. O número de pedras é sempre n, e em

um movimento trocamos duas pedras na posição k (digamos com )i jx x k= = por

1ix k= − e 1.jx k= + Temos então que 1

n

ii

x=∑ permanece constante e

2

1

n

ii

x=∑ aumenta a cada movimento, pois ( ) ( )2 2 2 21 1 2 2 2 .k k k k+ + − = + > Seja

agora m o maior número de copos vazios entre dois copos ocupados. Então, se0m ≠ , m não aumenta em nenhum movimento e, se m = 0, após um movimento m

passa a ser no máximo 1. Assim, a distância entre dois copos ocupados

consecutivos fica limitada, e como o centro de gravidade 1

1 n

ii

xn =∑ das pedras é

constante, a "energia" 2

1

n

ii

x=∑ também fica limitada, e como sempre aumenta, em

algum momento não será mais possível fazer nenhum movimento. O número de

movimentos é limitado por 2 21

1

( , ..., ) ,n

nj

f x x j=

= ∑� onde max{2, }r=� e,

se 1 2 ... nx x x≤ ≤ ≤ , 12max( ) 1.i i

i nr x x −≤ ≤

= − + Para cada 1 , . . . , nx x , seja

1 1( ,..., ) ( ,..., )n ng x x f x x≤ o número máximo possível de movimentos a partir da

posição inicial 1 2, ,..., .nx x x

Page 119: Eureka 2003

Sociedade Brasileira de Matemática

EUREKA! N°17, 2003

57

Suponha agora que exista uma posição inicial a partir da qual seja possível chegara duas posições finais distintas. Seja 1,..., nx x uma tal posição com

1( ,..., )ng x x mínimo. Esses dois jeitos de chegar em posições finais diferentes nãopodem começar com o mesmo movimento, pois senão, após esse movimento, ovalor de g diminui, e a posição final passa a ser única pela minimalidade de

1( ,..., )ng x x , absurdo.Agora, se os movimentos iniciais das duas seqüências de movimentos que levam aposições finais diferentes são feitos nas posições k e � , após cada um dessesmovimentos o valor de g diminui e as posições finais ficam determinadas.Por outro lado, se nos dois primeiros lances mexemos primeiro no copo k e depoisno copo � chegamos à mesma configuração que se primeiro mexermos no copo �e depois no copo k (de fato, se inicialmente i jx x k= = e r sx x= = � , chegaremos

após esses dois lances, em qualquer ordem, em 1, 1, 1i j rx k x k x= − = + = −� e

1sx = +� ) , donde as posições finais são iguais, absurdo.

80. Sejam 2

51+=α , { } , *A n nα= ∈ � e { }2 , *B n nα = ∈ � . Prove que

=∩ BA ∅ e *�=∪ BA .

Observação: x é o inteiro tal que .1+<≤ xxx

SOLUÇÃO DE RODRIGO VILLARD MILET (RIO DE JANEIRO – RJ)Temos que α² = α + 1.

1ªparte: A ∩ B = ∅Suponha o contrário, ou seja, que existem m e n naturais tais que

knm == ][][ 2αα . Daí temos que k < αm < k + 1 e k < α²n < k + 1

(a desigualdade é estrita, pois α é irracional), portanto :

1111

1 2+<+<⇒+<=+<

++

knmkk

nm

k

nm

αα

o que é uma contradição, já que k, m e n são naturais.

Page 120: Eureka 2003

Sociedade Brasileira de Matemática

EUREKA! N°17, 2003

58

2ªparte: A ∪ B = �*Suponha que exista um natural h que não está nem em A nem em B. Então existemnaturais m e n tais que αm < h < h + 1 < α (m + 1) e α²n < h < h + 1< α²(n + 1).Logo :

211

21

112

++<+<<+⇒+

++<=+<+nmhhnm

h

nm

h

nm

αα

o que é uma contradição, já que m + n + 1 é o único natural entre m + n em + n + 2.

Enviaram soluções de problemas anteriores os seguintes leitores da EUREKA!

Carlos Alberto da Silva Victor Nilópolis – RJDiêgo Veloso Uchôa Teresina – PIDiogo Diniz Pereira da Silva e Silva Enviado via correio eletrônicoHelder Oliveira de Castro Mogi das Cruzes – SPJoão Fernandes de Moura Niterói – RJLeno Silva Rocha Goiânia – GOMurilo Rebouças Fernandes de Lima Goiânia – GO

Page 121: Eureka 2003

Sociedade Brasileira de Matemática

EUREKA! N°17, 2003

59

PROBLEMAS PROPOSTOS

� Convidamos o leitor a enviar soluções dos problemas propostos e sugestões de novosproblemas para os próximos números.

81) Num triângulo isósceles ABC com AB = BC, temos AC = BH, onde BH é aaltura relativa ao lado AC. Traçamos uma reta BD que corta o prolongamentoda reta AC em D de tal forma que os raios dos círculos inscritos nos triângulos

ABC e CBD são iguais. Determine o ângulo ˆABD.

82) a) Demonstre a identidade

1

10

(2 )cos( ) cos(2 ) cos(4 )...cos(2 ) cos(2 )

2 ( )

nnn j

nj

sen

sen

αα α α α αα

+

+=

⋅ ⋅ = =⋅∑

b) Prove que 2

0

1 1 1 1 1 1 1 1 1 2... cos .

2 2 2 2 2 2 2 2 2 2 jj

ππ

+=

⋅ + ⋅ + + = = ∏

83) Seja {0,1,2,3,...}.=�Determine quantas funções :f →� � satisfazem f(2003) = 2003, f(n) ≤ 2003

para todo n ≤ 2003 e f(m + f(n)) = f(f(m)) + f(n), para todo m, n ∈ �.

84) Prove que se * {1,2,3,...}A⊂ =� é um conjunto não-vazio tal que

4n A n A∈ ⇒ ∈ e n A ∈ então *A=� .

Obs. x é o único inteiro tal que 1x x x− < ≤

85) Mostre que todo triângulo pode ser dividido em 9 pentágonos convexos deáreas iguais.

86) Encontre todas as triplas de inteiros positivos (a, m, n) tais que 1ma + divide

( 1) .na+

Page 122: Eureka 2003

Sociedade Brasileira de Matemática

EUREKA! N°17, 2003

60

87) Seja a(1) = 1 e, para cada inteiro n ≥ 2, a(n) igual ao menor inteiro positivo

que não pertence a {a( j), j < n} tal que 1

( )n

j

a j=

∑ seja múltiplo de n. Prove que

( ( ))a a n n= para todo inteiro positivo n.

88) Prove que se r∈� e cos( )r π⋅ ∈� então 1 1

cos( ) 1, ,0, ,1 .2 2

r π ⋅ ∈ − −

���� �����

� ������ ������ ���� � ���� ��� �� �� ������� ��������

�������� ���� � ������

�� � �� � �� � ��� ������� ��� ��� ����� � !�������

��������� ���� "���!�� ��� ��� � ������ ���������� ���

��# ��� ���$ � ����� � ��� ����� ��� ��� �����

��������% &�� �'��� �� (����������� �� )������

���$������ !� �� ������ !������� ��!����

�*+�,,�� � -./*.*/0� � -+10*1*�� � ��*-.*1/�

2 � ������ �� ���� ����3

Problema 81 proposto por Geraldo Perlino (Itapecerica da Serra – SP); Problema 82 propostopor Clodoaldo Lessa (Mogi das Cruzes – SP); Problema 83 adaptado de um problemaproposto por Gibran M. de Souza (Natal – RN); Problema 84 proposto por Anderson Torres(São Paulo – SP); Problema 85 proposto por Gibran M. de Souza (Natal – RN); Problema 88proposto por C.G. Moreira e José Paulo Carneiro (Rio de Janeiro – RJ).

Page 123: Eureka 2003

Sociedade Brasileira de Matemática

EUREKA! N°17, 2003

61

AGENDA OLÍMPICA

XXV OLIMPÍADA BRASILEIRA DE MATEMÁTICA

NÍVEIS 1, 2 e 3Primeira Fase – Sábado, 7 de junho de 2003

Segunda Fase – Sábado, 13 de setembro de 2003Terceira Fase – Sábado, 18 de outubro de 2003 (níveis 1, 2 e 3)

Domingo, 19 de outubro de 2003 (níveis 2 e 3 - segundo dia de prova).

NÍVEL UNIVERSITÁRIOPrimeira Fase – Sábado, 13 de setembro de 2003

Segunda Fase – Sábado, 18 e Domingo, 19 de outubro de 2003

IX OLIMPÍADA DE MAIO10 de maio de 2003

XIV OLIMPÍADA DE MATEMÁTICA DO CONE SUL23 a 30 de maio de 2003

Ica – Peru

♦XLIV OLIMPÍADA INTERNACIONAL DE MATEMÁTICA

07 a 19 de julho de 2003Tóquio – Japão

X OLIMPÍADA INTERNACIONAL DE MATEMÁTICA UNIVERSITÁRIA25 a 31 de julho de 2003

Universidade Babes-Bolyai, Cluj-Napoca, Romênia

♦XVIII OLIMPÍADA IBEROAMERICANA DE MATEMÁTICA

13 a 20 de setembro de 2003Argentina

♦VI OLIMPÍADA IBEROAMERICANA DE MATEMÁTICA UNIVERSITÁRIA

8 de novembro de 2003

♦♦♦

Page 124: Eureka 2003

Sociedade Brasileira de Matemática

EUREKA! N°17, 2003

62

COORDENADORES REGIONAIS

Alberto Hassen Raad (UFJF) Juiz de Fora – MG

Amarísio da Silva Araújo (UFV) Viçosa – MG

Ana Paula Bernardi da Silva (Universidade Católica de Brasília) Brasília – DF

Benedito Tadeu Vasconcelos Freire (UFRN) Natal – RN

Carlos Frederico Borges Palmeira (PUC-Rio) Rio de Janeiro – RJ

Claus Haetinger (UNIVATES) Lajeado – RS

Cleonor Crescêncio das Neves (UTAM) Manaus – AM

Élio Mega (Colégio Etapa) São Paulo – SP

Florêncio Ferreira Guimarães Filho (UFES) Vitória – ES

Gil Cunha Gomes Filho (Colégio ACAE) Volta Redonda – RJ

Ronaldo Alves Garcia (UFGO) Goiânia – GO

Reginaldo de Lima Pereira (Escola Técnica Federal de Roraima) Boa Vista – RR

Ivanilde Fernandes Saad (UC. Dom Bosco) Campo Grande– MS

Jacqueline Fabiola Rojas Arancibia (UFPB) João Pessoa – PB

Janice T. Reichert (UNOCHAPECÓ) Chapecó – SC

João Benício de Melo Neto (UFPI) Teresina – PI

João Francisco Melo Libonati (Grupo Educacional Ideal) Belém – PA

José Carlos dos Santos Rodrigues (Unespar) Campo Mourão – PR

José Cloves Saraiva (UFMA) São Luis – MA

José Gaspar Ruas Filho (ICMC-USP) São Carlos – SP

José Luiz Rosas Pinho (UFSC) Florianópolis – SC

José Vieira Alves (UFPB) Campina Grande – PB

Licio Hernandes Bezerra (UFSC) Florianópolis – SC

Luzinalva Miranda de Amorim (UFBA) Salvador – BA

Mário Rocha Retamoso (UFRG) Rio Grande – RS

Marcelo Rufino de Oliveira (Grupo Educacional Ideal) Belém – PA

Marcelo Mendes (Colégio Farias Brito, Pré-vestibular) Fortaleza – CE

Pablo Rodrigo Ganassim (Liceu Terras do Engenho) Piracicaba – SP

Ramón Mendoza (UFPE) Recife – PE

Raúl Cintra de Negreiros Ribeiro (Colégio Anglo) Atibaia – SP

Reinaldo Gen Ichiro Arakaki (INPE) SJ dos Campos – SP

Ricardo Amorim (Centro Educacional Logos) Nova Iguaçu – RJ

Sérgio Cláudio Ramos (IM-UFRGS) Porto Alegre – RS

Tadeu Ferreira Gomes (UEBA) Juazeiro – BA

Tomás Menéndez Rodrigues (U. Federal de Rondônia) Porto Velho – RO

Valdenberg Araújo da Silva (U. Federal de Sergipe) São Cristovão – SE

Valdeni Soliani Franco (U. Estadual de Maringá) Maringá – PR

Vânia Cristina Silva Rodrigues (U. Metodista de SP) S.B. do Campo – SP

Wagner Pereira Lopes (CEFET – GO) Jataí – GO

Page 125: Eureka 2003

CONTEÚDO AOS LEITORES 2 XIV OLIMPÍADA DE MATEMÁTICA DO CONE SUL 3 Enunciados e Soluções XLIV OLIMPÍADA INTERNACIONAL DE MATEMÁTICA 13 Enunciados e Soluções X OLIMPÍADA INTERNACIONAL DE MATEMÁTICA PARA ESTUDANTES UNIVERSITÁRIOS 23 Enunciados e Soluções XVIII OLIMPÍADA IBERO-AMERICANA DE MATEMÁTICA 34 Enunciados e Soluções ARTIGOS A DESIGUALDADE DE ERDÖS - MORDELL 42 Anderson Torres COMO É QUE FAZ? 53 SOLUÇÕES DE PROBLEMAS PROPOSTOS 54 PROBLEMAS PROPOSTOS 58 COORDENADORES REGIONAIS 62

Page 126: Eureka 2003

Sociedade Brasileira de Matemática

EUREKA! N°18, 2003

2

AOS LEITORES

Chegamos à última edição de 2003 publicando soluções de diversas

olimpíadas internacionais: a Olimpíada de Matemática do Cone Sul, a Olimpíada Internacional de Matemática, a Olimpíada Ibero-americana e a Olimpíada Internacional de Matemática para Estudantes Universitários, da qual participamos pela primeira vez. Publicamos também um artigo sobre a desigualdade de Erdös Mordell, que ajuda a resolver um dos problemas mais difíceis que já caíram na IMO.

Estamos propondo sete problemas bacanas na seção de problemas propostos, que se somam aos 6 problemas da última edição dos quais ainda não recebemos solução. Haverá portanto bastante diversão para o começo de 2004…

Em 2004 a Olimpíada Internacional de Matemática será realizada na

Grécia, assim como as Olimpíadas. Vamos torcer para o Brasil trazer belas medalhas nos dois casos…

Abraços e feliz ano novo a todos,

Os editores

Page 127: Eureka 2003

Sociedade Brasileira de Matemática

EUREKA! N°18, 2003

3

XIV OLIMPÍADA DE MATEMÁTICA DO CONE SUL Enunciados e Soluções

PROBLEMA 1 Em um torneio de futebol entre quatro equipes, A, B, C e D, cada equipe joga com cada uma das outras exatamente uma vez. a) Decidir se é possível que, ao finalizar o torneio, as quantidades de gols marcados e sofridos pelas equipes sejam:

A B C D Gols marcados 1 3 6 7 Gols sofridos 4 4 4 5

Se a resposta é afirmativa, dê um exemplo com os resultados das seis partidas; em caso contrário, justifique.

b) Decidir se é possível que, ao finalizar o torneio, as quantidades de gols marcados e sofridos pelas equipes sejam:

A B C D Gols marcados 1 3 6 13 Gols sofridos 4 4 4 11

Se a resposta é afirmativa, dê um exemplo com os resultados das seis partidas; em caso contrário, justifique. SOLUÇÃO DE FÁBIO DIAS MOREIRA (RIO DE JANEIRO – RJ) a) A configuração apresentada é possível. Por exemplo,

A × B 1 × 1 A × C 0 × 0 A × D 0 × 3 B × C 0 × 3 B × D 2 × 0 C × D 3 × 4

Page 128: Eureka 2003

Sociedade Brasileira de Matemática

EUREKA! N°18, 2003

4

b) Como D não joga contra si mesmo, todos os gols que marca são contra A, B e C. Logo todo gol marcado por D aumenta em um a contagem de gols marcados por D e aumenta em um a soma do número de gols sofridos por A, B e C. Mas como D marcou treze gols, A, B e C deveriam ter sofrido pelo menos treze gols no total, mas sofreram apenas 12, absurdo! Logo a situação apresentada é impossível.

PROBLEMA 2 Considere a seqüência {an} definida da seguinte maneira:

a1 = 1 a2 = 3 an+2 = 2an+1 an

+ 1, para todo inteiro n ≥ 1.

Provar que a máxima potência de 2 que divide a4006 − a4005 é 22003. SOLUÇÃO DE RODRIGO AGUIAR PINHEIRO (FORTALEZA – CE) Seja 1; e 2n n nA a a n n−= − ∈ ≥

Indução: *n∀ ∈ , temos 2n sendo a maior potência de 2 que divide 2nA e 1

2 12nnA++

Base de Indução: 3 2 1 2 1 2 11: 2 1 2 3 1 1 7 2n a a a A a a⋅= = ⋅ + = ⋅ ⋅ + = ⇒ = − = e

2 1 1 3 2 4A a a⋅ + = − =

Temos que 12 12 A ⋅ mas 2

2 12 não divide A ⋅ e 1 12 1 12 .A+⋅ + OK!!

4 3 22 : 2 1 2 7 3 1 43n a a a= = ⋅ ⋅ + = ⋅ ⋅ + =

5 4 32 1 2 43 7 1 603a a a= ⋅ ⋅ + = ⋅ ⋅ + =

4 43 7 36A = − = e 5 603 43 560A = − = 236 2 9,= ⋅ logo 2

2 22 ,A ⋅ mas 32 não divide 2 2A ⋅ 4560 2 7 5,= ⋅ ⋅ logo 2 1

2 2 12 A+⋅ + .

Passo Indutivo: Supondo que para todos os naturais menores ou iguais a k, valha a sentença da indução, provaremos que vale para k + 1.

2 2 2 2 2 1 2 1 2 2 2 1(2 1) (2 1)k k k k k k kA a a a a a a+ + + + −= − = ⋅ ⋅ + − ⋅ ⋅ + = 2 2 1 2 12 ( )k k ka a a+ −− = 2 2 1 2 2 2 12 ( )k k k k ka a a a a+ −− + −

2 2 2 2 1 22 ( )k k k kA a A A+ +⇔ = +

Page 129: Eureka 2003

Sociedade Brasileira de Matemática

EUREKA! N°18, 2003

5

Seja 2 2 22 ,kk k kA x x= ⋅ ímpar (hipótese de indução) e 1

2 1 2 12 .kk kA x++ += ⋅

Temos: 2 12 2 2 2 1 2 2 2 1 2

ímpar ímpar

2 (2 2 ) 2 (2 )k k kk k k k k k kA a x x a x x++ + += ⋅ ⋅ + = ⋅ +

Portanto 12 2

2kk

A++ , mas 22k+ não divide 2 2kA +

1 12 3 2 1 2 2 2 1 2 1 2 2 2 12 ( ) 2 (2 2 )k k

k k k k k k kA a A A a x x+ ++ + + + + + += + = ⋅ ⋅ + ⋅

22 1 2 2 2 12 ( )k

k k ka x x++ + += + , portanto 1 1

2( 1) 12k

kA+ +

+ +

Para k = 2003, temos que 20034006 40054006

2 ,A a a= − mas 20042 não divide 4006A . PROBLEMA 3 Seja ABC um triângulo acutângulo tal que o ângulo B mede 60o . A circunferência de diâmetro AC intersecta as bissetrizes internas de A e C nos pontos M e N respectivamente (M ≠ A, N ≠ C). A bissetriz interna do ângulo B intersecta MN e AC nos pontos R e S, respectivamente. Demonstrar que BR RS≤ . SOLUÇÃO DE THIAGO COSTA LEITE SANTOS (SÃO PAULO – SP)

α

30°

α – 30°

T S

β

C

30°

β

I

M K

30° 30°

A

B

R N α W

Page 130: Eureka 2003

Sociedade Brasileira de Matemática

EUREKA! N°18, 2003

6

Digamos que 2 2BAC BCAα β∠ = ≥∠ = , Sem perda de generalidade 2 2 120 60α β α β+ = °⇒ + = ° .

N está dentro do triângulo, pois como AC é diâmetro 90ANC AMC⇒∠ =∠ = ° 90NAC β⇒∠ = °− ⇒

90 30NAM β α⇒∠ = − − = ° e 30NAB α∠ = − °. De modo análogo M está fora do triângulo. Assim, 90 90 2 30MCA MCBα α β β∠ = °− ⇒∠ = °− − = °− Como KMI NCA∠ =∠ , pois ANMC é inscritível, KMI KCI IKMCβ∠ = =∠ ⇒ é inscritível 90 .IKC IMC⇒∠ =∠ = ° Como KMI β∠ = e 30 , 60KCM CKM NKBβ∠ = °− ∠ = ° =∠ e 60 .KWB∠ = ° Como I é o incentro,

IK IT r= = ⇒ 1sen30 2 22

KI BI KI rBI

°= = ⇒ = =

(r é o inraio). O triângulo KWB é equilátero, pois provamos que 60 .BWK BKW WBK∠ =∠ =∠ = ° Como BR é bissetriz de ,WBK BR WK∠ ⊥ .

No triângulo RKI: 1sen30 .2 2 2

RI KI rRIKI

°= = ⇒ = =

322 2r rBR BI RI r⇒ = − = − =

,SI IT r⇒ ≥ = pois 90ITS∠ = ° 3

2 2 2r r rRS RI SI SI r BR RS BR⇒ = + = + ≥ + = = ⇒ ≥ .

PROBLEMA 4 No triângulo acutângulo ABC, os pontos H, G e M encontram-se sobre o lado BC, de modo que AH, AG e AM são altura, bissetriz e mediana do triângulo, respectivamente. Sabe-se que HG = GM, AB = 10 e AC = 14. Determinar a área do triângulo ABC.

Page 131: Eureka 2003

Sociedade Brasileira de Matemática

EUREKA! N°18, 2003

7

SOLUÇÃO DE RODRIGO AGUIAR PINHEIRO (FORTALEZA – CE)

B x H G M

A

z 14

C y k k

10

Seja BH = x, HG = GM = k, MG = y e AH = z. Como AM é mediana, BM = MC ⇒ BH + HG + GM = MC ⇔ x + 2k = y (I) Apliquemos o teorema da bissetriz interna, considerando a bissetriz AG:

10 5( ) 7( )14

AB BG x k y k x kAC GC y k

+= ⇔ = ⇔ + = +

+

5 7 2y x k⇔ = + (II)

(I) em (II): 35 6 2 62

y x x k x y y x= + + = + ⇔ = (III)

(III) em (I): 3 12 2 42 2

x k y x x k x k+ = = ⇔ = ⇔ =

(III): 3 3(4 ) 6 .2 2

y x k y k= = ⇔ =

Apliquemos o teorema de Pitágoras no :AHC∆ 2 2 2 2 2 2(8 ) 14AC AH HC z k= + ⇔ + =

Teorema de Pitágoras no :AHB∆ 2 2 2 2 2 2(4 ) 10AB AH HB z k= + ⇔ + =

Temos: 2 2

2 2 2 22 2

64 19664 ( 16 ) 196 100

16 100z k

z k z kz k+ =

⇒ + − + = −+ =

2 248 96 2 2k k k⇔ = ⇔ = ⇔ =

2 2 2 216 100 16 2 100 68 68;z k z z z+ = ⇔ + ⋅ = ⇔ = ⇔ = Área 68 12 2 12 34

2⋅ ⋅

= = .

Page 132: Eureka 2003

Sociedade Brasileira de Matemática

EUREKA! N°18, 2003

8

PROBLEMA 5 Seja 13 += kn , onde k é um inteiro, 1≥k . Constrói-se um arranjo triangular de lado n formado por círculos de mesmo raio como o mostrado na figura para

7=n .

Determinar, para cada k, o maior número de círculos que podem ser coloridos de vermelho de tal modo que não existam dois círculos vermelhos tangentes entre si. ADAPTAÇÃO DA SOLUÇÃO DE THIAGO COSTA LEITE SANTOS (SÃO PAULO – SP) Obviamente 06 1n k= + ou 06 4n k= + , quando k = 2k0 ou 02 1k k= + , respectivamente. Observe que se tivermos seis círculos do seguinte formato:

nós só podemos pintar no máximo 2 bolas; caso pintemos 3, haverá duas bolas pintadas tangentes, o que é absurdo!

Primeiro caso: 0 02 6 1k k n k= ⇒ = + ⇒ Denote por ak o maior número de círculos que podemos pintar. No triângulo n × n pegue as 3 últimas linhas:

Primeira linha Segunda linha

3k +1

3k +2

3k +3

3(k+1)+1

… …

3k +2

3k +3

3k + 4

… …

Page 133: Eureka 2003

Sociedade Brasileira de Matemática

EUREKA! N°18, 2003

9

Dividimos aqueles círculos em peças de 6 círculos e uma peça de 3 círculos da seguinte forma:

3k +2

3k +3

3k + 4 …

1 2 3 4 3 2

2k +

Como cada peça tem no máximo 2 círculos pintados e nós temos 3 2

2k +

peças (note que 3 2

2k +

∈ pois 0(mod 2))k ≡ e na peça podemos pintar

um círculo ⇒ 1 3( 1)k ka a k+ ≤ + + nesse caso. Segundo caso: 0 02 1 6 4k k n k= + ⇒ = + As três últimas linhas são assim:

3k +2

3k +3

3k + 4 …

13 12

2k kka a r+

+ ≤ + +

onde r é o número de círculos pintados da peça .

Temos que 3r ≤ .(*) Se r = 2 temos de novo 1 3( 1)k ka a k+ ≤ + + e um jeito de montarmos é o seguinte:

Page 134: Eureka 2003

Sociedade Brasileira de Matemática

EUREKA! N°18, 2003

10

1

2

3 4

5

6

7

8

9

3k + 1= n

3k + 2 = n + 1

3k + 3 = n + 2

3k + 4 = n + 3

Pegue a peça destacada e coloque assim:

A partir daí fazemos por indução:

Nos dois casos (k par ou k ímpar), as três últimas linhas podem ser pintadas assim: Suponha pintado até a linha n e pinte as linhas n + 1, n + 2, n + 3 da seguinte forma. Na linha n + i = 3k + 1 + i, 1 ≤ i ≤ 3 faça o seguinte: enumere de 1 a n + i. Se i = 1: na linha n + 1, se o círculo tiver um número da forma 3k, pinte-o de vermelho. Se i = 2: na linha n + 2, se o círculo tiver um número da forma 3k + 2, pinte-o. Se i = 3: na linha n + 3, se o círculo tiver um número da forma 3k + 1, pinte-o.

Assim, teremos 1 13 ( 1)3( 1), 4 1 , 1.

2k k kk ka a k a a k+

+≥ + + = ⇒ ≥ + ∀ ≥

Vamos ver o que acontece se em (*) tivermos r = 3 e 3 12

2k r+ ⋅ +

círculos

pintados nas 3 últimas linhas.

A peça será pintada assim As 3 últimas linhas serão pintadas assim:

Page 135: Eureka 2003

Sociedade Brasileira de Matemática

EUREKA! N°18, 2003

11

3k + 1

… 3k + 2

3k + 3

3k + 4

… … …

3k 3k – 1

Na linha i coloque os números assim:

1 2 3 4 5 6 … i

Na linha 3k + 2 e 3k + 4 os números impares serão pintados e na linha 3k + 3

ninguém. Como cada peça da forma só tem no máximo 2 círculos pintados, nas linhas 3k + 2 e 3k + 4 todos os números ímpares serão pintados, porque se não

1 3( 1).k ka a k+ ≤ + + Assim, nas 3 linhas anteriores (3k – 1, 3k e 3k + 1) temos no máximo 2k círculos pintados (pois na linha 3k + 1 não podemos ter nenhum), donde 1 1 12 3 4 3 3( 1)k k ka a k k a k k+ − −≤ + + + ≤ + + + e, de qualquer jeito, temos

1

3 ( 1)1 3 1 ,2

k

kj

k ka j=

+≤ + = +∑ por indução. A resposta, portanto, é

3 ( 1)1 .2k

k ka += +

PROBLEMA 6 Demonstrar que existe uma seqüência de inteiros positivos ,...x,...,x,x n21 que satisfaz as duas condições seguintes: i) contém exatamente uma vez cada um dos inteiros positivos, ii) para cada 1,2,...,n = a soma parcial nxxx +++ ...21 é divisível por nn . SOLUÇÃO DE FÁBIO DIAS MOREIRA (RIO DE JANEIRO – RJ) Definiremos indutivamente uma tal seqüência. Seja x1 = 1, x2 = 3, e suponha a seqüência definida até x2n. Seja k o menor inteiro positivo que ainda não apareceu entre 1 2,..., .nx x Pelo Teorema Chinês dos Restos, existem infinitos 2 1nx + tais que

Page 136: Eureka 2003

Sociedade Brasileira de Matemática

EUREKA! N°18, 2003

12

2 12 1 1 2 2... (mod(2 1) )n

n nx x x x n ++ ≡ − − − + , e

2 22 1 1 2 2... (mod(2 2) )n

n nx x x x k n ++ ≡ − − − − +

já que 2 1 2 2(2 1,2 2) 1 ((2 1) ,(2 2) ) 1.n nmdc n n mdc n n+ ++ + = ⇒ + + = Em particular, existe um tal 2 1 1 2 2{ , ,..., , },n nx x x x k+ ∉ já que este último conjunto é

finito. Tomamos 2 1 2 1n nx x+ += e 2 2 .nx k+ = Note que 2 11 2 1(2 1) ...n

nn x x+++ + + e

2 21 2 2(2 2) ... ,n

nn x x+++ + + logo a extensão da seqüência respeita (ii). Como,

para 1 2 ,i n≤ ≤ 2 1 2 1, n i nx x x k+ +≠ ≠ por construção e 2 2n ix x+ ≠ pela definição de k, a extensão respeita que cada inteiro aparece no máximo uma vez. Mas depois de x extensões, todos os números 1, 2,…,x já apareceram, pois são os x menores inteiros positivos. Logo todo inteiro positivo x aparece até o (2x + 2)-ésimo termo, logo todo inteiro aparece pelo menos uma vez. Logo ( )i ix ∈ respeita (i), e

portanto ( )i ix ∈ satisfaz as condições do enunciado.

Page 137: Eureka 2003

Sociedade Brasileira de Matemática

EUREKA! N°18, 2003

13

XLIV OLIMPÍADA INTERNACIONAL DE MATEMÁTICA 07 a 19 de julho, Tóquio - Japão

PROBLEMA 1 Seja A um subconjunto do conjunto }1000000,,2,1{ …=S com exatamente 101 elementos. Demonstre que existem números 10021 ,,, ttt … em S tais que os conjuntos

}|{ AxtxA jj ∈+= , para 1, 2, ,100,j = …

são disjuntos dois a dois. SOLUÇÃO DE FÁBIO DIAS MOREIRA (RIO DE JANEIRO – RJ)

Seja { | , , }.A x y x y A x y∆ = − ∈ > Certamente 101

,| | 5050, 2

A S A

∆ ⊂ ∆ ≤ =

já que

há 101 2

pares de inteiros 2( , )x y A∈ tais que x > y. Afirmamos que se

1 100{ ,..., }T t t= é tal que , , {1,...,100},j it t A i j− ∉∆ ∀ ∈ então 1 100,...,t t satisfazem o

enunciado. De fato, se houver , , ,x y A x y∈ > tais que ,i jx t y t+ = + então

.j ix y t t− = − Mas o lado esquerdo da igualdade está em jA∆ , o direito não, absurdo! Então construímos T da seguinte forma: i) 1 T∈ ii) Para cada elemento it adicionado, proibimos todos os inteiros da forma

, .it x x A+ ∈∆ iii) Escolhemos o menor inteiro que não foi proibido, nem escolhido

anteriormente. iv) Repetimos ii) e iii) 99 vezes. Se existissem , ,i j j it t t t> em T, tais que ,j it t A− ∈∆ então, como j it t> ,

*,j it t δ δ += + ∈ , logo ,Aδ ∈∆ absurdo, pois jt teria sido proibido, pois é da

forma , .it x x A+ ∈∆ Logo basta verificar que .T S⊂ Mas como min 1T = , basta verificar

6max 10 .T ≤ Mas se 1 2 100{1 ... }T t t t= = < < < , então 2 5052t ≤ , pois há no máximo 5050 inteiros proibidos entre ele e 1t , além de 1 já escolhido. Da mesma

Page 138: Eureka 2003

Sociedade Brasileira de Matemática

EUREKA! N°18, 2003

14

maneira, 63 10010103,..., 99 5050 100 500050 10 ,t t≤ ≤ × + = < concluindo a

demonstração. PROBLEMA 2 Determine todos os pares de inteiros positivos ),( ba tais que

12 32

2

+− baba

é um inteiro positivo. SOLUÇÃO DE ALEX CORRÊA ABREU (NITERÓI – RJ) Primeiro caso: b = 1

2

2aa

⇒ é inteiro ⇔ a é par ⇒ temos as soluções (2k, 1) k ∈ N*.

Segundo caso: 1b > como queremos inteiro positivo 2 3 22 1 0 (2 ) 1 0ab b b a b⇒ − + > ⇒ − + > e como

1, devemos ter 2b b a> ≤ agora:

2 2 2 2 22 ')se 2 , (2 ) 1 (2 ) 1 1 ( 2 1)a b a b a b a a a b a a b a≥ ≥ − + ≤ ⇒ − + ≤ ⇒ ≤ − + ⇒

2b a⇒ = . Temos então as soluções ( ,2 )a a *a∈N . 2'') se b < a e (a, b) é uma solução, suponha que

22 2 3

2 3 2 ( 1) 02 1

a k a kb a b kab b

= ⇒ − + − =− +

então a equação 2 2 32 ( 1) 0x kb x b k− + − =

tem uma solução a e a outra é 22 ,kb a− que é positiva pois o produto das soluções é 3( 1) 0.b k− > Então vamos mostrar que

2 2 32 3 4 3

2 3 2 3

22 22 1 2 1

a b ab ab kb a b a b ab b b ab aab b ab b

−≥ − ⇔ ≥ − ⇔ ≥ ⇔ − + ≥ − ⇔

− + − +3 4 30 ( ) 0ab b b a b a b a b⇔ + − + ≥ ⇔ − + + ≥ o que é verdade, pois .a b> Assim, pelo

caso 2'), 2 2 3

2 3 4 32 3 2 3

4 2 22(2 ) 2 2 2 22 1 2 1

a b ab ab kb a a ab b b ab aab b ab b

−= − = − = ⇒ − + = −

− + − +

442

2b ba b b a −

⇔ = − ⇔ = , com b par, pois2

4bk = . Temos então as soluções

4

, , 12

b b b b −

>

par.

Page 139: Eureka 2003

Sociedade Brasileira de Matemática

EUREKA! N°18, 2003

15

PROBLEMA 3 Considere um hexágono convexo tal que para cada quaisquer dois lados opostos verifica-se a seguinte propriedade: a distância entre os seus pontos médios é igual a 2

3 vezes a soma dos seus comprimentos. Demonstre que todos os ângulos do hexágono são iguais. (Um hexágono convexo ABCDEF tem três pares de lados opostos: AB e DE , BC e EF , CD e FA ). SOLUÇÃO OFICIAL Primeira Solução: Lema: Considere o triângulo PQR com 60 .QPR∠ ≥ ° Seja L o ponto médio de QR. Logo 3 2PL QR≤ , com igualdade se e somente se o triângulo PQR é equilátero. Prova:

P

Q

S

R L

Seja S um ponto tal que o triângulo QRS é equilátero, e os pontos P e S encontram-se no mesmo semiplano limitado pela linha QR. Logo o ponto P pertence ao circuncírculo do triângulo QRS, que se encontra dentro do círculo de centro L e raio 3 2QR . Isto completa a prova do lema.

Page 140: Eureka 2003

Sociedade Brasileira de Matemática

EUREKA! N°18, 2003

16

B M A

C F

D N

P

E

As diagonais principais do hexágono convexo formam um triângulo embora o triângulo possa ser degenerado. Assim podemos escolher duas das três diagonais que formam um ângulo maior ou igual a 60°. Sem perda de generalidade, podemos assumir que as diagonais AD e BE do hexágono dado ABCDEF satisfazem 60APB∠ ≥ ° , onde P é a interseção dessas diagonais. Logo, usando o lema, temos:

( )3 ,2

MN AB DE PM PN MN= + ≥ + ≥

onde M e N são pontos médios de AB e DE, respectivamente. Assim pelo lema, os triângulos ABP e DEP são equiláteros. Consequentemente a diagonal CF forma um ângulo maior ou igual a 60° com uma das diagonais AD e BE. Sem perda de generalidade, assumimos que

60 ,AQF∠ ≥ ° onde Q é a interseção de AD e CF. Da mesma maneira acima, deduzimos que os triângulos AQF e CQD são equiláteros. Isto implica que

60 ,BRC∠ = ° onde R é a interseção de BE e CF. Usando o mesmo argumento acima pela terceira vez, obtemos que os triângulos BCR e EFR são equiláteros. Isto completa a solução.

Page 141: Eureka 2003

Sociedade Brasileira de Matemática

EUREKA! N°18, 2003

17

Segunda Solução: Seja ABCDEF o hexágono dado e seja , ,..., .a AB b BC f FA= = =

B M A

C F

D N E

b

a

f

e

d

c

Sejam M e N os pontos médios dos lados AB e DE, respectivamente. Temos

1 12 2

MN a b c d= + + + e 1 1 .2 2

MN a f e d= − − − −

Assim temos

1 ( ).2

MN b c e f= + − − (1)

Da propriedade dada temos

( )3 3 .2 2

MN a d a d= + ≥ − (2)

x = a – d, y = c – f, z = e – b. desde (1) e (2) obtemos: 3 .y z x− ≥ (3) Similarmente vemos que

3 ,z x y− ≥ (4)

3 .x y z− ≥ (5) Note que

(3) 2 2 22 3 ,y y z z x⇔ − ⋅ + ≥

Page 142: Eureka 2003

Sociedade Brasileira de Matemática

EUREKA! N°18, 2003

18

(4) 2 2 22 3 ,z z x x y⇔ − ⋅ + ≥

(5) 2 2 22 3 .x x y y z⇔ − ⋅ + ≥

Adicionando as últimas três inequações, obtemos

2 2 2 2 2 2 0,x y z y z z x x y− − − − ⋅ − ⋅ − ⋅ ≥

ou 2 0.x y z− + + ≥ Assim 0x y z+ + = e valem as igualdades em todas as inequações acima. Daí, concluímos que:

0,x y z+ + =

3 , // // ,y z x a d x− =

3 , // // ,z x y c f y− =

3 , // // .x y z e b z− =

Supondo que PQR é o triângulo tal que , , .PQ x QR y RP z= = = Podemos assumir 60QRP∠ ≥ ° , sem perda de generalidade. Seja L o ponto médio de QR,

logo 2 3 2 3 2.PL z x y Q R= − = = Segue do lema na solução 1 que o triângulo PQR é equilátero. Assim temos

... 120 .ABC BCD FAB∠ =∠ = = ∠ = ° Nota: Obtemos a caracterização completa dos hexágonos satisfazendo a propriedade dada. São obtidos a partir do triângulo equilátero cortando seus cantos na mesma altura. PROBLEMA 4 Seja ABCD um quadrilátero convexo cujos vértices estão sobre uma circunferência. Sejam P , Q e R os pés das perpendiculares às retas BC , CA e AB , respectivamente, passando por D . Demonstre que QRPQ = se e só se as bissetrizes dos ângulos ABC∠ e ADC∠ se intersectam sobre a reta AC .

Page 143: Eureka 2003

Sociedade Brasileira de Matemática

EUREKA! N°18, 2003

19

SOLUÇÃO DE ALEX CORRÊA ABREU (NITERÓI – RJ)

D

R

A β Q α

B

C

P

Seja CAB = β e BCA = α. Primeiro temos que as bisetrizes se intersectam sobreAC⇔

sen sen

AB AD ADBC DC CD

αβ

= ⇔ = .Mas sen

PQ DCQCP

= pois DQCP é inscritível

de diâmetro DC⇒ senPQ DC α= ⋅ e, analogamente, sensen sen sensen

ADQR AD PQ QR DC ADDC

αβ α ββ

= ⋅ ⇒ = ⇔ ⋅ = ⋅ ⇔ =

como queríamos. Observe que a nossa demonstração independe do fato de ABCD ser um quadrilátero inscritível. De fato, o quadrilátero mostrado no desenho não o é: A está no segmento BR e C está no segmento BP. Todos os argumentos utilizados continuam válidos se modificarmos as posições de A ou C. PROBLEMA 5 Sejam n um inteiro positivo e nxxx ,,, 21 … números reais tais que

nxxx ≤≤≤ 21 .

(a) Demonstre que

2 22

1 1 1 1

2( 1) ( ) .3

n n n n

i j i ji j i j

nx x x x= = = =

−− ≤ −

∑∑ ∑∑

Page 144: Eureka 2003

Sociedade Brasileira de Matemática

EUREKA! N°18, 2003

20

(b) Demonstre que a igualdade é válida se e só se nxxx ,,, 21 … formam uma progressão aritmética.

SOLUÇÃO Vamos provar (a) por indução em n. Para n = 1 os dois lados valem 0 e, para n = 2, valem 2

2 14( ) .x x− Façamos o passo da indução: Temos

1 2 1.n nx x x x +≤ ≤ ≤ ≤… Assim, sendo 1 1

n n

n i ji j

A x x= =

= −∑∑ e

( )2

1 1

,n n

n i ji j

B x x= =

= −∑∑ temos ( )1 1

1 11 1 1

2n n n

n i j n j ni j j

A x x x x A+ +

+ += = =

= − = − +∑∑ ∑

e ( )2

1 11

2 .n

n n j nj

B x x B+ +=

= − +∑ Queremos então provar que

( )( )22

21 1 1

1

2 1 12 2

3

n

n n j n nj

nA n x x A B+ + +

=

+ − = ⋅ − + ≤ ⋅ =

( ) 2 21 1

1 1

2 22 4 2 ,

3

n n

n n j j nj j

n nnx x x x B+ +

= =

+= − + +

∑ ∑ ou equivalentemente, que

( ) 222 2

1 11 1 1

4 1 2( 1) 2 ( 2)4 2 2 0.3 3 3

n n n

n j n n n j n jj j j

n n n n nx n x A x B x A x+ += = =

− − +− + + + − − ≥

∑ ∑ ∑

Olhando o lado esquerdo como uma função quadrática de 1nx + , concluímos que

ela atinge o seu mínimo para 11

2( 1) 3,

2 ( 1)

n

j nj

n

n x Ax

n n=

+

− +=

∑quando temos

2

212

1 11 1

2( 1) 32 2 2

1

n

j nn nj

n n j n j nj j

n x AA nx x A x A

n=

+ += =

− + = − + = − + = −

∑∑ ∑

22 ,1 n

n An+ ⋅ −

enquanto 2 21 1 1

1 1

2 4 2n n

n n n j j nj j

B nx x x x B+ + += =

= − + + =∑ ∑

Page 145: Eureka 2003

Sociedade Brasileira de Matemática

EUREKA! N°18, 2003

21

21 1

1 12 2 2

n n

n n j j nj j

x nx x x B+ += =

= − + + =

∑ ∑

1 1 2

1

2( 1) 3 3 2( 1)2

( 1) 2( 1)

n n

j n n j nj j

j nj

n x A A n xx B

n n n= =

=

− + − −= ⋅ + + =

− −

∑ ∑∑

2

22 21 2 2

2 21 1 1

29 912 2

2 ( 1) 2 ( 1)

n

j n n nj n n

j n n j jj j j

xA Ax B B n x x

n n n n n n=

= = =

− = + + + = + − + − −

∑∑ ∑ ∑ ,

mas ( )2

2 2

1 1 1 12 ,

n n n n

n j i j ji j j j

B x x n x x= = = =

= − = −

∑∑ ∑ ∑ donde 21 1

2 ( 2)3n n

n nA B+ ++

≤ é

equivalente a ( )( )

2 22

2 2

2 92 ( 2) 13 2 ( 1)1

nn n

n An n nA Bn n nn

+ + + ≤ + − −

2 2 23( 2) 2( 1)( 1) 9n n nn A n n B A⇔ + ≤ + − + 2

2 2 2 2( 1)3( 1) 2( 1)( 1) ,3n n n n

nn A n n B A B−⇔ − ≤ + − ⇔ ≤ que é a hipótese de

indução. Assim, o valor mínimo de nossa função quadrática é maior ou igual a 0, o que implica o resultado. Para provar b), note que, se valem as igualdades nas desigualdades acima,

devemos ter em particular 2

2 2( 1)3n n

nA B−= donde, por hipótese de indução,

1 ( 1)jx x j r= + − para um certo 0r ≥ , ;j n∀ ≤ além disso, devemos ter

1 11

2( 1) 3 2( 1) 6 ( )

2 ( 1) 2 ( 1)

n n

j n jj j i i j n

n

n x A n x j i rx

n n n n= = ≤ < ≤

+

− + − + −= = =

− −

∑ ∑ ∑

1

11 1

1 3( ( 1) ) ( )( 1)

n n

j kx j r n k kr

n n n

= =

= + − + − =−∑ ∑

1 1( 1) 3 ( 1)( 1) ,

2 ( 1) 6n n n n rx r x nr

n n− − + + + = + −

donde 1 1( )i i nx ≤ ≤ + é uma

progressão aritmética, como queríamos provar.

Page 146: Eureka 2003

Sociedade Brasileira de Matemática

EUREKA! N°18, 2003

22

PROBLEMA 6 Seja p um número primo. Demonstre que existe um número primo q tal que,

para todo inteiro n, o número pn p − não é divisível por q. SOLUÇÃO DE SAMUEL BARBOSA FEITOSA (FORTALEZA - CE) É evidente que p ≠ q pois se p = q basta tomarmos n múltiplo de q para termos um absurdo.

Se encontrarmos um primo q tal que p 1−q e )(mod11

qp pq

≠−

(1)

o problema terá acabado pois se )(mod qpn p ≡ , como qp ≠ então mdc (n, q)

= 1, donde 1 1

(mod )q qp

p pn p q− −

⋅≡ mas pelo pequeno teorema de Fermat temos

)(mod11 qn q ≡− . Isso produz um absurdo por (1), logo devemos ter

)(mod qpn p ≠ . Para encontrarmos o nosso tal primo q que satisfaz (1) consideremos o número

1...21 ++++= −− pppX pp . É claro que 1−ppX (2).

Seja q um divisor primo de X e pordk q= . De (2) temos que

1=⇒ kpk ou pk = . Se k = 1 temos )(mod1)(mod1 qpqp j ≡⇒≡ ; daí

)(mod1...11 qpX =++≡ mas Xq e pq ≠ , absurdo, logo k = p, mas

11)(mod11 +=⇒−=⇒≡− pyqqkpqp q para algum y inteiro. Existe algum fator primo q de X tal que y não seja divisível por p, pois se para todo fator primo q de X tivermos 21(mod )q p≡ , teríamos 21(mod )X p≡ , mas

)(mod11 2ppX ≠+≡ logo existe um primo q divisor de X tal que 1−qp e

)(mod1 2pq ≠ . Mostremos que tal primo q satisfaz (1). Como ppordk q == e

)(mod1 2pq ≠ pk =⇒ não divide ⇒−p

q 1 )(mod11

qp pq

≠−

, donde o primo

q satisfaz (1).

Page 147: Eureka 2003

Sociedade Brasileira de Matemática

EUREKA! N°18, 2003

23

X OLIMPÍADA INTERNACIONAL DE MATEMÁTICA PARA ESTUDANTES UNIVERSITÁRIOS

25 a 31 de Julho, Cluj - Napoca, Romênia PROBLEMA 1 a) Seja 1 2, , , ,...na a a… uma seqüência de números reais tais que 1 1a = e

13 , .2n na a n+ > ∀

Prove que a seqüência 132

nn

a−

tem um limite finito ou tende a infinito.

b) Prove que para todo 1α > existe uma seqüência 1 2, , , ,...na a a… com as

mesmas propriedades, tal que 1lim .32

nnn

aα−→∞

=

SOLUÇÃO

(a) Considere a seqüência 1

23 −

=n

nn

ab .

Temos 1 10, 132

n nn

nn

b ab n eab

+ +> ∀ = >

, logo (bn) é uma seqüência crescente.

Mas toda seqüência monótona limitada é convergente. Logo, há duas opções possíveis: ou (bn) é ilimitada e, como é crescente, tende a infinito ou então, se ela for limitada, ela tem um limite finito (pois é crescente).

(b) Tome α11−=q e considere a seqüência 12 ...1 −++++= n

n qqqb .

101 <<⇒> qα e portanto (bn) é uma seqüência convergente, com 1lim

1nnb

→∞= =

−.

Page 148: Eureka 2003

Sociedade Brasileira de Matemática

EUREKA! N°18, 2003

24

Portanto, a seqüência n

n

n ba1

23 −

= satisfaz as condições do enunciado.

Observação: A solução da letra (b) é bem mais natural do que parece. A letra (a) induz você a pensar apenas na seqüência (bn). E na tentativa de encontrar uma seqüência com determinadas condições, nada mais natural do que tentar uma seqüência fácil como uma PG (e aí descobrir o valor de q necessário). PROBLEMA 2 Sejam 1 2 51, , ,a a a… elementos não nulos de um corpo. Simultaneamente trocamos cada elemento pela soma dos outros 50. Desta forma a nova seqüência

1 2 51, , ,b b b… é uma permutação da anterior. Quais são os possíveis valores da característica do corpo? SOLUÇÃO Seja p a característica do corpo.

SSaSaSbSk

kk

kk

k −=−=−== ∑∑∑===

5151)(51

1

51

1

51

1, logo 49S = 0.

Se 0≠S , temos que 49 = 0, logo 49|p e como p é primo, p = 7. Um exemplo

de corpo que satisfaz essa propriedade é ( )*

7 , com 1=ka , para k = 1,2,...,51.

Se S = 0, cada ka é igual a jk ab −=− , para algum j, donde a permutação σ tal

que )(kk aa σ−= possui um ponto fixo, pois os números 1,2,...,51 estão divididos em pares (pela ação da σ ) e 51 é ímpar. Portanto existe um n tal que

nnn aaa −=−= )(σ , logo 02 =na e como 0≠na , temos que 2 = 0 e o corpo possui característica 2. Um exemplo de corpo que satisfaz essa propriedade é GF(22), isto é, o corpo cujos elementos são polinômios, tomados módulo x2 + x + 1, com coeficientes em 2 . Basta tomar )1,,1,...,1,,1,1,,1(),...,,( 5121 xxxxxxaaa +++= . PROBLEMA 3 Seja A uma matriz quadrada n × n tal que 3A3 = A2 + A + I. Prove que (Ak )k ∈ N converge a uma matriz idempotente B (i.e., a uma matriz B tal que B2 = B).

Page 149: Eureka 2003

Sociedade Brasileira de Matemática

EUREKA! N°18, 2003

25

SOLUÇÃO Seja m(x) o polinômio minimal de A. Como 03 23 =−−− IAAA , m(x) deve

ser divisor de ))()(1(13 2123 λλ −−−=−−− xxxxxx , com

321

2,1±−

=λ .

Isso implica que m(x) tem raízes distintas, e portanto A é diagonalizável, isto é, existe uma matriz P inversível tal que PDiagPA ⋅λλλλ⋅= − ),...,,,...,,1,...,1( 2211

1 , onde Diag(a,b,c,...) representa a matriz diagonal de entradas a,b,c,... Segue que PDiagPA kkkkk ⋅λλλλ⋅= − ),...,,,...,,1,...,1( 2211

1 .

Como 12,1 <λ , segue que PDiagPAB kk ⋅⋅== −

∞→ )0,...,0,1,...,1(lim 1 , que é

claramente idempotente. SOLUÇÃO ALTERNATIVA Seja .k

kA A= Então 3 2 13 .k k k kA A A A+ + += + + Resolvendo a recorrência (veja o artigo "Equações de recorrência", na Eureka! 9), obtemos 0 1 1 2 2

k kkA C C Cλ λ= + ⋅ + ⋅ para

todo k, sendo 0 1 2, ,C C C matrizes quadradas de ordem n. Como 1,2| | 1,λ ≤ 0lim .k

kA C

→∞=

Como 2 20 0 0lim lim ,k k

k kA A C C C

→∞ →∞= ⇔ = é idempotente.

PROBLEMA 4 Determine o conjunto de todos os pares (a, b) de inteiros positivos para os quais o conjunto dos inteiros positivos pode ser decomposto em dois conjuntos A e B tais que .a A b B⋅ = ⋅ SOLUÇÃO Note que o par (a, b) funciona se e somente se o par de coprimos

, ( , ) ( , )a b

mdc a b mdc a b

funciona. Vamos então analisar os casos com mdc(a, b)=1.

Suponha A∈1 (o outro caso é análogo). Então, BbaBbAa ⋅∈⇒⋅=⋅ , e portanto a é múltiplo de b (pois os elementos de B são todos inteiros), e mdc(a, b) = 1 implica b = 1. Reciprocamente, dado qualquer par da forma (a, 1), com a > 1, é possível construir conjuntos A e B satisfazendo o enunciado. De fato, dado Nn∈ , seja kn a maior potência de a que divide n. Tomando { | }, { | é ímpar}n nA n k é par B n k= = temos , , 1A B N A B a A B∪ = ∩ =∅ ⋅ = ⋅ .

Page 150: Eureka 2003

Sociedade Brasileira de Matemática

EUREKA! N°18, 2003

26

Portanto, os pares possíveis são os pares (a, b) com a ≠ b tais que a é múltiplo de b ou b é múltiplo de a.

PROBLEMA 5

Sejam :[0,1]g → R uma função contínua e : (0,1]nf →R a seqüência de

funções definida por 0 ( ) ( )f x g x= e 1 0

1( ) ( ) , (0,1], 0.x

n nf x f t dt x nx+ = ∀ ∈ ≥∫

Determine lim ( )nnf x

→∞para todo (0,1]x∈ .

SOLUÇÃO

Veja inicialmente que se g é polinômio, o problema é fácil, pois

se ∑=

=N

k

kk xaxg

0)( , temos claramente que ∑

= +=

N

k

kn

kn x

ka

xf0 )1(

)( , portanto

todos os coeficientes, com exceção do independente, tendem a zero quando +∞→n , e )0()(lim 0 gaxfn == . A idéia é tentar mostrar que isso também

vale no caso de g não ser polinômio.

Para isso, vamos usar o teorema da aproximação de Stone-Weierstrass: Dado

0>ε , existe um polinômio P tal que 3

)()( ε<− xgxP , para todo x em [0,1]

(pois g é contínua e [0,1] é compacto). Agora olhe para a seqüência nP~ tal que

∫=+

x

nn dttPx

xP0

1 )(~1)(~ e PP =0

~.

Como P é polinômio, )0()(~lim PxPnn=

+∞→, donde para n grande,

3)0()(~ ε<− PxPn .

Além disso, temos que se 3

)()(~ ε<− xfxP nn , então:

Page 151: Eureka 2003

Sociedade Brasileira de Matemática

EUREKA! N°18, 2003

27

[ ] ∫∫∫ =<−≤−=− ++

xx

nn

x

nnnn dtx

dttftPx

dttftPx

xfxP000

11 331)()(~1)()(~1)()(~ εε

Segue por indução que 3

)()(~ ε<− xfxP nn , para todo n natural e todo x em (0,1]

já que 3

)()(~00

ε<− xfxP . Agora fica fácil:

εεεε=++<−+−+−≤−

333)0()0()0()(~)(~)()0()( gPPxPxPxfgxf nnnn ,

para n grande, o que prova que )0()(lim gxfnn=

+∞→.

SOLUÇÃO ALTERNATIVA Vamos provar que lim ( ) (0), (0,1].nn

f x g x→∞

= ∀ ∈ Para isso, vamos mostrar que,

para todo (0,1]x∈ e 0,ε > limsup ( ) (0) .nn

f x g ε→∞

− ≤

Como g é contínua, existe 0δ > tal que ( ) (0)x g x gδ ε< ⇒ − < Se 0 ,x δ< < para todo 1,n ≥

1 10 0

1 1( ) (0) ( ( ) (0)) ( ) (0)x x

n n nf x g f t g dt f t g dtx x− −− = − ≤ −∫ ∫ donde,

por indução, ( ) (0) , .nf x g nε− < ∀ ∈ Se ,x δ≥ temos

( )1 0 0

1 1( ) (0) ( ) (0) ( ) (0)x x

n n nf x g f t g dt f t g dtx x+ − = − ≤ − =∫ ∫

( )0

1 ( ) (0) ( ) (0)x

n nf t g dt f t g dtx

δ

δ= − + −∫ ∫ , donde, se

max{ ( ) (0) , (0,1]},n nM f x g x= − ∈ temos

( )1 1

1max{ ,max ( ) } max{ , (1 ) }.n n nxM x M M

xδε δε δ ε δε δ+ < ≤

≤ + − = + − Assim, se

lim sup nn

L M→∞

= (que existe, pois 0max{ , }, )nM M nε≤ ∀ ∈ , temos

max{ , (1 ) },L Lε δε δ≤ + − donde L ε≤ ou (1 )L L L Lδε δ δ δε ε≤ + − ⇒ ≤ ⇒ ≤ , o que encerra a prova.

Page 152: Eureka 2003

Sociedade Brasileira de Matemática

EUREKA! N°18, 2003

28

PROBLEMA 6 Seja 1

1 1 0( ) ...n nn nf z a z a z a z a−

−= + + + + um polinômio com coeficientes reais. Prove que se as raízes de f estão no semi-plano esquerdo { | Re( ) 0}z z∈ <C então 3 1 2k k k ka a a a+ + +< para todo k = 0, 1,…, n – 3. SOLUÇÃO Podemos supor sem perda de generalidade que 1.na = Nesse caso, f(z) pode ser fatorado como produto de monômios da forma z + a ou z a bi+ + com a > 0 (no segundo caso, se b ≠ 0, deve aparecer também o fator z a bi+ − ).

Como 2 2 2( )( ) 2z a bi z a bi z az a b+ + + − = + + + e 2( ) ( ) ( ) ,z a z a z a a z aa+ + + = + + + temos que f(z) pode ser obtido a partir

de um polinômio de grau 0 ou 1 com todos os coeficientes positivos por meio de sucessivas multiplicações por polinômios da forma 2z Az B+ + , com A, B > 0. Em particular, todos os seus coeficientes são positivos. Vamos agora proceder por indução: para n = 0 ou n = 1 o resultado vale por vacuidade. Seja agora f(z) um polinômio de grau n + 2, da forma

20( ) ( ... )( ),n

nf z a z a z Az B= + + + + com 1na = e

0( ) ...nnf z a z a= + + satisfazendo a hipótese de indução. Convencionando

0ka = se 0k < ou k n> , e escrevendo 22 0( ) ...n

nf z a z a++= + + , temos,

para 0 2,k n≤ ≤ + 2 1 .k k k ka a Aa Ba− −= + + Queremos então provar que

1 2 3 2 1 1 2 1 1( )( ) ( )( )k k k k k k k k k k k ka Aa Ba a Aa Ba a Aa Ba a Aa Ba+ + + − − + + − ++ + + + < + + + + ,

mas 1 2 1 1 1 2 3 2 1( )( ) ( )( )k k k k k k k k k k k ka Aa Ba a Aa Ba a Aa Ba a Aa Ba+ + − + + + + − −+ + + + − + + + + 2 2

1 1 2 2 1 3 1 2 1( ) ( ) ( )k k k k k k k k k k k ka a a a B a a a a A a a a a− + − + + + + + −= − + − + − + 2 2

2 1 3 2 2 2 1 3 1( ) ( ) ( ).k k k k k k k k k kB a a a a A a a a AB a a a+ − + − + − + + −+ − + − + −

Temos, por hipótese de indução, 1 1 2 2 1 3 1 2 1, ,k k k k k k k k k k k ka a a a a a a a a a a a− + − + + + + + −≥ ≥ ≥

e, além disso, 3 2 1k k k ka a a a+ + +≤ e 1 2 1k k k ka a a a+ − −≤ , donde

1 3 2 1 2 1,k k k k k k k ka a a a a a a a+ + − + + −≤ e logo 3 2 2 1k k k ka a a a+ − + −≤ (de fato temos 1 0k ka a + > ,

a menos que k = n, quando 3 2 2 1 0)k k k ka a a a+ − + −= = ; 2 1 1k k k ka a a a+ − +≤ e

Page 153: Eureka 2003

Sociedade Brasileira de Matemática

EUREKA! N°18, 2003

29

1 2 1k k k ka a a a+ − −≤ , donde 21 1 2 2 1 1 ,k k k k k k ka a a a a a a− + + − − +≤ e logo 2

2 2k k ka a a+ − ≤ , e

analogamente 23 1 1,k k ka a a+ − +≤ sendo que pelo menos uma dessas duas últimas

desigualdades é estrita, o que conclui a prova.

SEGUNDO DIA PROBLEMA 1 Sejam A e B matrizes reais n × n tais que AB + A + B = 0. Prove que AB = BA. SOLUÇÃO

IIBIAIIBIBAIIBAABBAAB =++⇒=+++⇒=+++⇒=++ ))(()()(0Logo, A + I e B + I são inversas uma da outra, donde

IIAIBIBIA =++=++ ))(())(( . Expandindo a última desigualdade vem IIABBA =+++ e subtraindo esta da igualdade dada no enunciado obtém-se BAAB = . PROBLEMA 2

Calcule o seguinte limite: 2

0lim

mx

nxx

sen t dtt+→ ∫ (m, n naturais dados).

SOLUÇÃO

Como a função t

tsen é decrescente em ),0( π , e tende a 1 quando t tende a 0+,

temos que, para )2

,0( π∈x e x < t < 2x: 1sen

2)2sen(

<<t

tx

x e portanto:

∫∫∫ <<⋅

x

xn

mx

xn

mx

xn

mmdt

ttdt

ttdt

tt

xx 222 sen

2)2sen( .

Como 12

)2sen(lim 0 =

m

x xx , a desigualdade acima mostra que o limite

procurado é igual a

Page 154: Eureka 2003

Sociedade Brasileira de Matemática

EUREKA! N°18, 2003

30

21

0

22 1

0 0

20

0, 1 01

lim , 1 01

ln ln 2, 1 0

xm n

xx

xx m nm n

x xx x

xxx

t se m nm n

tt dt se m nm n

t se m n

− +

− +−

→ →

→ − + >

− += →∞ − + < − +

→ − + =

PROBLEMA 3 Seja A um subconjunto fechado de Rn e seja B o conjunto de todos os pontos b de

Rn tais que existe exatamente um ponto a0 em A tal que 0 infa A

a b a b∈

− = − .

Prove que B é denso em Rn. SOLUÇÃO Vamos mostrar que dado 0>ε e nx∈ , existe BxBy ∩ε∈ ),( . Se Ax∈ , então basta tomar y = x. Caso contrário, seja ||inf xaAa −=δ ∈ . Como A é fechado, existe um ponto

Aa∈ que realiza essa distância (basta observar por exemplo que ( ,2 )B x Aδ ∩ é compacto). Se esse ponto a não for único, considere um ponto )1,0(),( ∈−+= txatxy do segmento (x, a). Seja a’ um outro ponto de A. Se a’ estiver no prolongamento desse segmento de reta, então claramente |||'| yaya −>− . Caso contrário, temos a desigualdade triangular estrita |||'||'| xyyaxa −+−<− e portanto:

|||||||||'||'| yayxxayxxaya −=−−−≥−−−>− , onde a 2a desigualdade usa que a é um ponto de A tal que |a – x| é mínimo e a igualdade final usa o fato que y está no segmento de reta (x, a). Ou seja, todo ponto y escolhido dessa forma está em B. Escolhendo t

suficientemente pequeno (xa

t−⋅ε

=2

por exemplo) obtemos um ponto

BxBy ∩ε∈ ),( como desejado. PROBLEMA 4

Page 155: Eureka 2003

Sociedade Brasileira de Matemática

EUREKA! N°18, 2003

31

Encontre todos os inteiros positivos n para os quais existe uma família F de subconjuntos de três elementos de S ={1, 2, …, n} que satisfaz as seguintes condições: (i) para quaisquer elementos distintos a, b ∈ S existe exatamente um A ∈ F

tal que a, b ∈ A. (ii) Se a, b, c, x, y, z são tais que {a, b, x}, {a, c, y}, {b, c, z} ∈ F então {x, y, z} ∈ F. SOLUÇÃO Vamos mostrar que uma tal família F existe se e somente se 2 1kn = − para algum inteiro positivo k. De fato, se ( )2 2 2 ... 2kG = = × × × , onde está definida a adição

( )1 2 1 2 1 1, ,..., ( , ,..., ) ( (mod 2),n nx x x y y y x y+ = + 2 2 (mod 2),..., (mod 2))n nx y x y+ + , podemos definir \ {(0,0,...,0)}S G= e {{ , , }, , , }.F u v u v u v S u v= + ∈ ≠ A propriedade (i) segue com { , , }A a b a b= + e a propriedade ii) segue de

,x a b= + .y a c x y a a b c b c z= + ⇒ + = + + + = + = Sejam agora S e F como no enunciado. Consideramos um conjunto {0}G S= ∪ , onde 0 é um elemento de G que não pertence a S (um "zero artificial") e uma operação + definida em G por

se , , e { , , }0 se

se 0 se 0

x a b S a b a b x Fa b

a ba bb a

∈ ≠ ∈ =+ = = =

Não é difícil ver que com essa operação G é um grupo abeliano ( a + b = b + a para quaisquer a, b em G, (a + b) + c = a + (b + c), para quaisquer , ,a b c G∈ e para todo a G∈ existe b em G com a+b=0. Segue que G é isomorfo a ( )2 k para algum inteiro positivo k * , donde # # 1 2 1kn S G= = − = − , o que conclui a solução. * De fato, se , 0, {0, }a G a H a∈ ≠ = é um subgrupo de G isomorfo a 2 , e, se definimos em G a relação de equivalência ~ ,x y y x H⇔ − ∈ obtemos um quociente, G/H, o conjunto das classes de equivalência { | ~ },x y G y x= ∈ que é um grupo com a operação : x y x y+ = + , o qual têm as mesmas propriedades que G. Além disso, G é naturalmente isomorfo a ( ) ,G H H× e, por indução em #G ,

G H é isomorfo a ( )2 r para algum r∈ , donde G é isomorfo a ( ) 12 r+ .

Page 156: Eureka 2003

Sociedade Brasileira de Matemática

EUREKA! N°18, 2003

32

Obs.: O conjunto S é um espaço projetivo finito (a generalização k-dimensional do plano projetivo) sobre o corpo 2 (ou seja, de ordem 2), cuja família das retas é F. De fato, a propriedade (i) é equivalente a "por dois pontos passa uma única reta" e a propriedade (ii) é equivalente ao axioma de Veblen-Young: "dado um triângulo ABC, se uma reta r corta dois lados, então corta o terceiro lado também" (no caso, os pontos do triângulo são a,b,c e a reta é r={x,y,z}). As propriedades (i) e (ii) são, então, equivalentes aos axiomas que definem um espaço projetivo; veja o artigo "Aplicações de planos projetivos finitos em Teoria dos Números e Combinatória", de Carlos Shine, na Eureka! 15. PROBLEMA 5 a) Mostre que para toda função :f × →Q Q R existe uma função :g →Q R tal

que ( , ) ( ) ( ), ,f x y g x g y x y≤ + ∀ ∈Q . b) Encontre uma função :f × →R R R para a qual não existe :g →R R tal que f(x, y) ≤ g(x) + g(y), ∀x, y ∈ R. SOLUÇÃO a) Q é enumerável, digamos 1 2 3{ , , ,...}.r r r=Q Assim, podemos definir :g →Q R por ( ) max{ ( , ) ,1 , }.n i jg r f r r i j n= ≤ ≤

Assim, ( )max( , )( , ) ( , ) ( ) ( ), , .i j i j i j i jf r r f r r g r g r g r i j≤ ≤ ≤ + ∀

b) Podemos definir 0, se

( , ) 1 , se .

x yf x y x y

x y

== ≠ −

Se existisse :g → tal que ( , ) ( ) ( ), ,f x y g x g y x y≤ + ∀ ∈ , se definirmos,

para cada inteiro positivo n, { | ( ) 2},nX x g x n= ∈ ≤ teremos 1

nn

X∞

=

=∪ ,

donde, como é não-enumerável, algum dos nX deve ser não enumerável, e portanto tem pontos de acumulação, isto é, existe uma seqüência de termos distintos ( )k ky ∈ com k ny X∈ para todo k tal que ( )ky converge a um certo

x∈ . Em particular, 1lim 0k kky y x x+→∞

− = − = , e logo 11

1( , )k kk k

f y yy y+

+

=−

tende a +∞ , mas devemos ter 1 1( , ) ( ) ( )2 2k k k kn nf y y g y g y n+ +≤ + ≤ + = para todo

k ∈ , pois 1{ , } ,k k ny y X+ ⊂ donde 1( ( , ))k k kf y y + ∈ é limitada, absurdo.

Page 157: Eureka 2003

Sociedade Brasileira de Matemática

EUREKA! N°18, 2003

33

PROBLEMA 6

Seja 0 1, ,..., ,...na a a a seqüência definida por 0 1a = , 10

1 .1 2

nk

nk

aan n k+

=

=+ − +∑

Calcule 0 2

kk

k

a∞

=∑ (se existir).

SOLUÇÃO Os an são positivos e, por indução, tem-se 1,na n≤ ∀ (de fato, supondo válido até n tem-se

∑∑==

+ =+

≤+−+

≤n

k

n

kn nknn

a00

1 111

12

11

1).

Considere então a função geratriz ∑∞

==

0)(

n

nnxaxf (ele é convergente para 0 < x <

1 pela observação acima). Derivando e usando a expressão dada obtemos:

∑ ∑∑∑∞

= =

=+

=

+−

=⋅⋅+=⋅⋅=0 00

11

12

)1()('n

nn

k

k

n

nn

n

nn x

knaxanxanxf

Trocando a ordem do somatório obtém-se:

∑∑∑∑ ∑∞

=

=

−∞

=

=

= +⋅=

+−⋅

=

+−=

000 2)(

22)('

m

m

kn

kn

k

kk

k kn

nk

mxxf

knxxa

knxaxf

Portanto,

∑∫ ∑∞

=

++∞

=

+

+−

+=

++==−

0

11

0 0

1

)2()1()2)(1('))0(ln())(ln(

m

mmx

m

m

mx

mx

mmx

fffxf

Como f (0) = 1:

( )

−−

−=

+−

+=

+−

+= ∑∑∑∑

=

+∞

=

+∞

=

+∞

=

+x

xxxmx

xmx

mx

xmxxf

m

m

m

m

m

m

m

m

11ln1

11ln

11

121

1)(ln

1

1

0

1

0

2

0

1

Colocando 21

=x obtemos

2ln112ln22ln)21(ln −=+−=f , de modo que ln 2

0

1( )2 2 2

nn

n

a ef e e∞

=

= = ⋅ =∑ .

Estas soluções da International Mathematical Competition - 2003 foram redigidas por Márcio Assad Cohen, Rodrigo Villard Milet e Carlos Gustavo Moreira do Rio de Janeiro – RJ.

Page 158: Eureka 2003

Sociedade Brasileira de Matemática

EUREKA! N°18, 2003

34

XVIII OLIMPÍADA IBEROAMERICANA DE MATEMÁTICA 13 a 20 de setembro, Mar del Plata - Argentina

PRIMEIRO DIA

PROBLEMA 1 a) Têm-se duas sucessões, cada uma de 2003 inteiros consecutivos, e um

tabuleiro de 2 linhas e 2003 colunas

……… ………

Decida se é sempre possível distribuir os números da primeira sucessão na primeira linha e os da segunda sucessão na segunda linha, de modo que os resultados obtidos ao somar os dois números de cada coluna formem uma nova sucessão de 2003 números consecutivos. b) E se trocássemos 2003 por 2004? Tanto em a) como em b), se a resposta for afirmativa, explique como distribuiria os números, e se for negativa, justifique o porquê. SOLUÇÃO Note que somar ou subtrair uma constante de uma sucessão de números consecutivos a transforma em uma sucessão de números consecutivos. Note também que somar ou subtrair uma constante de uma linha do tabuleiro soma ou subtrai a mesma constante da sucessão formada pela soma das colunas, logo esta operação não altera a "consecutividade" das linhas do tabuleiro. Portanto, sem perda de generalidade, as sucessões escritas nas duas primeiras linhas do tabuleiro são 1, 2, …,n onde {2003,2004}n∈ a) Sim. Escreva na primeira linha 1, 2,…,2003 e na segunda linha 1002,

1003,…,2003, 1, 2, …, 1000, 1001, ou seja, o i-ésimo termo é 1001i + se 1002.i ≤ A seqüência final é 2 1001i + se 1002i ≤ e 2 1002i − se 1003i ≥ , que é obviamente permutação de 1003, 1004, 1005,…,3003, 3004,

3005. b) Não. Uma seqüência de 2004 números inteiros consecutivos tem forma

1,..., 2004,k k k+ + ∈ . Sua soma vale

Page 159: Eureka 2003

Sociedade Brasileira de Matemática

EUREKA! N°18, 2003

35

2004 20052004 2004 2004 1002 1002 1002(mod 2004).

2k k⋅+ = + ⋅ + ≡

A soma dos números da primeira e da segunda linhas vale 1002 (mod 2004). Como a terceira linha (formada pela soma das colunas) é formada pela soma das duas primeiras linhas, a soma das números da terceira linha é 1002 1002 0 (mod 2004).+ ≡ Mas se a terceira linha fosse composta por uma sucessão de números consecutivos em alguma ordem, sua soma seria 1002 (mod 2004), absurdo! Logo a soma das colunas não pode formar uma sucessão de números consecutivos. PROBLEMA 2 Sejam C e D dois pontos da semicircunferência de diâmetro AB tais que B e C estão em semiplanos distintos em relação à reta AD. Denotemos por M, N e P os pontos médios de AC, DB e CD, respectivamente. Sejam OA e OB os circuncentros dos triângulos ACP e BDP. Demonstre que as retas OAOB e MN são paralelas. SOLUÇÃO

2π β−

2π β−

2π γ−

2π γ+

A O B

S C

P T

D

β γ

Q

R β . .

. .

γ

. .

.

. N 2β

O2

M .

Sejam 2 ,2α β e 2γ as medidas de ,CD BD e AC , respectivamente. Seja O o centro da semicircunferência ACDB. Sejam R e Q os pés das perpendiculares a OP que passam por 2O e 1O , respectivamente.

Sejam ainda S e CP , T e PD pontos médios de CP e PD, respectivamente.

Page 160: Eureka 2003

Sociedade Brasileira de Matemática

EUREKA! N°18, 2003

36

Note que 2SCM πβ= + pois é ângulo inscrito. Note ainda que AMO AC⊥ e

ASO CP⊥ , pois são mediatrizes de AC e CP . Logo AMCSO é inscritível e

2AMO S π β= − .

Como A ASO O Q⊥ (pois ASO CP PO⊥ ⊥ , logo // APO SO ), .AQO O β=

Logo cos .4 4cos

AA

A A A

O Q SP CD CDO OO O O O O O

ββ

= = = ⇔ =

Analogamente, .4cosB

CDO Oγ

= Olhando para o triângulo OMA, MOA γ=

(pois M é ponto médio da corda AC , logo 2MOA AC= ), logo

cos cos .OM OM OM ROA R

γ γ= = ⇔ = Analogamente, cos .ON R β= Mas

cos 4cos cos .

4cosA B

OM R R ONCDO O CD O O

γ β β

γ

⋅= = = Logo existe uma homotetia de

centro O que leva M em AO e N em BO . Como homotetias preservam

paralelismo, // .A BMN O O PROBLEMA 3 Pablo copia o seguinte problema: Considere todas as sucessões de 2004 números reais 0 1 2 2003( , , ,..., ),x x x x tais que

0 1,x =

1 00 2 ,x x≤ ≤

2 10 2 ,x x≤ ≤

2003 20020 2 .x x≤ ≤ Entre todas estas sucessões, determine aquela para a qual a expressão seguinte assume o seu maior valor: S = … .

Page 161: Eureka 2003

Sociedade Brasileira de Matemática

EUREKA! N°18, 2003

37

Quando Pablo ia copiar a expressão S, apagaram o quadro. Só conseguia lembrar-se de que S era da forma

1 2 2002 2003... ,S x x x x= ± ± ± ± +

onde o último termo, 2003x , tinha coeficiente +1, e os anteriores tinham coeficiente +1 ou –1. Demonstre que Pablo, apesar de não ter o enunciado completo, pode determinar com certeza a solução do problema. SOLUÇÃO

Seja { 1,1}ic ∈ − o coeficiente associado ao termo ix na expressão de S. Em

particular, 2003 1c = . Dizemos que um termo ix é positivo se 1ic = , ou que é

negativo se 1ic = − . Dizemos que ix está maximizado se 12i ix x −= .

Lema: Se 1,...,i jx x+ estão maximizados, então:

0 11 1 1... ( 2 2 ... 2 )j i

i i i i j j i i i jc x c x c x x c c c −+ + ++ + + = + + +

Prova: É fácil ver que 2 , .k ik ix x i k j−= ≤ ≤ A prova segue trivialmente.

Corolário: A soma acima tem o mesmo sinal de jc .

Prova: Como 0ix ≥ , basta analisar o sinal de 02 ... 2 .j ii jc c −+ + Suponha jc

positivo (i.e. = 1). Então 1

2 2 2 2 1 2 1.j j

k i k i j i j i j ik

k i k ic

−− − − − −

= =

≥ − + = − + + =∑ ∑ O

caso 1jc = − é análogo.

É obvio que maximizar os termos positivos aumenta a soma (já que [0,2 ] [0,2 ]x y x y≤ ⇒ ⊆ , podemos aumentar um termo sem alterar nenhum

dos seguintes). Logo, sem perda de generalidade, na seqüência que maximiza S todos os termos positivos são máximos. Se houver algum termo negativo não maximizado, escolha o último deles, digamos ix e maximize tanto ele quanto todos os termos que o seguem. Sejam a e

b os valores antigo e novo de ix e A e B os valores da soma

1 1 2003 2003...i ic x c x+ + + + antes e depois da mudança na seqüência.

Seja ainda 0 1 20031 20032 2 ... 2 .i

i iC c c c−+= + + + Pelo lema, A = aC e B = bC.

Page 162: Eureka 2003

Sociedade Brasileira de Matemática

EUREKA! N°18, 2003

38

Logo B – A = C(b – a) > 0, pois, pelo corolário, C > 0, e como b > a, b – a > 0. Logo, na seqüência de S máximo, todos os termos são máximos ou poderíamos aumentar S maximizando algum termo.

Logo 1 02 2,x x= = 2 1 20032 1 1 20032 2 ,..., 2 2 ,..., 2i xix x x x−= = = = = , e

portanto a seqüência que maximiza S é 0 1 2003(2 ,2 ,...,2 ).

SEGUNDO DIA PROBLEMA 4 Seja M ={1, 2,…,49} o conjunto dos primeiros 49 inteiros positivos. Determine o maior inteiro k tal que o conjunto M tenha um subconjunto de k elementos em que não haja 6 números consecutivos. Para esse valor máximo de k, encontre a quantidade de subconjuntos de m, de k elementos, que tenham a propriedade mencionada. SOLUÇÃO Definição: Um conjunto A M⊆ é feliz se não contém seis inteiros consecutivos. Seja N M⊆ feliz, | | 42.N ≥ Então se ,| | 7.P M N P= − ≤ Logo a interseção de P com algum dos conjuntos

1 {1,2,3,4,5,6} A = 2, {7,8,9,10,11,12}

A = 3, {13,14,15,16,17,18}

A =

7 {37,38,39,40,41,42}A = 8, {43,44,45,46,47,48,49}A =

é vazia. Chame de i algum inteiro tal que iA P∩ =∅ . Então ,iA N⊂ absurdo, pois então N teria uma seqüência de seis inteiros consecutivos. Por outro lado, é óbvio que {6,12,18,24,30,36,42,48}N M= − é feliz.

Assim, o maior k tal que existe N M⊆ feliz é 41 (como se | | 42N ≥ então N não é feliz, | | 41;N ≤ e acabamos de exibir um exemplo para 41).

Seja N M⊆ feliz, 1 2 8| | 41, { ... }.N P M N n n n= = − = < < < Os oito elementos de P separam naturalmente N em nove conjuntos

1 9 i,..., , , , N 5i j iN N N N N N∩ =∅ = ≤∪ e cada iN composto apenas de números consecutivos. É fácil ver que o número de possíveis conjuntos N é o número de soluções de 1 2 9| | | | ... | | 41N N N+ + + = (pois estamos escolhendo o

Page 163: Eureka 2003

Sociedade Brasileira de Matemática

EUREKA! N°18, 2003

39

tamanho dos 'iN s ) onde | | 5iN ≤ e exigimos que os Ni estejam ordenados pelos seus menores elementos. Seja | | .i ia N= Seja 5 5 .i i i ib a b a= − ⇔ − = Como 0 5,0 5.i ia b≤ ≤ ≤ ≤ Substituindo na equação,

1 9 1 9 1 95 5 ... 5 41 45 ( ... ) 41 ... 4.ib b b b b b b− + − + + − = ⇔ − + + = ⇔ + + = Mas então a restrição 5ib ≤ é redundante, logo o número de ,| | 41,N M N N⊆ = feliz, é o número de soluções de 1 9... 4b b+ + = nos inteiros não negativos, que é

9 4 1 12 12 11 10 9 495. 4 4 24+ − ⋅ ⋅ ⋅

= = =

PROBLEMA 5 No quadrado ABCD, sejam P e Q pontos pertencentes aos lados BC e CD respectivamente, distintos dos extremos, tais que BP = CQ. Consideram-se pontos X e Y, X ≠ Y, pertencentes aos segmentos AP e AQ respectivamente. Demonstre que, quaisquer que sejam X e Y, existe um triângulo cujos lados têm os comprimentos dos segmentos BX, XY e DY. SOLUÇÃO

A 1

1

B

P

X a

C D

Y

y b

1 – λ

λ1 – λ

λ

c

Q

Seja 1,AB BC CD DA= = = = ,BP CQ λ= = ,BX a= ,XY b= .YD c=

Page 164: Eureka 2003

Sociedade Brasileira de Matemática

EUREKA! N°18, 2003

40

Como ,a b y BY+ ≥ = basta provar que y c> para demonstrar que

.a b c+ > Mas a mediatriz de BD é a reta AC , que divide o plano em dois

semiplanos Bβ ∋ e Dδ ∋ . À exceção do ponto A, todo o segmento AQ está contido em δ , logo Y y cδ∈ ⇔ > (o caso Y A≡ é trivial). Logo a b c+ > e, analogamente, .c b a+ > Basta provar que a c b+ > Seja

(0,0); (1,0); (1,1); (1, ); ( ,1); ( , );A B C P Q X x xα β α= = = = = =(0,1); ( , )D y y yβ= = , onde 1α β+ = (para que BP CQ= ).

Então 2 2 2( 1) ,a x xα= − + 2 2 2( 1) ,c y yβ= + − 2 2 2 2 2 2 2 2( ) ( ) 2 2b x y x y x xy y x xy yβ α β β α α= − + − = − + + − + =

2 2 2 2 2 2 2 2 2 2 22 ( ) ( ) .b x y x y xy x y x yα β α β α β= + + + − + = + + −

Mas 2 2 22b a c b a ac c< + ⇔ < + + (já que a, b, c são positivos). Basta demonstrar que 2 2 2 ,a c b+ ≥ já que 2ac > 0. Logo basta provar que

2 2 2 2 2 2 2 2 2 2 2 2( 1) ( 1) 2x x y y x y x xy yα β α β− + + − + ≥ + + − + ⇔ 2 2 2 22 1 2 1 2 0 (1 )(1 ) 0.x x y y x xy y x y⇔ − + + − + − + − ≥ ⇔ − − ≥ Mas

x AP∈ , logo [0,1] 1 [0,1].X x∈ ⇔ − ∈ Analogamente, 1 [0,1]y− ∈ e o resultado segue trivialmente. PROBLEMA 6 Definem-se as sucessões 0 0( ) ,( )n n n na b≥ ≥ por:

0 01, 4a b= = e 2001

1 + ,n n na a b+ = 20011n n nb b a+ = + para 0.n ≥

Demonstre que 2003 não divide nenhum dos termos destas sucessões. SOLUÇÃO Observe que 2003 é primo; logo, se

2002 2001 10 (mod 2003), 1(mod 2003) (mod 2003).x x x x−≡ ≡ ⇔ ≡/

Page 165: Eureka 2003

Sociedade Brasileira de Matemática

EUREKA! N°18, 2003

41

Suponha que exista n tal que 1 12003 | n na b+ + (note que

0 0 4 0(mod 2003)a b = ≡/ ); escolha o menor deles. Suponha que 12003 | na + (o outro caso é análogo). Temos 2001 1

10 (mod 2003),n n n n na a b a b−+≡ ≡ + ≡ +

logo 1 (mod 2003).n na b− ≡ − Como nem na nem nb são zero (mod 2003), podemos inverter os dois lados, obtendo

1 1 (mod 2003)n n n na b b a− −≡ − ⇔ ≡ − (usamos o fato de que 1( 1) 1).−− ≡ − Mas 1

1 0(mod 2003),n n n n nb b a a a−+ ≡ + ≡ − + ≡ logo 1 12003 | 2003 | .n nb a+ +⇔

É fácil ver que, nesse caso, 1( ) 1(mod 2003).n n n na b a a−≡ − ≡ − Seja 1nc a −= e

1nd b −= (para n = 0, temos 20011 1 4 5 0(mod 2003)a = + = ≡/ , logo podemos

supor 1).n ≥ Então 1na c d−≡ + e 1(mod 2003),nb c d −≡ + logo

1 1 1 1 12 ( ) (mod 2003).n na b c c c d dc dd cd cd− − − − −≡ + + + ≡ + + (Como n é

o menor possível, 2003 não divide i ia b para todo i n≤ , logo existe 1( )cd − ). Seja x = cd.

1 21 2 3 1 0(mod 2003).x x x x−− ≡ + + ⇔ + + ≡ Esta equação tem uma raiz

(que é cd), logo seu discriminante 23 4 1 1 5∆ = − ⋅ ⋅ = é resíduo quadrático (mod 2003) : de fato,

2 2(2 3) 4( 3 1) 5 5(mod 2003).x x x+ = + + + ≡ Mas pela lei de Reciprocidade

Quadrática, 2003 1 5 1

2 25 2003 ( 1)2003 5

− −⋅ = − ⇔

20025 3 5 5( 1) ( 1) 1 1,2003 5 2003 2003 ⋅ = − ⇔ ⋅ − = ⇔ =−

logo 5 não é resíduo

quadrático módulo 2003, absurdo! Assim, não é possível que

1 12003 | n na b+ + , logo {2003 | } ({ } { } ) .k k k kx x a b∈ ∈∈ ∩ ∪ =∅

Estas soluções da Olimpíada Ibero-americana de Matemática - 2003 foram redigidas por Fábio Dias Moreira de Rio de Janeiro – RJ.

Page 166: Eureka 2003

Sociedade Brasileira de Matemática

EUREKA! N°18, 2003

42

A DESIGUALDADE DE ERDÖS-MORDELL Anderson Torres, São Paulo - SP

♦ Nível Avançado

Neste artigo demonstraremos (várias vezes) a desigualdade de Erdös-Mordell e mostraremos uma bela aplicação na resolução do problema 5 da IMO de 1996, realizada em Mumbai, Índia.

1- Uma história do teorema “Considere um triângulo ABC e um ponto P do mesmo plano. Sejam CBA PPP ,, as projeções ortogonais de P nos lados BC, CA, AB respectivamente. Vale então a desigualdade:

CPBPAPPPPPPP CBA ++≤++ )(2

com igualdade se e somente se P for o circuncentro de um triângulo ABC eqüilátero”. Este é o enunciado da famosa Desigualdade de Erdös-Mordell. Ela foi inicialmente conjecturada pelo matemático húngaro Paul Erdös e demonstrada no mesmo ano por Louis Mordell, na revista American Mathematical Monthly (problema n° 3740). Logo após surgiram várias soluções e alguns artigos sobre a desigualdade, cada uma usando variadas técnicas: trigonometria (Louis J. Mordell e P.F. Barrow), desigualdades angulares e semelhanças (Leon Bankoff), teorema de Ptolomeu (André Avez e Hojoo Lee), áreas de polígonos (V. Komornik). Mostraremos algumas delas, acrescidas de um pequeno comentário:

Lema Importante: B CAP BC AB PP AC PP⋅ ≥ ⋅ + ⋅ ,

com igualdade se, e somente se, BCPP CB // . A esmagadora maioria das demonstrações difere apenas na demonstração desta pequena desigualdade. Veja que esta desigualdade equivale a estas desigualdades (as outras duas seguem por permutação cíclica das variáveis):

Page 167: Eureka 2003

Sociedade Brasileira de Matemática

EUREKA! N°18, 2003

43

B C

C A

A B

AB ACAP PP PPCB BCCB BABP PP PPAC CACA CBCP PP PPBA AB

≥ ⋅ + ⋅

≥ ⋅ + ⋅

≥ ⋅ + ⋅

Ao somá-las, obtemos:

A B CCA BA CB AB AC BCAB BP CP PP PP PPBA CA AB CB BC AC

+ + ≥ + ⋅ + + ⋅ + + ⋅

e lembrando que a soma de um real positivo com seu inverso não pode ser menor que 2, e esse valor só se iguala a dois se o número em questão for 1 (isto é conseqüência da desigualdade das médias), a desigualdade segue, com igualdade se e apenas se AB = BC = CA. Além disso, devemos ter //A BP P AB , //A CP P AC

e BCPP CB // , o que implica facilmente que P é o circuncentro do triângulo ABC. Vamos então demonstrar este lema! Demonstração 1: (trigonometria) O quadrilátero CBPPAP é cíclico, pois os ângulos retos são opostos e somam 180°. Assim, pela Sagrada Lei dos Senos Generalizada,

sen sen2

B C B C

B C

B C

P P P PAPP PP BAC

AP BC R P P

= = ⇔∠ ∠

⋅ = ⋅

em que R é o circunraio do triângulo ABC.

Page 168: Eureka 2003

Sociedade Brasileira de Matemática

EUREKA! N°18, 2003

44

A

P B

P C

C B P A

P

Pela Sagrada Lei dos Cossenos,

² ² ² 2 cos ( )² ² 2 cos ( )

( sen sen )²( cos cos )²

B C B C B C B C

B C B C A C A B

B B A C C A

B B A C C A

P P PP PP PP PP P PPPP PP PP PP P PP P PPPP P PP PP P PPPP P PP PP P PP

= + − ⋅ ⋅ ⋅ ∠

= + − ⋅ ⋅ ⋅ ∠ +∠

= ⋅ ∠ + ⋅ ∠+ ⋅ ∠ − ⋅ ∠

,

onde usamos o fato (bastante conhecido ☺): 1²cos²sen =+ xx para fatorar. Assim, jogando um dos parênteses fora, obtemos:

sen sensen sen

2

B C B B A C C A

B C B C

B C B C

P P PP P PP PP P PPP P PP ACB PP ABCAP BC R P P PP AB PP AC

≥ ⋅ ∠ + ⋅ ∠ ⇔≥ ⋅ ∠ + ⋅ ∠ ⇔

⋅ = ⋅ ≥ ⋅ + ⋅

A igualdade ocorre se, e somente se,

cos cos 02 sen cos 2 sen cossen( ) sen( ) sen( ) sen( );

B B A C C APP P PP PP P PPPAB ABC PAC ACB

PAB ABC PAB ABC PAC ACB PAC ACB

⋅ ∠ − ⋅ ∠ = ⇔⋅ ∠ ⋅ ∠ = ⋅ ∠ ⋅ ∠ ⇔∠ +∠ + ∠ −∠ = ∠ + ∠ + ∠ −∠

mas 180PAB ABC PAC ACB∠ +∠ +∠ +∠ = ° ,donde ( ) ( ),sen PAB ABC sen PAC ACB∠ +∠ = ∠ +∠ e

logo sen( ) sen( )PAC ACB PAB ABC∠ −∠ = ∠ −∠ PAB ABC PAC ACBPAB PAC ABC ACB

∠ −∠ = ∠ −∠ ⇔∠ −∠ =∠ −∠

Page 169: Eureka 2003

Sociedade Brasileira de Matemática

EUREKA! N°18, 2003

45

e (fica como exercício mostrar que) isto equivale, de fato, a BCPP CB // .

Demonstração 2: (áreas de paralelogramos)

Escolha dois pontos ABCACB ∈∈ 11 , e construa os paralelogramos 1'CAPC e

1' BAPB .

',' PCPB cortam BC em X,Y e 11CB em 11,YX respectivamente, caso P seja interno ao triângulo (mas isto não afeta muito a demonstração). Veja que

11 '' CCBB é um paralelogramo.

C 1

B

C ' B '

Y 1

Y

X 1

X C

B 1

P

A

P B P C

Por congruências, ]''[][ 11 CPBCAB = , em que [algo] significa área de algo. Agora, veja:

1 1 1 1 1 1 1 1 1

1 1 1 1 1 1 1 1

1 1 1 1 1 1 1 1

1 1 1 1

[ ] [ ] [ ' ] [ ' ] [ ' ] [ ' ][ ' '] [ ] [ ' ] [ ' ] [ ' ] [ ' ][ ' '] [ ' ] [ ' ] [ ' ] [ ' ][ ' ] [ ' ] [ ' ' ]

ABC PX Y C CY B B X APC C APB BPB C PX Y C CY B B X APC C APB BC Y X B C CY B B X APC C APB BAPC C APB B B B C C

− + + = + ⇔− + + = ++ + = ++ =

.

Com isto vemos que

1 1 1 1 1 1 1'C BAC PP AB PP B C C C AP B C⋅ + ⋅ ≤ ⋅ = ⋅ , com igualdade se, e apenas se,

111' CBCC ⊥ , ou 11CBAP ⊥ , ou seja, AP contém o circuncentro do triângulo

Page 170: Eureka 2003

Sociedade Brasileira de Matemática

EUREKA! N°18, 2003

46

1 1AB C . Fazendo ABABACAC == 11 , , teremos por congruências (para variar...)

11BCBC = , e (por paralelismo mesmo!☺) BCPP CB // , e pronto! Fim!

Observação: Veja que é possível modificar esta demonstração apenas usando uma reflexão pela bissetriz do ângulo ABC∠ para obter os pontos

ABABACAC == 11 , , ABCACB ∈∈ 11 , . Esta observação será útil mais tarde.

Demonstração 3: (teorema de Ptolomeu) Sejam ',' CB pontos da reta CB PP tais que CB PPCCBB ⊥'//' . Então é fácil ver que

' ' ' '' 'C C B B

C C B B

BC B C B P P P P CAP BC AP B P AP P P AP C P

≥ = + + ⇔⋅ ≥ ⋅ + ⋅ + ⋅

,

com igualdade se, e somente se, BCPP CB // .

B P A C

B ' P C

C '

P B

A

P

Vamos calcular cada uma das parcelas em relação ao ponto P. Veja que '~ CCPAPP BC ∆∆ pois os ângulos correspondentes são iguais. De fato,

temos os ângulos retos, e 'CCPPAPAPP BCBC ∠=∠=∠ (quadrilátero cíclico e ângulos opostos pelo vértice). Assim, obtemos as relações:

Page 171: Eureka 2003

Sociedade Brasileira de Matemática

EUREKA! N°18, 2003

47

' 'CBB C B

B

P PC P AP C P P P P CP C AP

= ⇔ ⋅ = ⋅ .

Analogamente, ' 'C B

C B CC

B P P P AP B P P P P BP B PA

= ⇔ ⋅ = ⋅

Pelo Teorema de Ptolomeu-Euler,

B C B C C BP P AP AP PP AP PP⋅ = ⋅ + ⋅

Adicionando as igualdades, obtemos:

( ' ')C C B B

C B C B B C B C

C B

AP BC AP B P P P P CP P P C P P AP P P AP P P P BPP AC PP AB

⋅ ≥ ⋅ + += ⋅ + ⋅ + ⋅ + ⋅

= ⋅ + ⋅

2- Problema 5, IMO 1996 (Mumbai, Índia)

“Seja ABCDEF um hexágono convexo tal que AB é paralelo a DE, BC é paralelo a EF, e CD é paralelo a FA. Sejam ECA RRR ,, os circunraios dos triângulos FAB, BCD, DEF respectivamente, e seja P o perímetro do hexágono. Prove que: 2/PRRR ECA ≥++ .” Este foi um dos problemas mais difíceis (e é considerado o mais difícil por muitos problemistas) já propostos na história da IMO. Para se ter uma idéia, apenas seis participantes (dois romenos e quatro armênios) fecharam este problema, enquanto os seis estudantes da equipe chinesa zeraram-no! Mostraremos neste artigo duas soluções. A primeira é um esboço de como foi criado o problema, segundo a Banca Examinadora da IMO de 1996 (o problema foi proposto pela Armênia), segundo a referência [Nairi M. Sedrakian, The History of a Creation of a 1996 IMO Problem, Mathematics Competitions, n° 2 vol.9], e se assemelha muito com a solução oficial, presente na Eureka! N° 11. A segunda (com algumas modificações), totalmente sintética, considerada a mais bela das soluções, é de autoria de Ciprian Manolescu, da equipe da Romênia, o único Perfect Score (também conhecido como Ouro-42) da IMO 1996.

Page 172: Eureka 2003

Sociedade Brasileira de Matemática

EUREKA! N°18, 2003

48

Solução 1: usaremos o seguinte lema (demonstre-o!): “Considere um triângulo de circunraio R, lados a e b, e o ângulo γ entre eles. Então, para quaisquer βα , tais que , 0, 2α π β α β γ π< > + + = , é válida a

desigualdade: sen sen2sen sen

R a bβ αγ γ

≥ ⋅ + ⋅ ”

(Sugestão: note que sen cos ,2πβ β = −

sen cos ,

2πα α = −

e considere uma

reta fazendo um ângulo 2πα − com o lado b; calcule a medida da projeção

ortogonal do lado do triângulo oposto ao ângulo γ nessa reta). Usando este lema, podemos estimar os raios. Para tal sejam

DEFBCDFAB ∠=∠=∠= γβα ,, , fEFeDEdCDcBCbABaFA ====== ,,,,, .

Com isto, sen sen sen sen sen sen2 ;2 ;2sen sen sen sen sen senA C ER b a R c d R e fγ β γ α α β

α α β β γ γ≥ ⋅ + ⋅ ≥ ⋅ + ⋅ ≥ ⋅ + ⋅

Vamos tentar obter outra estimativa para AR2 , desta vez em relação aos lados d e e. Pela Sagrada Lei dos Senos, 2 senABF R α= ⋅ . Podemos então escrever

sen senBF b aγ β≥ ⋅ + ⋅ . Veja que BF não pode ser menor que a distância entre as retas BC e EF. Olhando este fato, vamos projetar o ponto A nas retas BC, EF obtendo os respectivos pontos EFBC AA , . Analogamente para o ponto D,

obtemos o retângulo BCEFEFBC DDAA .

B

F

A

ABC c

β

α

AEF

a

f E

γ

DBC

d

DEF

e

D

C

b

Page 173: Eureka 2003

Sociedade Brasileira de Matemática

EUREKA! N°18, 2003

49

Com isto, podemos escrever:

sen sen

sen senBC EF BC EF BC EF BC EF

BF b aAA AA A A D D DD DDd e

γ β

β γ

≥ ⋅ + ⋅= + = = = += ⋅ + ⋅

Concluímos as seguintes desigualdades:

sen sen sen sen sen sen2 ;2 ;2sen sen sen sen sen senA C ER d e R a f R c bβ γ α γ β α

α α β β γ γ≥ ⋅ + ⋅ ≥ ⋅ + ⋅ ≥ ⋅ + ⋅

Somando tudo:

( )

( )

( )

4 ( )

sen sensen sen

sen sensen sen

sen sensen sen

A C ER R R

a d

b e

c f

α ββ α

α γγ α

β γγ β

⋅ + +

≥ + +

+ + +

+ + +

E o problema segue, aplicando a Desigualdade das Médias aos parênteses e dividindo tudo por 4. E fim!

Solução 2: Este problema, por si só, já incita o uso de Erdös-Mordell ou de alguma generalização conveniente (muito provavelmente até às ultimas conseqüências (☺)). Para tal, devemos de algum modo produzir a configuração deste teorema. Aproveitando o paralelismo, desenhe os paralelogramos MDEF, NFAB, PBCD. Com isto já temos algo dentro do hexágono (mesmo que não seja um ponto, como em Erdös-Mordell, mas já é alguma coisa... Às vezes é necessário um pouco de coragem para não desistir de algumas idéias, mesmo que pareçam não dar certo. Muitos problemas de IMO e vários problemas difíceis em geral são, na verdade, aplicações de fatos simples até às últimas conseqüências).

Page 174: Eureka 2003

Sociedade Brasileira de Matemática

EUREKA! N°18, 2003

50

A

F

B

M N

P

E

C

D X

Y

Z

O problema agora é tentar achar um modo de identificar os raios. Lembrando que raios e diâmetros têm tudo a ver com perpendicularidade, desenhe o triângulo XYZ, com , ,XFY FN YBZ BP ZDX DM⊥ ⊥ ⊥ . Assim, o quadrilátero FMDX é inscritível de diâmetro MX. Mas os triângulos FED e FMD são congruentes, logo 2 AXM R= ⋅ . Com isso o problema é demonstrar a seguinte desigualdade:

FNFMDMDPBPBNZPYNXM +++++≥++ .

Vamos dividir em dois casos:

1- M = N = P. E este caso é a própria Desigualdade de Erdös-Mordell. 2- O triângulo MNP existe (não é degenerado). A partir daqui vamos adaptar a demonstração de Erdös-Mordell. Estimaremos XM primeiro. Sejam Y’ e Z’ as reflexões dos pontos Y e Z em relação à bissetriz de YXZ∠ . Sejam G e H as projeções de M, X em Y’Z’ respectivamente. Como [XYZ] = [Y’XZ’] = [Z’MY’] + [XMZ’] + [Y’MX], temos:

YZ XH YZ MG ZX FM XY DM⋅ = ⋅ + ⋅ + ⋅ . Mas, usando a desigualdade triangular no triângulo XMG e a desigualdade cateto < hipotenusa no triângulo XHG (ou mesmo distância de X à reta Y’Z’), obtemos:

MGXHXMXHXGMGXM −≥⇒≥≥+

Substituindo na igualdade recém-descoberta, XY XZXM DM FMYZ YZ

≥ ⋅ + ⋅

Page 175: Eureka 2003

Sociedade Brasileira de Matemática

EUREKA! N°18, 2003

51

Analogamente, XY XZXM DM FMYZ YZ

YX YZYN BN FNXZ XZZX ZYZP BP DPXY XY

≥ ⋅ + ⋅

≥ ⋅ + ⋅

≥ ⋅ + ⋅

Somando tudo: XM YN ZP

XY XZDM FMYZ YZYZ YXFN BNXZ XZZX ZYBP DPXY XY

+ +

≥ ⋅ + ⋅

+ ⋅ + ⋅

+ ⋅ + ⋅

Agora falta pouco...Basta arranjar um modo de sumir com as frações. Agora vamos usar a Desigualdade das Médias para concluir. Para tal, outra estimativa. Primeiramente, veja que os triângulos XYZ e MNP são semelhantes, o que nos

permite definir ZX

DMDPYZ

BPBNXY

FNFMk −=

−=

−= . Com isto, podemos

escrever:

2

2

( )

ZX YXBP BNXY XZ

XZ XY BN BPXY XZ

XZ XY BN BPXY XZ

XZ YZ XY YZBP BN kXY XZ

⋅ + ⋅

+ = + +

− −

⋅ ⋅ ≥ + − ⋅ −

Analogamente,

Page 176: Eureka 2003

Sociedade Brasileira de Matemática

EUREKA! N°18, 2003

52

( )

( )

( )

XZ XY XZ YZ XY YZBP BN BP BN kXY XZ XY XZYX ZY XY XZ XZ YZDM DP DM DP kZY YX YZ XYXZ YZ YZ XY XZ XYFM FN FM FN kYZ XZ XZ YZ

⋅ ⋅ ⋅ + ⋅ ≥ + − ⋅ −

⋅ ⋅ ⋅ + ⋅ ≥ + − ⋅ −

⋅ ⋅ ⋅ + ⋅ ≥ + − ⋅ −

Agora, basta somar estas desigualdades e acabamos o problema!! REFERÊNCIAS : [1]A demonstração de Hojoo Lee pode ser encontrada na famosa revista Forum Geometricorum, a qual você pode ler no site http://forumgeom.fau.edu .Neste artigo você encontra as referências de toda a história deste problema enquanto ele se passava na American Mathematical Monthly. [2]Na lista de discussão de problemas da OBM ([email protected]; ver também www.obm.org.br/lista.htm) foi deixada, há algum tempo atrás, a demonstração de Ciprian Manolescu. [3]Na Internet tem uns livros do Kiran Kedlaya.Vá ao site abaixo e faça o download: http://www.unl.edu/amc/a-activities/a4-for-students/problemtext/ Um deles trata sobre desigualdades, e outro sobre geometria euclidiana plana. Ainda tem uns dois livros com provas de algumas olimpíadas de matemática de várias partes do mundo. [4]Após uma longa caça achei este artigo,que trata de uma generalização interessante: A weighted Erdös-Mordell Inequality for Polygons. Este livro pode ser encontrado no endereço: www.math.technion.ac.il/~shafrir/pub_ps/m18.ps.gz [5]Um site de divulgação cientifica: http.://mathworld.wolfram.com [6]Rafael Tajra Fonteles - Trigonometria e desigualdades em problemas de olimpíadas, Eureka! 11, p. 24-33. [7]A segunda demonstração da desigualdade de Erdös-Mordell também foi objeto de uma questão da fase final da OPM-2001. Confira no livro da OPM-2001 ou no site http://www.opm.mat.br/

Page 177: Eureka 2003

Sociedade Brasileira de Matemática

EUREKA! N°18, 2003

53

COMO É QUE FAZ?

PROBLEMA 3 PROPOSTO POR DAVI MÁXIMO ALEXANDRINO NOGUEIRA (FORTALEZA – CE) É possível escolher 102 subconjuntos com 17 elementos cada do conjunto {1,2,3...102} tais que a interseção de quaisquer 2 deles tem no máximo 3 elementos? SOLUÇÃO A ideia é olhar para o plano projetivo P sobre Z/17Z, que é o quociente de (Z/17Z)3\{(0,0,0)} pela relação de equivalência ~ ,x y y a x⇔ = ⋅

17 \{0}.a∃ ∈ P tem 172 + 17 + 1 = 307 pontos e as retas em P têm 18 pontos cada (e duas delas sempre se intersectam num ponto). Vamos fazer uma espécie de quociente de P. Para isso, considere o isomorfismo linear T de (Z/17Z)3 dado por T(x, y, z) = (y, z, x). Note que T3 = Id. As retas em P são dadas por vetores não nulos w de (Z/17Z)3 (de fato por elementos de P): uma reta Rw é o conjunto dos v tais que < v, w > = 0. Note que T tem um único ponto fixo em P: o elemento v0 = [1:1:1], correspondente ao vetor (1,1,1). Jogamos v0 fora e dividimos os outros 306 pontos de P em 102 classes de equivalência de 3 elementos (as orbitas de T): a classe de equivalência de u é {u, Tu, T(Tu)}. Temos 307 retas em P. Jogamos fora a

0.vR O quociente de cada uma das outras retas tem 17 elementos. De fato, se Rw

contém pontos da forma u e Tu, devemos ter <u, w> = 0 e < u, T*w> = <Tu, w> = 0. Como w não é v0 então T*w = T–1(w) não é múltiplo de w, donde há apenas um elemento de P satisfazendo essas duas igualdades, isto é, apenas dois pontos em Rw são identificados pela nossa equivalência, donde as retas (projetadas pelo quociente por essa relação de equivalência) têm agora 17 pontos cada. Para cada reta na projeção, existem exatamente 3 retas (em P) que se projetam sobre ela: Rw, RTw e ( )T TwR . Dadas duas retas na projeção, elas se intersectam em (no máximo) 3 pontos: se elas são as projeções de Rw e Rv, suas interseções serão as projeções das interseções de Rw com Rv, de RTw com Rv e de RT(Tw) com Rv. Assim, na projeção (ou, se você preferir, no quociente), temos 102 pontos e 102 retas, cada uma com 17 elementos, sendo que duas delas se intersectam em (no máximo) 3 pontos.

Page 178: Eureka 2003

Sociedade Brasileira de Matemática

EUREKA! N°18, 2003

54

SOLUÇÕES DE PROBLEMAS PROPOSTOS Publicamos aqui algumas das respostas enviadas por nossos leitores.

81) Num triângulo isósceles ABC com AB = BC, temos AC = BH, onde BH é a

altura relativa ao lado AC. Traçamos uma reta BD que corta o prolongamento da reta AC em D de tal forma que os raios dos círculos inscritos nos triângulos ABC e CBD são iguais. Determine o ângulo ˆABD.

SOLUÇÃO DE FRANCISCO JARDEL ALMEIDA MOREIRA (FORTALEZA – CE)

O1 O2

C H A F E D

B

X

Sejam, O1 o incentro do ABC∆ , O2 o incentro do BCD∆ , 1 2X BC OO= ∩ ,

2E BO AD= ∩ e finalmente, F o pé da perpendicular de 2O até AD .

Note que 1 2OO FH é um retângulo, pois 1 2 90O HF O FH= = ° e 1 2O H O F= ,

daí 1 2 // .O O AD Agora observe que:

1 1 2 2 180HCO BCO O CB DCO+ + + = ° ademais, 1 1 2 2 e HCO BCO O CB DCO= =

portanto, 1 2 90O CO = ° . Do fato de 1 2 90O CO = ° e 1 2 // ,O O AD X é médio de

1 2O O (de fato, 1 1 1,X O C HCO X CO= = donde 1XC XO= e logo CX é

mediana do triângulo retângulo 1 2O CO )

é médio de .HE Logo, .HB AC HE AC HB= ⇒ = =

Page 179: Eureka 2003

Sociedade Brasileira de Matemática

EUREKA! N°18, 2003

55

Como 45BHE = ° e assim 2 90 .ABD H BE= ⋅ = ° 82) a) Demonstre a identidade

1

10

(2 )cos( ) cos(2 ) cos(4 )...cos(2 ) cos(2 )2 ( )

nnn j

nj

sensen

αα α α α αα

+

+=

⋅ ⋅ = =⋅∑

b) Prove que 20

1 1 1 1 1 1 1 1 1 2... cos .2 2 2 2 2 2 2 2 2 2 j

j

ππ

+=

⋅ + ⋅ + + = =

SOLUÇÃO DE WALLACE ALVES MARTINS (RIO DE JANEIRO – RJ) a) Demonstraremos a identidade utilizando o princípio da indução.

Para n = 0 temos cosα e 0+1

0 1

sen(2 ) sen2 2sen cos cos .2 sen 2sen 2sen

α α α α αα α α+ = = = . Assim que a

propriedade é válida para n = k, temos: 1

10

sen(2 )cos(2 ) .2 sen

kkj

kj

ααα

+

+=

=∏ Multiplicando-se ambos os membros da identidade por

1cos(2 )k α+ temos: 1

1 1 ( 1) 11

1 1 ( 1) 10

sen(2 2 )sen(2 )cos(2 ) sen(2 )2cos(2 ) .

2 sen 2 sen 2 sen

k

k k kkj

k k kj

αα α αα

α α α

+

+ + + ++

+ + + +=

= = =∏

Portanto a propriedade também é válida para n = k + 1. Logo, pelo princípio da indução segue-se que a identidade é válida , 0.n n∀ ∈ ≥ b) Demonstraremos primeiramente a identidade entre o primeiro membro e o membro central da identidade acima; por indução.

Para n = 0, temos que 0 2

1cos cos .2 4 2π π+

= =

Assim a propriedade é

verdadeira para n = 0. Vamos mostrar que para todo k temos

20

( 1) Radicais

1 1 1 1 1 1 1 1 1... ... cos .2 2 2 2 2 2 2 2 2 2

k

jj

k

π+

=

+

⋅ + ⋅ ⋅ + + + =

∏ Para isso, vamos

Page 180: Eureka 2003

Sociedade Brasileira de Matemática

EUREKA! N°18, 2003

56

mostrar que para todo k temos 2

( 1) Radicais

1 1 1 1 1... cos .2 2 2 2 2 2k

k

π+

+

+ + + =

Considere

( 2) Radicais

1 1 1 1 1...2 2 2 2 2

k

x

+

= + + + temos que 2 22 2

1 1 cos cos 2 1.2 2 2 2k kx xπ π

+ + − = ⇔ = −

Como sabemos 2cos(2 ) 2cos 1.α α= − Logo 2

32cos cos

2 2k

kx

ππ+

+

= =

satisfaz a equação. Logo ( 1) 2

[( 1) 1] Radicais

1 1 1 1 1... cos .2 2 2 2 2 2 k

k

x π+ +

+ +

= + + + =

Portanto a

identidade também é válida para n = k + 1. Logo, pelo Princípio da Indução segue-se que a identidade é válida , 0.n n∀ ∈ ≥ Em particular, quando n→∞ temos a identidade conforme acima.

Agora, sabemos que 2 20

cos cos cos cos ... cos2 4 8 16 2

n

j nj

π π π π π+ +

=

= ⋅ ⋅ ⋅ ⋅ =

2 10

cos cos ... cos cos cos(2 ),2 2 8 4

nj

n nj

π π π π α+ +=

= ⋅ ⋅ ⋅ ⋅ =

∏ onde 2 .2n

πα +=

Utilizando (a) temos:

12

1 102 2

sen 2 ,12cos(2 ) .

2 sen 2 sen2 2

nn n

j

n njn n

π

απ π

++

+ +=+ +

= =

Sabemos que, quando n→∞ então 122 sen .

2 2n

n

π π++

⋅ →

(Pois 0

senlim 1)x

xx→

= .

Logo: 210

2

1 2lim cos lim .2 2

2

n

jn n njnsen

ππ π+→∞ →∞ +=+

= =

Page 181: Eureka 2003

Sociedade Brasileira de Matemática

EUREKA! N°18, 2003

57

83) Seja {0,1,2,3,...}.=N Determine quantas funções :f →N N satisfazem f(2003) = 2003, f(n) ≤ 2003 para todo n ≤ 2003 e f(m + f(n)) = f(f(m)) + f(n), para todo m, n ∈ N. 84) Prove que se * {1,2,3,...}A⊂ =N é um conjunto não-vazio tal que

4n A n A∈ ⇒ ∈ e n A ∈ então *A=N .

Obs. x é o único inteiro tal que 1x x x− < ≤ 85) Mostre que todo triângulo pode ser dividido em 9 pentágonos convexos de

áreas iguais. 86) Encontre todas as triplas de inteiros positivos (a, m, n) tais que 1ma + divide

( 1) .na+ 87) Seja a(1) = 1 e, para cada inteiro n ≥ 2, a(n) igual ao menor inteiro positivo

que não pertence a {a( j), j < n} tal que 1

( )n

ja j

=∑ seja múltiplo de n. Prove que

( ( ))a a n n= para todo inteiro positivo n.

88) Prove que se r∈ e cos( )r π⋅ ∈ então 1 1cos( ) 1, ,0, ,1 .2 2

r π ⋅ ∈ − −

Seguimos aguardando as soluções dos problemas: 83, 84, 85, 86, 87 e 88…

Page 182: Eureka 2003

Sociedade Brasileira de Matemática

EUREKA! N°18, 2003

58

PROBLEMAS PROPOSTOS

Convidamos o leitor a enviar soluções dos problemas propostos e sugestões de novos problemas para os próximos números.

89) Uma prova de múltipla escolha com n questões é feita por k alunos. Uma

resposta correta na i-ésima questão vale pi pontos, onde pi é um inteiro positivo, para 1 .i n≤ ≤ A nota de cada aluno é a soma dos pontos correspondentes às questões que ele acertou. Após a realização a prova, foi observado que, mudando os pesos pi, as notas dos alunos podem estar em qualquer uma das k! possíveis ordens (em que não há duas notas iguais). Dado n, qual é o maior valor possível de k?

90) Prove que, para todo inteiro positivo n e para todo inteiro não nulo a, o

polinômio 1 2 ... 1n n nx ax ax ax− −+ + + + − é irredutível, i.e., não pode ser escrito como o produto de dois polinômios não constantes com coeficientes inteiros.

91) Um jardinero deve construir um canteiro com a forma de setor circular. Ele

dispõe de 100 metros de fio para cercá-lo. Figura:

r

r

Qual deve ser o valor do raio do círculo para que o canteiro tenha área máxima? Qual é a área máxima? 92) Seja (Fn)n∈N a seqüência de Fibonacci, definida por F1 = 1, F2 = 1 e

2 1 , .n n nF F F n+ += + ∀ ∈ Prove que mdc ( , )( , )m n mdc m nF F F= para quaisquer inteiros positivos m e n.

Page 183: Eureka 2003

Sociedade Brasileira de Matemática

EUREKA! N°18, 2003

59

93) Um inteiro positivo n é dito perfeito se n é igual à soma dos divisores positivos de n que são menores que n. Prove que um número par n é perfeito se e somente se existe um número primo 2p≥ tal que 2 1p − é primo e

12 (2 1)p pn −= − . 94) A ilha das amazonas é habitada por amazonas e homens.

As amazonas mandam em tudo, são inteligentíssimas, ciumentíssimas e muito fofoqueiras. O que uma amazona mais gosta de fazer é trair outra amazona com o marido desta. Consumada a traição, ela conta o seu feito a todas as amazonas da ilha menos à amazona traída. As outras amazonas também não contam nada à vítima da traição. Mas se uma amazona descobre que está sendo traída ela mata o seu marido na próxima meia noite. A rainha das amazonas, que é viúva, vê esta situação com desagrado. Ela vê que há traição na ilha mas, como nunca ninguém descobre nada, nenhum marido morre. No dia 1 de janeiro de 3333, então, contrariando a tradição, ela chama todas as amazonas para a praça central e faz uma proclamação solene: "Há traição nesta ilha." Nenhuma amazona sonha em duvidar da palavra da rainha e todas as amazonas sabem disso. Como já foi dito, todas são inteligentes e ciumentas: estes e os outros fatos mencionados neste enunciado até aqui são conhecimento comum entre as amazonas. Supondo que haja 1000 amazonas na ilha e que 365 delas tenham sido traídas, o que acontecerá?

95) "Resta-Um" é um jogo de tabuleiro na qual as peças ocupam um tabuleiro

formando parte de um reticulado retangular (na verdade, existem variações em tabuleiros de reticulado triangular). O único movimento permitido consiste em tomar duas peças em casas adjacentes vizinhas a uma casa vazia, e fazer a peça mais distante da casa vazia pular sobre a outra peça, ocupando a casa vazia. A peça pulada é retirada.

(esse movimento pode ser feito para a direita, para a esquerda, para cima ou para baixo). Agora imagine um tabuleiro que é um reticulado retangular infinito e uma reta que contém uma linha do reticulado, dividindo-o em dois lados. Todas as casas de um dos lados da linha estão vazias e cada casa do outro lado da linha pode ou não ter uma peça.

Page 184: Eureka 2003

Sociedade Brasileira de Matemática

EUREKA! N°18, 2003

60

Quantas peças, no mínimo, precisamos para chegar a uma casa do lado vazio do tabuleiro, a uma distância n da linha ? Abaixo indicamos uma casa a distância n, para n = 1,2,3,4,5.

54321

… …… …… …… … … … … … …

Você sabia…

Que 209960112 1− é primo? Este é o maior primo conhecido, tem 6320430 dígitos e foi descoberto por Michael Shafer, um participante do GIMPS (um projeto distribuído para procurar primos de Mersenne. Veja: http://www.mersenne.org para mais informações). Agora são conhecidos 40 expoentes p para os quais 2 1p − é primo (e portanto 12 (2 1)p p− − é perfeito - veja o problema proposto 93): 2, 3, 5, 7, 13, 17, 19, 31, 61, 89, 107, 127, 521, 607, 1279, 2203, 2281, 3217, 4253, 4423, 9689, 9941, 11213, 19937, 21701, 23209, 44497, 86243, 110503, 132049, 216091, 756839, 859433, 1257787, 1398269, 2976221, 3021377, 6972593, 13466217 e 20996011.

Problema 89 proposto na 27 Olimpíada Russa de Matemática em 2001, problema 90 proposto na Olimpíada Romena de Matemática de 1992, problema 91 proposto por Osvaldo Mello Sponquiado (Ilha Solteira – SP).

Page 185: Eureka 2003

Sociedade Brasileira de Matemática

EUREKA! N°18, 2003

61

Você sabia… Que existem infinitos inteiros positivos ímpares k tais que

2 1 nk ⋅ + é composto para todo n∈ ? Tais inteiros k são chamados números de Sierpinski. Em 1962, John Selfridge provou que 78557 é um número de Sierpinski, e conjectura-se que seja o menor deles. Atualmente há 11 números menores que 78557 sobre os quais não se sabe se são números de Sierpinski ou não: 4847, 10223, 19249, 21181, 22699, 24737, 27653, 28433, 33661, 55459 e 67607. O número 5359 fazia parte dessa lista até 6/12/2003, quando Randy Sundquist ( um participante do Seventeen or Bust, um projeto distribuído para atacar o problema de Sierpinski) encontrou o primo

50545025359 2 1⋅ + , que tem 1521561 dígitos e é o quarto maior primo conhecido, e maior primo conhecido que não é de Merssenne. Veja: http://www.seventeenorbust.com para mais informações. Exercício: Prove que 78557 é um número de Sierpinski, e que existem infinitos números de Sierpinski a partir das congruências

01278557 0 ≡+⋅ (mod 3) 01278557 1 ≡+⋅ (mod 5) 01278557 7 ≡+⋅ (mod 7) 01278557 11 ≡+⋅ (mod 13)

3 3978557 2 1 78557 2 1 0⋅ + ≡ ⋅ + ≡ (mod 73) 01278557 15 ≡+⋅ (mod 19) 01278557 27 ≡+⋅ (mod 37).

Page 186: Eureka 2003

Sociedade Brasileira de Matemática

EUREKA! N°18, 2003

62

COORDENADORES REGIONAIS Alberto Hassen Raad (UFJF) Juiz de Fora – MG Amarísio da Silva Araújo (UFV) Viçosa – MG Ana Paula Bernardi da Silva (Universidade Católica de Brasília) Brasília – DF Benedito Tadeu Vasconcelos Freire (UFRN) Natal – RN Carlos Frederico Borges Palmeira (PUC-Rio) Rio de Janeiro – RJ Claus Haetinger (UNIVATES) Lajeado – RS Cleonor Crescêncio das Neves (UTAM) Manaus – AM Élio Mega (Colégio Etapa) São Paulo – SP Florêncio Ferreira Guimarães Filho (UFES) Vitória – ES Gil Cunha Gomes Filho (Colégio ACAE) Volta Redonda – RJ Ronaldo Alves Garcia (UFGO) Goiânia – GO Reginaldo de Lima Pereira (Escola Técnica Federal de Roraima) Boa Vista – RR Ivanilde Fernandes Saad (UC. Dom Bosco) Campo Grande– MS Jacqueline Fabiola Rojas Arancibia (UFPB) João Pessoa – PB Janice T. Reichert (UNOCHAPECÓ) Chapecó – SC João Benício de Melo Neto (UFPI) Teresina – PI João Francisco Melo Libonati (Grupo Educacional Ideal) Belém – PA José Carlos dos Santos Rodrigues (Unespar) Campo Mourão – PR José Cloves Saraiva (UFMA) São Luis – MA José Gaspar Ruas Filho (ICMC-USP) São Carlos – SP José Luiz Rosas Pinho (UFSC) Florianópolis – SC José Vieira Alves (UFPB) Campina Grande – PB Licio Hernandes Bezerra (UFSC) Florianópolis – SC Luzinalva Miranda de Amorim (UFBA) Salvador – BA Mário Rocha Retamoso (UFRG) Rio Grande – RS Marcelo Rufino de Oliveira (Grupo Educacional Ideal) Belém – PA Marcelo Mendes (Colégio Farias Brito, Pré-vestibular) Fortaleza – CE Pablo Rodrigo Ganassim (Liceu Terras do Engenho) Piracicaba – SP Ramón Mendoza (UFPE) Recife – PE Raúl Cintra de Negreiros Ribeiro (Colégio Anglo) Atibaia – SP Reinaldo Gen Ichiro Arakaki (INPE) SJ dos Campos – SP Ricardo Amorim (Centro Educacional Logos) Nova Iguaçu – RJ Sérgio Cláudio Ramos (IM-UFRGS) Porto Alegre – RS Tadeu Ferreira Gomes (UEBA) Juazeiro – BA Tomás Menéndez Rodrigues (U. Federal de Rondônia) Porto Velho – RO Valdenberg Araújo da Silva (U. Federal de Sergipe) São Cristovão – SE Valdeni Soliani Franco (U. Estadual de Maringá) Maringá – PR Vânia Cristina Silva Rodrigues (U. Metodista de SP) S.B. do Campo – SP Wagner Pereira Lopes (CEFET – GO) Jataí – GO